Anda di halaman 1dari 111

Tim Akademik ANANTARA 2015

Divisi SL (Soal Latihan)


Fakultas Kedokteran Universitas Padjadjaran

ANANTARA
FAMILY MEDICINE – UTS

1. A 15-year-old girl bring by her family to the primary health care. The family reports she
has a very short temper, argue with them and teachers at school, often refuses request at
both settings and usually blames her teachers in her parents for the behaviors she is
showing. She denies being depressed, anxious, or any psychotic symptoms. What might
be the best diagnosis?
a) Condust Disorder
b) Oppositional Defiant Disorder
c) Antisocial Personality Disorder
d) Attention Deficit Hyperactivity Disorder
e) Post traumatic Stress Disorder.

2. According to Kazdin & Weisz (2003, teaching parents to identify and reward positive
behavior also helps to prevent parents from focussing on the negative and disruptive
behaviours exhibited by children with both ADHD and conduct disorder. This can be
achieved through?
a) Time-out from positive reinforcement
b) Systemic family therapy
c) Behavior management techniques
d) Parent training programs

3. It is mostly patriarchical in nature and everything is often decided by the father. Parents
who use this style have a strict set of rules and expectations, if rules are not followed it
ends up with punishment. There is usually no explanation of giving the punishment just
that the children are in trouble and should listen accordingly. What style of this parenting
pattern?
a) Authoritarian
b) Authoritative
c) Permissive
d) Uninvolved
e) Neglected

4. A 35-year-old woman came to the doctor to consultation for having baby. She was not
pregnant yet after got marriage 4 year ago. This couple was not having any serious
problem in reproductive system. Her BMI was 29 and her lipid profile and blood glucose
is never in normal limit. Which of the following condition that most likely found in this
patient that can influence her reproductive quality?
a) Anatomical abnormality of reproductive system
b) Hypercholesterolemia due to HPA axis
Tim Akademik ANANTARA 2015
Divisi SL (Soal Latihan)
Fakultas Kedokteran Universitas Padjadjaran

c) Atherosclerosis
d) Blood glucose due to insulin resistence
e) Abnormal meal patern

5. An 18-month-old boy with 13.6 kg body weight. This is at the 95th percentile for his
height and sex. His mother was concerned because his family has many members who
are overweight and she worries that the child's future health is at risk. The subject of
language development among toddler arises. At what age do you alert the parents to
expect the child to begin to speak in sentences of 2 to 3 words with a 50-word
vocabulary?
a) 15-18 months
b) 18-26 months
c) 2-3 years
d) 3-4 years
e) 4-5 years

6. Vision of IAPB (International Agency of Prevention of Blindness is a world in which no


one is needlessly blind or visually impaired and where those with unavoidable vision loss
can achieve their full potential. IAPB collaborate with WHO, NGO, and eye care
professional launched Vision 2020. Which of the following is trends and projections of
global blindness in 2020 if vision 2020 not implemented?
a) 45 millions.
b) 38 millions.
c) 76 millions.
d) 69 millions.
e) 62 millions.

7. A 40-year-old-man came with occasional episodes of headache. And he also complained


blurred vision since two days ago. His father died due to hypertension and stroke. He
enjoyed high cholesterol food and very rare to go for exercise. He is found to have a
blood pressure of 170/90. Which of the following complication that most likely
occurred?
a. Fever
b. Stroke
c. Vomitus
d. Headache
e. Orbital pain

8. Strategy for childhood obesity prevention, except :


a. Promote healthy nurtrition and activity
b. Consumed large sugar
c. Reduce time watching television
d. Enhance infrastructure supporting bicycling and walking
e. Increase support for breastfeeding
Tim Akademik ANANTARA 2015
Divisi SL (Soal Latihan)
Fakultas Kedokteran Universitas Padjadjaran

9. IAPB promotes the global initiative Vision 2020 the right to sight.Which of the
following is the aims of Vision 2020 the right to sight?
a) A world in which no one is needlessly blind or visually impaired and where those
with unavoidable vision loss can achieve their full potential.
b) To eliminate the main causes of avoidable blindness by the year 2020.
c) Brings together governments and NGO to facilitate the planning, development, and
implementation of sustainable national eye care programmes.
d) Based on the three core strategies of disease control, human resource development,
and infrastructure development.
e) To eliminate blindness by the year 2020

10. Which the following drug should be avoided to be given with Salbutamol?
a) Theophylin
b) Amoxcycillin
c) Ephedrin
d) Diltiazem
e) Simvastatin

11. An 18-month-old boy with 13.6 kg body weight. This is at the 95th percentile for his
height and sex. His mother was concerned because his family has many members who
are overweight and she worries that the child's future health is at risk.If the family
success in this stage, they will enter the following stage?
a. Pre-school age family
b. Family with toddlers
c. School age family
d. Family with teenagers
e. Family with launching young adults

12. A young mother brings to you her 18-month-old boy who weights 13.6 kg. This is at the
95thpercentile for his height and sex. She is concerned because her family has many
members who are overweight and she worries that the child's future health is at risk.The
subject of language development arises. At what age do you alert the parents to expect
the child to begin to speak in sentences of 2 to 3 words with a 50-word vocabulary?
a. 15-18 months
b. 18-26 months
c. 2-3 years
d. 3-4 years
e. 4-5 years

13. A 75-year-old had been diagnosed with stroke infarction. He still has communication
disorder and weakness of right extremities as a sequele. He lived with his wife and son
that has been married and has a job with 3 years old boy. He bed ridden only. He was a
Tim Akademik ANANTARA 2015
Divisi SL (Soal Latihan)
Fakultas Kedokteran Universitas Padjadjaran

retired government officer. He cannot work. He was often looked angry because of
communication difficulty.What is the most functional problem of the patient?
a. Impairment.
b. Disability.
c. Handicap.
d. Impairment and disability.
e. Impairment, disability and handicap.

14. A 55-year-old obese male has been diagnosed as diabetes mellitus type 2. His doctor told
him about his management plan. Beside pharmacology treatment, he also has to modify
his lifestyle especially about routine exercise. The doctor told him that exercise can help
him to decrease body weight and increase cardiovascular fitness.Which of the following
statement that most appropriate reason for that management plan?
a) Insulin stimulates the liver and muscles to take in excess glucose
b) Exercise stimulates the insulin to change fat deposit into glucose
c) Exercise can stimulate the insulin sensitivity
d) Intense exercise stimulates stress hormone
e) Exercise can increase his heart rate

15. Most of parents unclear when they give parenting for their children. Some of them
believed the best time for parenting begins from birth.What is the correct timing that the
parents should give parenting to their children?
a) Birth - Adolescent
b) Birth - Young adult
c) Conception - Adolescent
d) Conception - Young adult
e) Conception - Middle-old adult

16. A 23-year-old man weighs 81,5 kg at a height of 1,78 m. the patient has no family
history of cardiovascular disease, hypertension and there is no dyslipidemia in his or his
family's history. He has never smoked. He is concerned about his weight because his
mother was significantly overweight and died because of heart attack; and his father is
modesty overweight. His total cholesterol is 240 mg/dl, LDL is 110 mg/dl and
triglyceride s 130 mg/dlWhat screening should be recommended for him?
a) Blood pressure, every 2 year
b) Blood pressure, every year for people with hypertension
c) Hyperlipidemia: total cholesterol at age 45-65 years old
d) Colorectal cancer: DRE and FOB
e) Prostate cancer: DRE

17. Everyone can do Endurance activities, which the benefit is?


a. Improve the health of the heart, lungs, and circulatory system
b. These exercises can increase body weight.
c. This exercises help prevent falls.
Tim Akademik ANANTARA 2015
Divisi SL (Soal Latihan)
Fakultas Kedokteran Universitas Padjadjaran

d. This exercise helps keep the body limber.


e. This exercise helps keep the body flexible.

18. Mr. and Mrs. Purcell, Mary, 21, and Bob, 17, came to the family meeting suggested by
Dr. M. After Dr. M. shared her findings, Mary spoke first and expressed concern about
her father's "long-standing" health problems. She blamed her mother for "not taking
better care of him." Bob quickly defended his mother, saying she had been working very
hard and had "a lot on her mind." Bob became upset with Mary for "attacking" their
mother. Mrs. Purcell told Dr. M. that her husband had health complaints for as long as
she had known him. Mr. Purcell then said that his chest pains were worse since the last
doctor's appointment.Family APGAR score=3.
Who did act in a coalition in this family?
a) Mary and Bob
b) Mr. Purcell and Bob
c) Mrs. Purcell and Bob
d) Dr. M and Mrs. Purcell
e) Mr. and Mrs. Purcell

19. This style consists of following the same rules as the authoritarian parents. With having
strict rules and expectations however there is more open communication with parents and
children. When children have problems with rules and they are broken these parents tend
to be more receptive. They monitor instead of trying to rule the child's life they are less
restrictive parents but still assertive.What type of this parenting style?
a) Authoritarian
b) Authoritative
c) Permissive
d) Uninvolved
e) Neglected

20. An 8-year-old boy is referred to Primary Health Care for an evaluation of behavioral
difficulties in school. His teachers report that he is unable to sit still and to complete
class work because he is so easly distracted. The boy's mother reports that he has always
had a lot of energy. She says that preparing to leave fot school in the morning is
extremely difficult because of her son's disorganization and forgetfulness. Otherwise, she
denies that her son produces any repetitive movements disorders. Which of the following
is the most likely diagnosis ?
a) Conduct Disorder
b) Oppositional Deficit Disorder
c) Attention Deficit Hyperactivity Disorders
d) Autistic Disorder
e) Disruptive Behavior Disorder not otherwise specified
Tim Akademik ANANTARA 2015
Divisi SL (Soal Latihan)
Fakultas Kedokteran Universitas Padjadjaran

21. Vision 2020 is partnership among WHO, IAPB, Ministries of Health, NGDOs,
Professions, and Corporations. In a country it will be implemented in National Vision
2020 bodies.Which of the following is the service units of Vision 2020?
a) In district level.
b) In province level.
c) In county level.
d) There is not service units for Vision 2020.
e) In primary health service.

22. A 40-year-old male came to his family doctor with headache. His blood pressure was
150/90 mmHg. The family doctor then does head to toe physical examination, laboratory
examination and other supportive examination. The doctor then decides to treat him with
supportive therapy for headache and lifestyle modification including exercise.Which of
the following that most appropriate reason for the management plan?
a. Age is < 60 year old
b. There is no renal insufficiency
c. There is no target organ damage
d. No family history of hypertension
e. Cardiovascular condition within normal limit

23. There is not much structure here for children, and parents often do not set rules or have
guidelines for the child. They do not have many expectations for the children, they avoid
conflict and are more nurturing to the child. They are more lenient when it comes to
misbehaviour and often do not punish the children for wrong doing. Children under this
parenting style have a hard time communicating with parents about things they found
important to them.What type of this parenting style?
a. Authoritarian
b.Authoritative
c. Permissive
d.Uninvolved
e. Neglected

24. Mr. and Mrs. Purcell, Mary, 21, and Bob, 17, came to the family meeting suggested by
Dr. M. After Dr. M. shared her findings, Mary spoke first and expressed concern about
her father's "long-standing" health problems. She blamed her mother for "not taking
better care of him." Bob quickly defended his mother, saying she had been working very
hard and had "a lot on her mind." Bob became upset with Mary for "attacking" their
mother. Mrs. Purcell told Dr. M. that her husband had health complaints for as long as
she had known him. Mr. Purcell then said that his chest pains were worse since the last
doctor's appointment.Family APGAR score=3. When a positive relationship between any
two members of a system occurred, what is condition represent in the family?
a. Coalition
b. Hierarchy
c. Role
Tim Akademik ANANTARA 2015
Divisi SL (Soal Latihan)
Fakultas Kedokteran Universitas Padjadjaran

d. Alliance
e. Influence

25. Billy, a 6-year-old male is brought by his parents. Billy was known cannot remain still
and frequently running around. He also did the same at his school, he cannot remain in
his seat for more than a few minutes, cannot focus his attention on one task and is easily
distracted. The other children are annoyed by his inability to wait his turn when playing
games and his intrusion during conversations. Which of the following is the most likely
diagnosis?
a. ADHD
b. Delinquency
c. Conduct disorder
d. Oppositional disorder
e. Antisocial personality disorder

26. A 16-year-old boy writes a family history of mental retardation presents with long
standing poor school performance and aggresive behavior toward peers.Which of the
following test is use to determine in this boy?
a) Minnesota Multiphse Personality Inventory (MMPI
b) Folstein Mini Mental States Examination (MMSE
c) Mental Status Examination (MSE
d) WAIS-R
e) Beck Depression Inventory

27. A 6-year-olds diagnosed with thalassemia who require blood transfusion the child on an
ongoing basis. You have managed to inform his parents.What is Communication
techniques should given to the child?
a. Simply tell the child's parents
b. Give hope to the child
c. Provide age-appropriate information, the development and understanding
d. Inform other treatment alternatives
e. Provide information only as needed.

28. What is the correct measurement to assess quantitative food intake?


a. Food frequency questionnaire.
b. Food recall
c. Periodic body weight measurement
d. Dietary history
e. Nutritional survey

29. A 35-year-old female with pre-diabetes came to the doctor and ask an advice about her
exercise program that most suitable for her. His BMI, physical examination and blood
glucose level is normal.
Which of the following exercise program that most appropriate?
Tim Akademik ANANTARA 2015
Divisi SL (Soal Latihan)
Fakultas Kedokteran Universitas Padjadjaran

a. 30 minutes a day, 3 times a week, mild intensity jogging or swimming


b. 30 minutes a day, 2 times a week, mild intensity jogging or swimming
c. 60 minutes a day, 2 times a week, high intensity weight training
d. 60 minutes a day, 3 times a week, moderate intensity treadmill
e. 30 minutes a day, 3 times a week, mild intensity of static cycling

30. A young medical doctor was doing home visit to a patient, a 65-year-old man, post
stroke infarction. He was bedridden and he looked grimace if the right shoulder is
moved. Four weeks ago, he has been diagnosed stroke infarction and been hospitalized
for that. He also had history of hypertension and diabetes mellitus for 15 years ago but
didn't control regularly. He got antihypertension drug from the doctor after hospitalized
and he took it only if he felt headache. He always ate "sate kambing" in the weekend. He
was a retired government officer and had health insurance.
Which of the following anti platelet drugs that is the most appropriate for this patient?
a) Heparin.
b) Warfarin.
c) Streptokinase.
d) Aspirin.
e) Dipyridamol.

31. A 56-year-old female with type 2 diabetes mellitus came to the doctor and ask an advice
about her exercise program that most suitable for her. Except for her BMI (obese, her
physical examination and blood glucose level is normal.
Which of the following exercise program that most appropriate?
a) 30 minutes a day, 3 times a week, mild intensity jogging or swimming
b) 30 minutes a day, 2 times a week, mild intensity jogging or swimming
c) 60 minutes a day, 2 times a week, high intensity weight training
d) 60 minutes a day, 3 times a week, moderate intensity treadmill
e) 30 minutes a day, 3 times a week, mild intensity of static cycling

32. Rani is 26-year-old woman, G2P1A0, present in the initial visit with a 4-week history of
fatigue and nausea. She had a positive pregnancy test 2 weeks ago.Rani has been on a
multivitamin preparation with folic acid, she has been immunized against Hepatitis B
within the past 3 years. From physical examination, she is a well appearing, mildly
overweight. Vital signs and other examination are within normal limit. Your assessment
is intrauterine pregnancy.What is your plan for her?
a) Amniocentesis
b) Ultrasound for identify the sex of her baby
c) Stop multivitamin, change to Iron tablet
d) Diet, exercise and sexually activity discussed
e) At least follow up 3 times during her pregnancy

33. A 6-year-old boy is referred to General Hospital by his parents to evaluate his difficulty
with reading.it is suspected that he suffers from reading disorder, but a subtle impairment
Tim Akademik ANANTARA 2015
Divisi SL (Soal Latihan)
Fakultas Kedokteran Universitas Padjadjaran

in vision or hearing must be ruled out as a causative or contributing factors.Learning


disorders are most commonly associated with which of the following disorders?
a) Attention Deficit Hyperactivity Disorders
b) Bipolar Disorder
c) Schizophrenia
d) Tourette
e) Aspager

34. Mr. and Mrs. Purcell, Mary, 21, and Bob, 17, came to the family meeting suggested by
Dr. M. After Dr. M. shared her findings, Mary spoke first and expressed concern about
her father's "long-standing" health problems. She blamed her mother for "not taking
better care of him." Bob quickly defended his mother, saying she had been working very
hard and had "a lot on her mind." Bob became upset with Mary for "attacking" their
mother. Mrs. Purcell told Dr. M. that her husband had health complaints for as long as
she had known him. Mr. Purcell then said that his chest pains were worse since the last
doctor's appointment.Family APGAR score=3.
Based on Family APGAR score, what is the characteristic of this family?
a) Radiate a sense of integrity
b) Share power while negotiating decisions.
c) Cope relatively well with adversity
d) Creating coercive power differentials
e) Coming out of a crisis stronger for the experience

35. A 50-year-old male with hypertension stage 2 now have management plans from his
family doctor. The management plans include lifestyle modification such as exercise. He
told the doctor that he always does sport. He was playing badminton twice a week, but
uncertain about the duration because he must did it alternately with his other friends. The
doctor told him that that kind of sport is not enough for improving his health condition.
Which of the following that most appropriate reason for doctor explanation?
a. Badminton is not aerobic exercise
b. His sport activity is not done regularly
c. There is no weight training in badminton
d. Lack of frequency
e. Badminton make him prone to injury

36. A 5-year old girl was brought to the office because she is shorter than other girls at her
age. On examination you found, she was less than fifth percentile on growth curves, even
when you correct for mid parental height. There are multiple potential causes for short
stature. She reveals that she speaks only six-word sentences but they are intelligible.
What screening would you recommend for a girl?
a. Blood screening (hemoglobinopathy, phenylketonuria, congenital hypothyroidism,
HIV
b. Hearing
c. Fluoride to prevent dental cavitation
Tim Akademik ANANTARA 2015
Divisi SL (Soal Latihan)
Fakultas Kedokteran Universitas Padjadjaran

d. Injury prevention
e. Breast feeding

37. Refer to the genogram below:

Interpret a genogram above from biopsychosocial perspective!


a. Index patient has DM
b. Index patient has risk for hypertension
c. N died because of old age
d. R has risk for obesity

38. A mother brings her child 5-year-old male child to the hospital. His mother reported that
he is shorter than his friends. Other developments within normal limits. He is the
youngest child. His older siblings are normalheight.His mother was 165 cm and his
father was 170 cm height.Which of the following diagnostic tools and criteria that most
appropriate for this patient?
a. Weight for age below 2 SD at the WHO growth curves
b. Height for body weight below 2 SD at the WHO growth curves
c. BMI for age below 2 SD at the WHO growth curves
d. Head of circumference for age below 2 SD on the growth curves

39. Mrs. B, 48-year-old women comes to your clinic for the consultation about her lump in
her breast.
Dr. F : The result suggest that is not just an 'ordinary' lump
Mrs. B : This sounds like bad news.
Dr. F : I was hoping to be able to reassure you. It seems that some of the cells we
looked at were abnormal. It is important, though, that we picked this up early.
Mrs. B : Are you saying it's the big 'C'?
Dr. F : Yes, it probably is cancer.
Mrs. B : We all know what that means...................
As a doctor, what would you say to show empathy to Mrs. B?
Tim Akademik ANANTARA 2015
Divisi SL (Soal Latihan)
Fakultas Kedokteran Universitas Padjadjaran

a. I realize this probably comes as a shock to you


b. I believe that you can cope the problem
c. I know is very frightening for you
d. I felt so awful that you have this diagnosis
e. I sorry for you, I can feel your feel.

40. Bill is a 72-year-old retired driver who was admitted to your practice because of back
pain. He has been a smoker for 30 years and smokes 15 cigarettes every day. Bill lives
alone in a small house and has lost about 5 kg of weight in the past few months. His diet
has deteriorated since his wife passed away two years ago. Bill's daughter accompanies
him to visit his GP, as she is concerned about his recent weight loss.His current medicine
history includes ramipril 5 mg once daily in the morning for well-controlled
hypertension. He has no significant family medical history. On examination Bill's blood
pressure is 115/78 mmHg. A bone mineral density test T-score in the hip is -1.8 and in
the spine is -2.7.What information would you provide to patient regarding "physical
activity"?
a. Low-impact weight-bearing exercises
b. Low-impact weight-bearing exercises combined with high-intensity strength
(resistance training
c. Low-impact weight-bearing exercises combined with low-intensity strength
(resistance training
d. High-impact weight-bearing exercises

41. Boy 13-year-old boy brought to your clinic by his parents with complaint of physical
aggression to other children including his younger sister. The school was complained to
the parents because he regularly gets into trouble at school for bullying and truants from
school and also stealing from school's cafetaria. He was a nice boy but his behaviour
start to change and became worse since 2 years ago. Boy comes from a high
socioeconomic family. He lives with his parents, his sister, his grandmother and
grandfather, his aunt and his nephews. His father is a harsh and very punitive. Her
mother gives more attention to his younger sister.. His grandmother wants him to be a
doctor and always encourages him to study hard.Which of the following is the most
likely diagnosis?
a. ADHD
b. Conduct disorder
c. Mood disorder in adolescent
d. Oppositional deviant disorder
e. Antisocial personality disorder

42. Menu planning is very important to fulfill protein and energy requirement to prevent and
treat malnourished patient. We have to determine the nutritional requirements before
formulating the menu.
Which of the following steps that most appropriate?
Tim Akademik ANANTARA 2015
Divisi SL (Soal Latihan)
Fakultas Kedokteran Universitas Padjadjaran

a. Calculate total calorie requirement - calculate protein requirement - calculate fat


requirement - calculate carbohydrate requirement.
b. Calculate total calorie requirement - calculate protein requirement - calculate
carbohydrate requirement - calculate fat requirement.
c. Calculate total calorie requirement - calculate carbohydrate requirement - calculate
protein requirement - calculate fat requirement.
d. Calculate total calorie requirement - calculate carbohydrate requirement - calculate
fat requirement - calculate protein requirement.
e. Calculate total calorie requirement - calculate fat requirement - calculate
carbohydrate requirement - calculate protein requirement.

43. A 5-year old girl was brought to the office because she is shorter than other girls at her
age. On examination you found, she was less than fifth percentile on growth curves, even
when you correct for mid parental height. There are multiple potential causes for short
stature. She reveals that she speaks only six-word sentences but they are intelligible.
What the family influence could be a potential cause for her condition?
a. Psychological and social characteristics of parents
b. Spreading of infectious diseases among family members
c. Parental deprivation
d. Klinefelter syndrome
e. Environmental condition of the family

44. A 70-years-old woman with sleeping disorders comes to family physician and based on
the result from anamnesis, physical examination and laboratory examination found that
all value in the within normal limit.
Which of the following sentence is the most correct concept for the case above?
a. Handicap concept
b. Injury concept
c. Pathological concept
d. Social culture concept
e. Psychological concept

45. You are counseling an expectant couple (Woman: P0A0 regarding prevention care
should be performed. They are not planning for pregnant at least in the next 3 months.
No history of smoking, alcohol intake, previous disease and family hereditary diseases.
What kind of prevention care should be performed at this time?
a. Administration of MMR, Varicella and Hepatitis B vaccines.
b. Starting prenatal vitamins and/or folic acid supplementation
c. Education on the benefit of breast feeding
d. Performing amniocentesis
e. Administration of Tetanus Toxoid vaccine

46. To do continuous aerobic activity, we can use the Target Heart Rate Methods if?
a. The patient has a pacemaker in the heart.
Tim Akademik ANANTARA 2015
Divisi SL (Soal Latihan)
Fakultas Kedokteran Universitas Padjadjaran

b. The patient has irregular heart rhythm.


c. The patient takes medication that changes the heart rate.
d. The patient is overweight but the ECG is normal
e. The patient have atrial fibrillation

47. Mr. and Mrs. Purcell, Mary, 21, and Bob, 17, came to the family meeting suggested by
Dr. M. After Dr. M. shared her findings, Mary spoke first and expressed concern about
her father's "long-standing" health problems. She blamed her mother for "not taking
better care of him." Bob quickly defended his mother, saying she had been working very
hard and had "a lot on her mind." Bob became upset with Mary for "attacking" their
mother. Mrs. Purcell told Dr. M. that her husband had health complaints for as long as
she had known him. Mr. Purcell then said that his chest pains were worse since the last
doctor's appointment.Family APGAR score=3
Based of Family APGAR, when the patient expressed "the satisfaction attained in
solving problems by communicating", what are you asking regarding the characteristics
of the family?
a. Adaptation
b. Partnership
c. Growth
d. Affection
e. Resolve

48. A 50-year-old woman claims she is having heart problem. She already had
echocardiogram 2 weeks ago with normal result, but she insisted that perhaps there was
a mistake, and ask you for more examination for her cardiac condition. Your physical
examination is within normal limits.
Which of the following is the most likely diagnosis?
a. Malingering
b. Hypochondriasis
c. Conversion disorder
d. Factitious disorder
e. Somatization disorder

49. Which the following drug cannot be given with antacid?


a. Amoxcycillin
b. Propranolol
c. Salbutamol
d. Glibenclamid
e. Cyprofloxacin

50. A 58-year-old male with nephropathy diabetes want to start exercise program. He takes
insulin for controlling his blood glucose level. The doctor plan to do blood glucose
monitoring before, during and after exercise before arrange his exercise program.
Which of the following reason that most appropriate for the blood glucose monitoring?
Tim Akademik ANANTARA 2015
Divisi SL (Soal Latihan)
Fakultas Kedokteran Universitas Padjadjaran

a. Exercise has insulin-type effect


b. Exercise can alter the glucagon level
c. Carbohydrate will abruptly decrease during exercise
d. Exercise dosage is depend on the glucose blood level after exercise
e. Exercise for nephropathy diabetes is only can be done under doctor monitoring

51. During his life time career of 40 years, a doctor will spend approximately 150.000 to
200.000 patient contacts and communication. Regarding to that issues, what is the
definition of communication?
a. Communication is a process in the context of the doctor-patient relationship
b. Communication is a process by which an individual transmitted stimuli (usually
verbal to modify the behavior of other individual
c. Communication is a process between doctor and patient that make the patient cure
d. Communication is a process to make the environment in the hospital became hostile.
e. Communication is a process to build a good condition of the patient

52. A 15-year-old girl gets into fights, intimidates others, steals, skips school, resulting in
falling grades. She does not exhibit any symptom for her behavior. She denies having
any depressive symptoms and says she feels pretty good about herself.
Which of the following is the most likely diagnosis?
a. Oppositional Defiant Disorder
b. Conduct Disorder
c. Anti Social Personality Disorder
d. Borderline Personality Disorder
e. Post Traumatic Stress Disorder

53. A young mother brings to you her 18-month-old boy who weights 13.6 kg. This is at the
95th percentile for his height and sex. She is concerned because her family has many
members who are overweight and she worries that the child's future health is at risk.
Your best advice is which of the following?
a. She should commit the child to a weight-reduction program forthwith
b. The child’s weight-related health is not an issue until he reaches adolescence
c. The child should be committed to a weight-loss program but not before he reaches
the age of 2 years
d. A child’s obesity-related health issues involve vascular disease, diabetes and joint
health only, and these problems do not begin until adulthood
e. Obesity in early childhood is not a health issue

54. You are deciding on preventive screening for a 16-year-old-girl who began menarche at
age 13. He cycles are every 30 days, with 4 days of moderate flow. Her last menstrual
period was 2 weeks ago. She denies ever being sexually active.
At what age should Pap smears be performed on her?
a. At age 16
b. At age 17
Tim Akademik ANANTARA 2015
Divisi SL (Soal Latihan)
Fakultas Kedokteran Universitas Padjadjaran

c. At age 20
d. At the age of first sexual activity
e. At the age of regular sexual axtivity

55. To do exercise with Safety Endurance activity?


a. It must be breath so hard that you cannot talk.
b. Should not cause dizziness.
c. Must reach maximal heart rate.
d. Not recommended for disabilities people.
e. Need to warm up and cool down (example: easy walking.

56. Balance diet from quality and quantity can support the optimal growth and development
What is the most cause of combination between animal and plant protein food sources is
better than plant/animal food sources?
a. Animal food sources is more expensive
b. Animal food sources consist more of saturated fat
c. Plant food sources need more quantity to eat
d. Balance protein and fat content with phytochemical substance
e. Plant food sources consist incomplete mineral content

57. Ampicillin and Paracetamol cannot be mixed in one preparation because of the following
reason?
a. Ampicillin is a causative drug and paracetamol is a symptomatic drug
b. Ampicillin need to be given in different time with paracetamol therefore it require to
be written in different prescription
c. Paracetamol can be given with dexamethasone in one preparation
d. Ampicillin can be given with paracetamol in one preparation because both has to be
given before meal
e. Ampicillin can only be written in one prescription and as antibiotic the
administration has to be with meal

58. The correct signature for Ampicillin as an antibiotic is?


a. S .3.d.d.Cap 2. 1. ha.c
b. S. 4.d.d.Cap 1. 1 h p.c
c. S .4.d.d. Cap 1. 2 h p.c
d. S. 3. d. d Cap 1. 2 h a.c
e. S. 3. d.d. Cap 1. 2 h.p.c

59. A 45-year-old woman returns to you for routine preventive care, having not seen you
except for episodic problems for the past 6 years. Her family history is unremarkable
except for the mention of colon polyps in her sister, who is 10 years older than her. Her
weight is 55 kg at a height of 165 cm. she is nonsmoker.
Which of the following would be the most appropriate combination of screening test for
her?
Tim Akademik ANANTARA 2015
Divisi SL (Soal Latihan)
Fakultas Kedokteran Universitas Padjadjaran

a. ECG, CBC, liver and kidney function test


b. Pap smear, mammogram, colorectal cancer screening, CBC
c. Chest X ray, Pap smear, EKG and comprehensive chemical profile
d. Lipid profile, FOBT, mammogram and blood sugar
e. Chest X ray, pulmonary function, lipid profile and CBC

60. A 50-years-old man with chronic cough, especially at night and during the rainy season
refused go to Puskesmas because he thought the treatment not work well. He should go
to Puskesmas many times and the cough becomes worse especially if the weather feel
cold.
Which of the following sentence is the most correct for health care utilization for the
case above
a. The used of transitional medicine
b. The used of conventional medicine
c. The used of cosmopolitan medicine
d. The used of traditional medicine
e. The used of modern medicine

61. Dr. Rio, a family physician plans to do a clinical audit for his clinical practice setting. He
selects a structure of care to be audited.
Which of the followings could be audited in term of structure of care?
a. Operating time
b. Management of patients
c. Mortality
d. Health indices
e. Patient satisfaction

62. A 60-year-old obese female has been diagnosed as diabetes mellitus type 2. Her doctor
told her about her management plan. Beside pharmacology treatment, she also has to
modify her lifestyle especially about routine exercise. The doctor told her that exercise
can help her to decrease body weight and increase cardiovascular fitness.
The exercise can be effective to decrease the body weight if the duration is :
a. 15 minute
a. 20 minute
b. 30 minute
c. 60 minute
b. 120 minute

63. Young woman develops severe headache, but she does not appear as distraught as would
be expected. Her full physical and neurological examination reveals no abnormality. This
happened after her grandmother passed away. She was with her grandmother since
toddler, her parents was working abroad. Her parents put aside their work to focus on
their daughter's illness.
What is the most appropriate diagnosis?
Tim Akademik ANANTARA 2015
Divisi SL (Soal Latihan)
Fakultas Kedokteran Universitas Padjadjaran

a. Malingering
b. Hypochondriasis
c. Conversion disorder
d. Factitious disorder
e. Somatization disorder

64. A 64-year-old woman, P4A2, Mrs. Yusi visited you with severe pain in hip area. The
pain is constant, boring sometimes sharp with infrequent electric-shock like sensation
radiating to left foot.She was diagnosed as cervical cancer stadium IIIB patient and
receiving chemo radiotherapy. Complaintaccompanied bynausea,vomiting, and dyspnea
forthe last3 days. She has not received any medication for relieving pain.
The aim of palliative care is to provide integrated and comprehensiveness care to prevent
and relieve all aspects of a patient's suffering.What is the different between pain and
suffering?
a. Pain is experienced by persons
b. Pain is part of suffering
c. Suffering is experience by a body and mind
d. Suffering is physically and psychologically pain

65. Vision 2020 was launched in 1999. There are some progress towards 2020 for cataract,
trachoma, and onchocerciasis.
Which of the following is not the characteristic of 10 years of success for cataract?
a. Low cost IOLs.
b. Training in ECCE initially and now SICS.
c. High volume surgery in S.E Asia region.
d. Sustainability models.
e. Training of ophthalmoc nurses in surgery.

66. What relatively common medical condition should be identified and addressed during
preconception care?
a. Diabetes, hypertension, seizure disorder, connective tissue disorder
b. Hypertension, anemia, diabetes
c. Diabetes, hypertension, seizure disorder, anemia
d. Hypertension, anemia, leukorhoe

67. An 18-month-old boy with 13.6 kg body weight. This is at the 95th percentile for his
height and sex. His mother was concerned because his family has many members who
are overweight and she worries that the child's future health is at risk.
A couple now prepare to have a new member of the family. The wife was 8-month
pregnant.
Which of the following developmental task will be raised in this stage?
a. Adapting to the critical needs and interests of preschool children in stimulating,
growth promoting ways
b. Coping with energy depletion and lack of privacy as parents
Tim Akademik ANANTARA 2015
Divisi SL (Soal Latihan)
Fakultas Kedokteran Universitas Padjadjaran

c. Encouraging children’s educational achievement


d. Establishing a mutually satisfying marriage
e. Having, adjusting to and encouraging the development of children

68. Mrs. Irna, 30-year-old woman work as a secretary at an oil company and diagnosed with
pharyngitis and given amoxicillin without asking about her allergy condition. Later on,
she return to the doctor with rash all over her body.
Which of the following sentences is right for next therapy planning?
a. Stop amoxicillin, replace with ampicillin tablet 500mg 4 times a day + antihistamine
b. Stop amoxicillin, replace with Erythromycin capsule 250mg 4 times a day, +
antihistamine
c. Stop amoxicillin, replace with Erythromycin capsule 500mg 2 times a day, +
antihistamine
d. Stop amoxicillin for a while, give antihistamin, then continue amoxicillin after the
rash disappear.
e. Add antihistamine without stopping or replacing amoxicillin.

69. A 48-year-old man has manifested BP levels of 150/100, 145/95 and 170/105 on
consecutive separate days over a 3-week period. He has a family history of deaths by
stroke and renal failure
If the patient was suggested to do physical exercise, which of the following is a valid
reason to terminate an exercise session?
a. The systolic BP rises and the diastolic falls during strenuous exercise
b. The pulse rate doubles during strenuous exercise
c. The patient experiences persistent headache during exercise
d. The patient is fatigued during strenuous exercise
e. Patient develops tachypnea

70. Bill is a 72-year-old retired driver who was admitted to your practice because of back
pain. He has been a smoker for 30 years and smokes 15 cigarettes every day. Bill lives
alone in a small house and has lost about 5 kg of weight in the past few months. His diet
has deteriorated since his wife passed away two years ago. Bill's daughter accompanies
him to visit his GP, as she is concerned about his recent weight loss.His current medicine
history includes ramipril 5 mg once daily in the morning for well-controlled
hypertension. He has no significant family medical history. On examination Bill's blood
pressure is 115/78 mmHg. A bone mineral density test T-score in the hip is -1.8 and in
the spine is -2.7.
What is your diagnosis?
a. Osteopenia
b. Osteomalacia
c. Osteoporosis
d. Osteoarthritis
e. Low Back Pain
Tim Akademik ANANTARA 2015
Divisi SL (Soal Latihan)
Fakultas Kedokteran Universitas Padjadjaran

71. Mr. H is an accountant aged 40 years. He comes to the accident and emergency
department following and attack of chest pain. He is seen first by Dr. A.
Dr. A : I see from your notes that you have had some chest pain. Do you still have the
pain?
Mr. H : No, not now.
Dr. A : Was it tight or dull?
Later, Mr. H is seen by Dr. I.
Dr. I : I understand that you have had pain. Would you please tell me more about it?
Mr. H : Well, it was in my chest and it come on when I was sitting at my desk. It was a
funny dull pain that stayed in the middle of my chest. I've had it a few times recently,
always when I'm, at work.
Based on the history above, what is the limitation of a closed style of questioning?
a. Some of the information may not be relevant
b. The information obtained is restricted to the question asked
c. More relevant information can be obtain in a given time
d. The interview may take longer and more difficult to control
e. Recording answers may be more difficult

72. A 10-year-old boy came to a clinic with chief complain he have a big body. Result of
the examination is he have BMI 32kg/m2.
The diagnosis of this patient is:
a. Normal
b. Overweight
c. Obesity
d. Underweight
e. B and c is correct

73. A 48-year-old man has manifested BP levels of 150/100, 145/95 and 170/105 on
consecutive separate days over a 3-week period. He has a family history of deaths by
stroke and renal failure.
Which of the following is the examination to obtain before starting the patient on the
chosen drug therapy?
a. ECG
b. Blood urea nitrogen
c. Electrolytes
d. Complete blood count
e. Liver function test

74. You are seeing a 28-year-old woman who complained having abdominal pain,
constipation, diarrhea and headache as well as symptoms of vague in swallowing. The
abdominal examination is negative for deep and rebound tenderness. Others are normal.
She lives with her husband and three children. She had been complaining all symptoms
since 2 years ago.
Tim Akademik ANANTARA 2015
Divisi SL (Soal Latihan)
Fakultas Kedokteran Universitas Padjadjaran

What the following statement is correct?


a. Your diagnosis is depression
b. Modern technology can diagnose her disease
c. Understanding natural systems continuum is needed to treated her
d. Refer the patient to gastroenterologist
e. Elaborate her past medical condition is necessary to treated her

75. A 40-years-old woman with mental disorder use "jamu" since 2 years ago after she went
to hospital many times but her family thinks the condition not became well.
Which of the following sentence is the most correct for health care utilization for the
case above?
a. The used of transitional medicine
b. The used of conventional medicine
c. The used of cosmopolitan medicine
d. The used of traditional medicine
e. The used of modern medicine

76. A 56-year-old female with proliferative retinopathy and controlled type 2 diabetes
mellitus came to the doctor and ask an advice about her exercise program that most
suitable for her. Except her visual acuity, her body weight, physical examination and
blood glucose level is normal.
Which of the following exercise program that most appropriate?
a. 30 minutes a day, 3 times a week, mild intensity jogging or swimming
b. 30 minutes a day, 2 times a week, mild intensity jogging or swimming
c. 60 minutes a day, 2 times a week, high intensity weight training
d. 60 minutes a day, 3 times a week, moderate intensity treadmill
e. 30 minutes a day, 3 times a week, mild intensity of static cycling

77. Mrs. Partina, a shop assistant age 27, attended medical outpatients at her local hospital.
Here is her story :
Tim Akademik ANANTARA 2015
Divisi SL (Soal Latihan)
Fakultas Kedokteran Universitas Padjadjaran

"When I went into the room, which was big and bare, I felt lost. I didn't know where to
sit, the doctor had his head down and was writing, the nurse was on the telephone and
there were some medical students talking to each other. I waited around and wanted to
run out the door. After what seemed like ages the doctor told me to sit down and asked
what was wrong. I didn't know what his name and I'm not sure that he knew mine. I'd
been thinking about my problems and what I wanted to tell the doctor- but I forgot it all-
he didn't seem very interested anyway. Hope I don't have to go again".
Regarding to the story above, what should the doctor do when beginning an interview?
a. Asked the patient problems
b. Telling your own name
c. Make a comfortable setting
d. Listen carefully
e. Encourage the patient to be relevant

78. A 48-year-old female with stage 2 hypertension and no cardiovascular disease need
exercise prescription for her family doctor. She was feeling uncomfortable about
exercise.
Which of the following that most appropriate exercise prescription for her?
a. 20-30 minutes/day continuous aerobic activity at maximum heart rate 50-65%
b. Light-moderate intensity until evaluated and conditioned with periodic monitoring
c. Light-moderate start with 20-30 minutes lower intensity/day with periodic
monitoring
d. 60 minutes/day of aerobic exercise without periodic monitoring
e. Start with 60 minutes/day of resistance training

79. A 17-years-old woman with minor skin disorders feel uncomfortable with her condition
because she will celebrate her birthday next week.
Which of the following sentence is the most correct for health care utilization usually
used for the case above?
a. The used of transitional medicine
b. The used of conventional medicine
c. The used of cosmopolitan medicine
d. The used of traditional medicine
e. The used of modern medicine

80. A young medical doctor was doing home visit to a patient, a 65-year-old man, post
stroke infarction. He was bedridden and he looked grimace if the right shoulder is
moved. Four weeks ago, he has been diagnosed stroke infarction and been hospitalized
for that. He also had history of hypertension and diabetes mellitus for 15 years ago but
didn't control regularly. He got antihypertension drug from the doctor after hospitalized
and he took it only if he felt headache. He always ate "sate kambing" in the weekend.
He was a retired government officer and had health insurance.
Which of the following impact of the illness to the family?
a. Bad communication.
Tim Akademik ANANTARA 2015
Divisi SL (Soal Latihan)
Fakultas Kedokteran Universitas Padjadjaran

b. Dependency.
c. Cost of treatment.
d. Risk factor modification.
e. Disability.

81. Communication in not just 'being nice' but produces a more effective consultation fot
both patient and doctors.To gain an effective communication, what is the effect of Good
Communication skill?
a. Patients are less likely to complain
b. Patients will cure
c. Patients are sympathy to the doctor
d. Patients will have a professional action
e. Patients are more likely to comply with symptom

82. For women who had previous child with an Neural Tube Defect (NTD how much folic
acid supplementation recommended before preconception?
a. 400 µg
b. 10 mg
c. 4 mg
d. 1 mg

83. A 5-year-old girl was brought to the office because she is shorter than other girls at her
age. On examination you found, she was less than fifth percentile on growth curves, even
when you correct for midparental height. There are multiple potential causes for short
stature.
She reveals that she speaks only six-word sentences but they are intelligible.
The parents are concerned about her development as well. Which of the following
responses is most appropriate?
a. Refer the child for specialized evaluation of development
b. Arrange a conference with the child’s teacher
c. Order a magnetic resonance image of the child’s brain
d. Reassure the parents that the child is developing within normal limits
e. Review the milestones of growth and development in this child

84. A 16-year-old female with overweight was doing consultation. The doctor was trying to
determine her nutritional status.
Which of the following reference that most appropriate?
a. CDC Growth chart
b. WHO Growth chart
c. NCHS Growth chart
d. Graphic of disease risk factor
e. DEPKES graphic
Tim Akademik ANANTARA 2015
Divisi SL (Soal Latihan)
Fakultas Kedokteran Universitas Padjadjaran

85. Bill is a 72-year-old retired driver who was admitted to your practice because of back
pain. He has been a smoker for 30 years and smokes 15 cigarettes every day. Bill lives
alone in a small house and has lost about 5 kg of weight in the past few months. His diet
has deteriorated since his wife passed away two years ago. Bill's daughter accompanies
him to visit his GP, as she is concerned about his recent weight loss.His current medicine
history includes ramipril 5 mg once daily in the morning for well-controlled
hypertension. He has no significant family medical history. On examination Bill's blood
pressure is 115/78 mmHg. A bone mineral density test T-score in the hip is -1.8 and in
the spine is -2.7.
What is your recommendation for him to reduce risk of fracture?
a. Hormone replacement therapy
b. Hip protective padding
c. Gait and balance training
d. Prescribing high dose vitamin D

86. Balanced diet is needed to prevent malnutrition. In balanced diet planning we have to
determine appropriate total calorie requirement.
What aspect do we have to consider in determining total calorie requirement?
a. Food processing.
b. Body posture.
c. Socio-economic condition.
d. Appetite.
e. Age.

87. A 35-year-old woman came to the doctor to consultation for having baby. She was not
pregnant yet after got marriage 4 year ago. This couple was not having any serious
problem in reproductive system. Her BMI was 29 and her lipid profile and blood glucose
is never in normal limit.
What is the most priority management of nutritional aspect in the infertility problem?
a. Decrease cholesterol level.
b. Decrease blood pressure
c. Insulin resistance management
d. Increase physical activity
e. Decrease carbohydrate intake.

88. How is the mechanism of hypertension caused by anxiety?


a. Behavioral effect
b. Physical effect
c. Emotional effect
d. Cognitive effect
e. Social effect
Tim Akademik ANANTARA 2015
Divisi SL (Soal Latihan)
Fakultas Kedokteran Universitas Padjadjaran

89. You are deciding on preventive screening for a 16-year-old-girl who began menarche at
age 13. He cycles are every 30 days, with 4 days of moderate flow. Her last menstrual
period was 2 weeks ago. She denies ever being sexually active.
Which of the following vaccines is due at this time?
a. DTP, sixth dose
b. MMR, third dose
c. Diphteria and tetanus booster
d. Varicella, second dose
e. BCG, second dose

90. A 74-year-oldwoman came with left hip pain. She reported that she awoke with pain in
her hip approximately 2 weeks ago, after her fall. It did not radiate, initially sharp and
severe and limiting her activities for first 4 or 5 days. She denied fall, trauma and unusual
or new activities. Before she fall, she often felt dizziness if she stand after sitting. The
pain was occurred when she was getting out of bed. Your diagnosis was left hip fracture
secondary to osteoporosis. You prescribed calcium, vitamin D, bisphosphonate, and
educated her and her husband about fall prevention and caring for elderly.
What intrinsic factors that may cause fall in this patient?
a. Contracture
b. Balance disorder
c. Visual impairment
d. Reduce muscle strength
e. Postural hypotension

91. The Doctor-Patient Communication can be Influence in some factors.This factor is


related to Self Confidence in ability to communicate
a. Doctor-related factors
b. The Interview setting
c. Work Place
d. Medicine and Health Supply
e. Patient-related factors

92. A 61-year-old business executive with a long history of high blood pressure collapsed
while jogging over the lunch hour. His jogging mate quickly contacted a police officer
who helped carry the man to a hospital just down the road. At the hospital, an MRI was
performed that revealed a blockage of a major cerebral artery and ischemic changes to
the portion of the brain supplied by that artery. With quick medical attention, the man
was stabilized, and he slowly improved over the next three weeks. The following signs
and symptoms did persist, however:
- paralysis of the right leg and foot
- loss of sensation on the skin of the right leg and foot
- when blindfolded, inability to identify a tennis ball placed in the left hand, but
ability to name it if placed in the right hand
Tim Akademik ANANTARA 2015
Divisi SL (Soal Latihan)
Fakultas Kedokteran Universitas Padjadjaran

- inability to throw the tennis ball with his left hand, but ability to throw it with
his right hand
There are many categories to placement a patient into home care program. If you plan for
home program for this patient, what is the category for this patient?
a. Unstable chronic illness
b. Acute exacerbations of illness
c. Patient still require medical supervision
d. Patient require rehabilitation
e. Advanced illness

93. A 64-year-old woman, P4A2, Mrs. Yusi visited you with severe pain in hip area. The
pain is constant, boring sometimes sharp with infrequent electric-shock like sensation
radiating to left foot.She was diagnosed as cervical cancer stadium IIIB patient and
receiving chemo radiotherapy. Complaintaccompanied bynausea,vomiting, and dyspnea
forthe last3 days. She has not received any medication for relieving pain.
What type of pain is Mrs. Yusi suffers from?
a. Somatic pain
b. Visceral pain
c. Referred pain
d. Neuropathic pain
e. Combined somatic and neuropathic pain

94. The Doctor-Patient Communication can be Influence in some factors. This factor is
related to Psychological factors related to illness and/or medical care
a. Doctor-related factors
b. The Interview setting
c. Work Place
d. Medicine and Health Supply
e. Patient-related factors

95. A 9-year-old boy diagnosed as short stature. The doctor was planning to gave him
hormonal therapy. Which of the criteria that most likely the boy had?
a. Weight for age below 2 SD at the WHO growth curves
b. Dysmorphic disorder
c. There is a specific syndrome
d. Delayed bone age more than 2 years
e. Height below -3 SD

96. You were doing nutritional counseling for a diabetes mellitus patient. You were
explaining about food choice and daily menu variation.
Which of the following tools that most likely to optimized this explanation?
a. Food recall data
Tim Akademik ANANTARA 2015
Divisi SL (Soal Latihan)
Fakultas Kedokteran Universitas Padjadjaran

b. Food frequency questionnaire data


c. Food list exchange
d. Nutritional requirement data
e. Food weighing data

97. A 55-year-old male, former tennis athlete, with type 2 diabetes mellitus came to his
family doctor. He also diagnosed has moderate cardiovascular disease. He told the
doctor that he still want to do some sport.
Which of the following aspect must be done before start his exercise program?
a. ECG
b. Stress testing
c. CKMB
d. Cardiovascular fitness test
e. Angiography

98. Boy 13-year-old boy brought to your clinic by his parents with complaint of physical
aggression to other children including his younger sister. The school was complained to
the parents because he regularly gets into trouble at school for bullying and truants
from school and also stealing from school's cafetaria. He was a nice boy but his
behaviour start to change and became worse since 2 years ago. Boy comes from a high
socioeconomic family. He lives with his parents, his sister, his grandmother and
grandfather, his aunt and his nephews. His father is a harsh and very punitive. Her
mother gives more attention to his younger sister.. His grandmother wants him to be a
doctor and always encourages him to study hard.
Which of the following situation is the most affecting the occurrence of the disorder?
a. High socioeconomic family
b. Harsh and punitive father
c. Lives with extended family
d. Always encouraged to study hard
e. Mother gives more attention to his sister

99. A 45-years-old man with diagnose leprosy 10 years ago, already recover since 5 years
ago, but until now still isolated by his families and neighbors.
Which of the following sentence is the most correct concept for the case above?
a. Handicap concept
b. Injury concept
c. Pathological concept
d. Social culture concept
e. Psychological concept

100. A 19-year-old boy was brought by the mother due to shorter height than his other
friends. The doctor observes that all the member of the family have normal height. The
doctor was doing some examination to define constitutional delay.
Which of the criterias that most likely for diagnostic ?
Tim Akademik ANANTARA 2015
Divisi SL (Soal Latihan)
Fakultas Kedokteran Universitas Padjadjaran

a. Family history of short stature


b. Normal pubertal development
c. Normal bone age
d. Late pubertal development
e. Osteogenesis imperfecta

101. You was doing counseling to a patient with chronic hypertension. The patient was
asking about food sources that most appropriate to control sodium intake.
Which of the following answer that most correct?
a. Dairy product
b. Fresh food
c. Bread to exchange rice
d. Processed fruit
e. Processed fish

102. Mr. H is an accountant aged 40 years. He comes to the accident and emergency
department following and attack of chest pain. He is seen first by Dr. A.
Dr. A : I see from your notes that you have had some chest pain. Do you still have the
pain?
Mr. H : No, not now.
Dr. A : Was it tight or dull?
Later, Mr. H is seen by Dr. I.
Dr. I : I understand that you have had pain. Would you please tell me more about it?
Mr. H : Well, it was in my chest and it come on when I was sitting at my desk. It was a
funny dull pain that stayed in the middle of my chest. I've had it a few times recently,
always when I'm, at work.
What is the disadvantage of using open style of questioning?
a. The interview may take longer and more difficult to control
b. Patient feel more involved in the interview
c. The information obtained is restricted to the question asked
d. More relevant information can be obtain in a given time
e. The interviewer is controlled by the interviewer, who decides the content of the
questions.

103. Mr. and Mrs. Purcell, Mary, 21, and Bob, 17, came to the family meeting suggested by
Dr. M. After Dr. M. shared her findings, Mary spoke first and expressed concern about
her father's "long-standing" health problems. She blamed her mother for "not taking
better care of him." Bob quickly defended his mother, saying she had been working
very hard and had "a lot on her mind." Bob became upset with Mary for "attacking"
their mother. Mrs. Purcell told Dr. M. that her husband had health complaints for as
long as she had known him. Mr. Purcell then said that his chest pains were worse since
the last doctor's appointment.Family APGAR score=3.
How is "hierarchy" taking place in this family?
Tim Akademik ANANTARA 2015
Divisi SL (Soal Latihan)
Fakultas Kedokteran Universitas Padjadjaran

a. Parents in charge of their children


b. Parents controlled by children
c. Overtly distributed
d. Covertly distributed

104. A 14-year-old boy, brought by his mother with the abrasion on the dorsum right hand.
Which of the following response is most appropiate to use to determie the etiology of
the abrasion on the dorsum of the boy's right hand?
a. “Tell me about the scratches on your hand.”
b. “How did the scratches happen?”
c. “Do you scratches yourself in addition to bitting your nails?”
d. “Sometimes I see the boy who make themselves throw up. Have you ever done
that?”
e. “I see you have scratches on your hand, do you have a cat?”

105. A 61-year-old business executive with a long history of high blood pressure collapsed
while jogging over the lunch hour. His jogging mate quickly contacted a police officer
who helped carry the man to a hospital just down the road. At the hospital, an MRI was
performed that revealed a blockage of a major cerebral artery and ischemic changes to
the portion of the brain supplied by that artery. With quick medical attention, the man
was stabilized, and he slowly improved over the next three weeks. The following signs
and symptoms did persist, however:
- paralysis of the right leg and foot
- loss of sensation on the skin of the right leg and foot
- when blindfolded, inability to identify a tennis ball placed in the left hand, but
ability to name it if placed in the right hand
- inability to throw the tennis ball with his left hand, but ability to throw it with
his right hand
What should physician do for his caregivers in term of "collaborating"?
a. Guiding patient and families through the intricacies of the healthcare delivery
system
b. Advocating the patient and family members in preserving autonomy and choice
regarding medical care
c. Assessing family members in their decision making processes
d. Gathering information, educating family members about the care of their loved one
e. Supporting the patient and family members in situation where care giving is
demanding

106. A 48-year-old man has manifested BP levels of 150/100, 145/95 and 170/105 on
consecutive separate days over a 3-week period. He has a family history of deaths by
stroke and renal failure.
Which of the following is the screening should be performed by his children?
a. Blood pressure every 6 months for people with hypertension
Tim Akademik ANANTARA 2015
Divisi SL (Soal Latihan)
Fakultas Kedokteran Universitas Padjadjaran

b. Blood pressure every 12 months for people with hypertension


c. Blood pressure every 2 years for people with hypertension
d. Blood pressure every 18 months for people with hypertension
e. Blood pressure every 18 months for people without hypertension

107. A young mother brings to you her 18-month-old boy who weights 13.6 kg. This is at the
95th percentile for his height and sex. She is concerned because her family has many
members who are overweight and she worries that the child's future health is at risk.
Your best advice is which of the following?
a. She should commit the child to a weight-reduction program forthwith
b. The child’s weight-related health is not an issue until he reaches adolescence
c. The child should be committed to a weight-loss program but not before he reaches
the age of 2 years
d. A child’s obesity-related health issues involve vascular disease, diabetes and joint
health only, and these problems do not begin until adulthood
e. Obesity in early childhood is not a health issue

108. Prior clinical audit in the management of asthma patient, Dr. Yono develop a standard,
target and criterion.
Which of the following is a criterion?
a. Patients who have been diagnosed as asthma will be recorded in the practice
asthma register
b. The diagnosis of asthma has been confirmed
c. The patient education regarding warning symptoms has been delivered
d. An assessment is made of the level of control of asthma by the presence of
nocturnal and daytime symptoms
e. 95% the diagnosis of asthma has been confirmed by anamnesis, physical
examination and laboratory examination

109. Nutritional management of hypertension is?


a. High fat diet
b. Low salt diet
c. High protein diet
d. High carbohydrat
e. Low Calsium

110. A 61-year-old business executive with a long history of high blood pressure collapsed
while jogging over the lunch hour. His jogging mate quickly contacted a police officer
who helped carry the man to a hospital just down the road. At the hospital, an MRI was
performed that revealed a blockage of a major cerebral artery and ischemic changes to
the portion of the brain supplied by that artery. With quick medical attention, the man
was stabilized, and he slowly improved over the next three weeks. The following signs
and symptoms did persist, however:
- paralysis of the right leg and foot
Tim Akademik ANANTARA 2015
Divisi SL (Soal Latihan)
Fakultas Kedokteran Universitas Padjadjaran

- loss of sensation on the skin of the right leg and foot


- when blindfolded, inability to identify a tennis ball placed in the left hand, but
ability to name it if placed in the right hand
- inability to throw the tennis ball with his left hand, but ability to throw it with
his right hand
What stage of illness trajectory for this patient condition?
a. Onset of illness
b. Diagnosis phase
c. Major therapeutic efforts
d. Recovery
e. Adjustment to the permanency

111. A 55-year-old obese male has been diagnosed as diabetes mellitus type 2. His doctor
told him about his management plan. Beside pharmacology treatment, he also has to
modify his lifestyle especially about routine exercise. The doctor told him that exercise
can help him to decrease body weight and increase cardiovascular fitness.
Which of the following type of exercise that most appropiate for him?
a. Aerobic
b. Anaerobic
c. Weight training
d. Endurance
e. Resistance

112. A 60-year-old postmenopausal woman is seeing you for her annual checkup. She has
been in good general health for the past several years. She experienced menopause in
her early 50s and initiated hormone therapy (HT with estrogen/progestin for her
menopausal symptoms; she has remained on HT primarily for prevention of
osteoporosis. She?is currently not on any chronic medications other than HT.Although
the patient has no personal history of fractures, her older sister experienced a hip
fracture. She is 165 cm and weighs 42 kg. She smokes cigarettes, drinks several cups of
coffee a day, and rarely exercises. Her height has remained stable over the past 4 years.
A physical exam was performed, with no abnormalities noted. A BMD test performed
on this patient revealed a left hip T-score of -1.5. A hip T-score of -1.5. What is your
diagnosis?
a. Osteopenia
b. Osteomalacia
c. Osteoporosis
d. Osteoarthritis

113. A young medical doctor was doing home visit to a patient, a 65-year-old man, post
stroke infarction. He was bedridden and he looked grimace if the right shoulder is
moved. Four weeks ago, he has been diagnosed stroke infarction and been hospitalized
for that. He also had history of hypertension and diabetes mellitus for 15 years ago but
Tim Akademik ANANTARA 2015
Divisi SL (Soal Latihan)
Fakultas Kedokteran Universitas Padjadjaran

didn't control regularly. He got antihypertension drug from the doctor after hospitalized
and he took it only if he felt headache. He always ate "sate kambing" in the weekend.
He was a retired government officer and had health insurance.
Which of the following is the main management of this patient?
a. Rehabilitation program and risk factor modification.
b. Hospitalization and anti-hypertension drugs.
c. Management of nutrition.
d. Physical activity.
e. Analgetic drugs.

114. A 10-year-old boy who has constantly acting out in class. He often lies about things he
has done in class. At home he sprays water on the family cat and sometimes hits the
animal; broken the toy.
The most likely diagnosis is?
a. Autistic Disorder
b. Chidhood Schizophrenia
c. Oppositional Defiant Disorder
d. Conduct Disorder
e. Childhood Schizophrenia

115. A young mother brings to you her 18-month-old boy who weights 13.6 kg. This is at
the 95th percentile for his height and sex. She is concerned because her family has
many members who are overweight and she worries that the child's future health is at
risk.
If the family success in this stage, they will enter the following stage:
a. Pre-school age family
b. Family with toddlers
c. School age family
d. Family with teenagers
e. Family with launching young adults

116. A 35-years-old woman with fever and cough for 7 days, since 5 days ago she treated
herself by herbal from her friend. Her friend got the herbal from herbalist in Garut. She
refused to go to health facilities because of her beliefs her symptom will better soon.
As a doctor, which of the following sentence is the most correct for the woman help
seeking behaviour mentioned above?
a. She can use herbal remedies from any source
b. She can use herbal remedies mixed with modern medicine from the doctor
c. She have to start with low dosages for the herbal
d. She can use the herbal remedies for long treatment
e. She can give the herbal remedies her daughter, 5 years old, because have the same
symptom since 2 days ago
Tim Akademik ANANTARA 2015
Divisi SL (Soal Latihan)
Fakultas Kedokteran Universitas Padjadjaran

117. A 35-year-old women came with progressing nervousness, fatigue, palpitation, and the
recent development of a resting hand tremor.
What is chief complaint for this patient?
a. A 35-year-old women came with nervousness, fatigue, palpitation, and the recent
development of a resting hand tremor.
b. A 35-year-old women came with nervousness and fatigue.
c. Nervousness, fatigue, and palpitation.
d. A 35-year-old women came with came with resting hand tremor.
e. Tremor

118. Mrs. Y (48-years-old) had malignant growth surgically removed from her breast after
several weeks of worrying about the lump. She had felt too frightened to tell her
husband or doctor but eventually went to her GP when she started to lose weight and
had difficulty sleeping. How is the communication techniques that are permitted in
conveying the bad news,even when there is uncertainty of the prognosis for the case of
Mrs.Y:
a. Tell the truth
b. Submission of information through the person closest first
c. Convey of information considered required by the patient or the patient's family
d. Delivering a true medical condition in detail
e. Showing openness about prognosis and treatment of a patient's view oftherapy.

119. Boy 13-year-old boy brought to your clinic by his parents with complaint of physical
aggression to other children including his younger sister. The school was complained to
the parents because he regularly gets into trouble at school for bullying and truants
from school and also stealing from school's cafetaria. He was a nice boy but his
behaviour start to change and became worse since 2 years ago. Boy comes from a high
socioeconomic family. He lives with his parents, his sister, his grandmother and
grandfather, his aunt and his nephews. His father is a harsh and very punitive. Her
mother gives more attention to his younger sister.. His grandmother wants him to be a
doctor and always encourages him to study hard.
Which neurobiological factor below hypothesized to play a role in clinical symptoms in
this case?
a. Low level of plasma dopamine a-hydroxylase x
b. Increased noradrenergic functioning x
c. High 5-HIAA levels in CSF x
d. Low serotonin levels in blood. x
e. 8Lower right frontal EEG activity during rest

120. A 77-year-old had been diagnosed with infarction stroke accompanied with
hypertension and diabetes.
He had been hospitalized three weeks ago. He couldn't communicate well but still
understand what people said. He also felt weakness at his left extremities as a sequele.
He was bedridden. He had been diagnosed with hypertension and diabetes mellitus for
Tim Akademik ANANTARA 2015
Divisi SL (Soal Latihan)
Fakultas Kedokteran Universitas Padjadjaran

15 years ago but he didn't control regularly. He got anti-hypertensive drug from the
doctor and only took it if he felt unwell. He always ate "tongseng kambing" in the
weekend.
Which of the following management that most likely for this patient?
a. Bed positioning education
b. Training communication
c. Training gait
d. Educate to bed positioning, training communication and gait
e. Training communication and gait

121. Nutritional management with time schedule arrangement of main course and snack
between meals is important to control blood glucose. The snack should be low calorie
and contains high fiber food.
Which of the following aspects that can be controlled by this method?
a. Hunger satiety
b. Protein absorption
c. Vitamin absorption
d. Increase fat utility
e. Increase mineral absorption

122. A man, aged 25 years comes to you in your clinic with chief complaints purulent
urination. From the medical records of patients stated marital status is unmarried.
What should you ask as a doctor to look for the sexual history?
a. "Are you married?"
b. "Sorry ...are you gay?"
c. "When did you last have a sexual contact?"
d. "Your previous sexual intercourse with a male or female?"
e. "Do you use protection during sexual intercourse?"

123. A 30-years-old man with sleeping disorders used herbal medicine since two months
ago.
Which of the following sentence is the most correct evaluation of effectiveness therapy
for the case above?
a. Based on clinical adjustment by the doctor
b. Based on physical examination by the doctor
c. Based on laboratory examination
d. Based on mental health examination
e. Based on self-control from the patient

124. A 61-year-old business executive with a long history of high blood pressure collapsed
while jogging over the lunch hour. His jogging mate quickly contacted a police officer
who helped carry the man to a hospital just down the road. At the hospital, an MRI was
performed that revealed a blockage of a major cerebral artery and ischemic changes to
the portion of the brain supplied by that artery. With quick medical attention, the man
Tim Akademik ANANTARA 2015
Divisi SL (Soal Latihan)
Fakultas Kedokteran Universitas Padjadjaran

was stabilized, and he slowly improved over the next three weeks. The following signs
and symptoms did persist, however:
- paralysis of the right leg and foot
- loss of sensation on the skin of the right leg and foot
- when blindfolded, inability to identify a tennis ball placed in the left hand, but
ability to name it if placed in the right hand
- inability to throw the tennis ball with his left hand, but ability to throw it with
his right hand
What is a physician role in this stage?
a. Ease the patient and family anxieties and concerns
b. Responsible for making a clinical judgment about the amount of information
c. Alert about potential problem may be occurred
d. Define psychological state of patient and family about therapeutic plan
e. Anticipate the second crisis of adaptation to the outcome

125. A 5-year old girl was brought to the office because she is shorter than other girls at her
age. On examination you found, she was less than fifth percentile on growth curves,
even when you correct for mid parental height. There are multiple potential causes for
short stature. She reveals that she speaks only six-word sentences but they are
intelligible.
The parents are concerned about her development as well. Which of the following
responses is most appropriate?
a. Refer the child for specialized evaluation of development
b. Arrange a conference with the child’s teacher
c. Order a magnetic resonance image of the child’s brain
d. Reassure the parents that the child is developing within normal limits
e. Review the milestones of growth and development in this child

126. The doctor was doing nutritional assessment from cumulative effect borderline food
intake during catch up growth to a 9-year-old girl?
Which of the following condition that most likely found in this patient?
a. Development parameter
b. Very low immunity status
c. Short stature
d. Extreme underweight
e. No abnormality

127. A 58-year-old woman had been diagnosed with cervical cancer and complained back
pain since 4 weeks ago. She also had radiating pain and tingling in her left leg.
Neurological examination revealed : there are Laseque and Kernig limitation of left leg
due to pain, normal physiological reflex, and pathological reflex (-).
What of the following analgetic that most appropriate for this patient?
a. Asetaminofen.
Tim Akademik ANANTARA 2015
Divisi SL (Soal Latihan)
Fakultas Kedokteran Universitas Padjadjaran

b. Meloxicam.
c. Opioid.
d. Antidepressantrisiclic.
e. Diclofenac sodium.

128. The Doctor-Patient Communication can be Influence in some factors. This factor is
related to comfortable surroundings
a. Doctor-related factors
b. The Interview setting
c. Work Place
d. Medicine and Health Supply
e. Patient-related factors

129. A 48-year-old man has manifested BP levels of 150/100, 145/95 and 170/105 on
consecutive separate days over a 3-week period. He has a family history of deaths by
stroke and renal failure.
Which of the following may be the single most appropriate drug to be prescribed?
a. Hydrochlorothiazide
b. ACE Inhibitor
c. Beta-adrenergic blocker
d. ACE receptor blocking agent
e. Vasodilator

130. A 58-year-old woman had been diagnosed with cervical cancer and complained back
pain since 4 weeks ago. She also had radiating pain and tingling in her left leg.
Neurological examination revealed : there are Laseque and Kernig limitation of left leg
due to pain, normal physiological reflex, and pathological reflex (-).
What is the most likely problem of this patient?
a. Cervical cancer.
b. Hernia Nucleous Pulposus.
c. Osteoarthritis.
d. Paralysis.
e. Vertebral metastasis.

131. What is the correct role of family in weight management in adolescent?


a. Follow up daily activity living
b. Skip many high calorie food
c. Increase physical activity
d. Motivation behavioral changes
e. Decrease sedentary life style

132. When should preconception counseling be performed:


a. 6 months before couple considering pregnancy
b. By the time of the first prenatal visit
Tim Akademik ANANTARA 2015
Divisi SL (Soal Latihan)
Fakultas Kedokteran Universitas Padjadjaran

c. After married
d. 1-2 years before couple considering pregnancy

133. Boy 13-year-old boy brought to your clinic by his parents with complaint of physical
aggression to other children including his younger sister. The school was complained to
the parents because he regularly gets into trouble at school for bullying and truants
from school and also stealing from school's cafetaria. He was a nice boy but his
behaviour start to change and became worse since 2 years ago. Boy comes from a high
socioeconomic family. He lives with his parents, his sister, his grandmother and
grandfather, his aunt and his nephews. His father is a harsh and very punitive. Her
mother gives more attention to his younger sister.. His grandmother wants him to be a
doctor and always encourages him to study hard.
Which of the following statements is appropriate in family psycho education for this
case?
a. Accept and care for the patient
b. Avoid too much praising or rewarding the patient
c. Confrontates the patient every time he shows aggressive behaviors
d. Give a hard punishment to the patient every time he does not behave properly
e. Have a long discussion with the child whenever he does not behave properly, as
soon as possible

134. A young medical doctor was doing home visit to a patient, a 65-year-old man, post
stroke infarction. He was bedridden and he looked grimace if the right shoulder is
moved. Four weeks ago, he has been diagnosed stroke infarction and been hospitalized
for that. He also had history of hypertension and diabetes mellitus for 15 years ago but
didn't control regularly. He got antihypertension drug from the doctor after hospitalized
and he took it only if he felt headache. He always ate "sate kambing" in the weekend.
He was a retired government officer and had health insurance.
Which of the following linguistic and communication disorder is most likely occurred
to this patient?
a. Sensoric aphasia.
b. Transcortical motoric aphasia.
c. Global aphasia.
d. Motoric aphasia.
e. Transcorticalsensoric aphasia.

135. An 8-year-old male is frequently rude, hostile and always argumentative towards his
teachers. He becomes easily annoyed by others and disobeys classroom rules. He also
only has a few friends. However, he has not been doing something that violates the
rights of others.
Which of the following symptoms is mostly directing the diagnosis towards
oppositional defiant disorder rather than conduct disorder?
a. Disobey classroom rules
b. Easily annoyed by others
Tim Akademik ANANTARA 2015
Divisi SL (Soal Latihan)
Fakultas Kedokteran Universitas Padjadjaran

c. Always argumentative towards his teacher


d. Has not been violating the rights of others
e. Frequently rude and hostile towards his teachers

136. The parent-child positive relationship also becomes important when the child is in
school, when positive relations form at home children will have an easier time forming
positive relations in school. Children with good communication and bonding in the
home bring positive behaviours to school which allows for good communication with
peers, teachers and authorities.
Which one the structure of family that have better influences?
a. Single parent families (mother
b. Single parent families (father
c. Blended families
d. Intact families
e. The other

137. Refer to the genogram below:

Interpret a genogram above from the life cycle perspective!


a. It is a nuclear family, Family with teenagers
b. It is extended family, Family with teenagers
c. It is a nuclear family, Family with launching young adults
d. It is extended family, Family with launching young adults
e. It is a nuclear family, Middle aged parents

138. A 75-year-old man is in good health and mentally competent to care for himself. He
underwent a complete examination 5 years ago that included a normal screening
colonoscopy, and he has submitted fecal occult blood test in the intervening years. He
lives alone and wishes to undergo a routine health maintenance examination.
Which of the following would be least fruitful as a screening test?
a. Flexible sigmoidoscopy
Tim Akademik ANANTARA 2015
Divisi SL (Soal Latihan)
Fakultas Kedokteran Universitas Padjadjaran

b. Thyroid – stimulating hormone test


c. Urinalysis
d. Blood pressure reading
e. “Up and go test”

139. A 60-year-old female with diabetes mellitus type 2 want to ask exercise prescription
advice from her family doctor. She realizes that exercise helps her to maintain her
health condition, so she always takes jogging for minimum 30 minutes, 3 times a week
at nearby park. But recently she just diagnosed having peripheral neuropathy at her
lower limb.
Which of the following type of exercise that most appropriate for her recent condition?
a. Jogging, but only for 20 minutes
b. Jogging on the treadmill
c. Tennis
d. Golf
e. Swimming

140. A 60-years-old woman comes to family physician with loss of body weight with mild
fever and malaise since 5 months ago. Based on the result from anamnesis, physical
examination and laboratory examination found that some value in abnormal limit.
Which of the following sentence is the most correct concept for the case above?
a. Handicap concept
b. Injury concept
c. Pathological concept
d. Social culture concept
e. Psychological concept

141. You were doing nutrition counseling to a parent with 18-month-old baby girl. Their
daughter was picky eaters and have mild anemia due to food intake difficulty. They
have opinion that milk can fulfill the nutritional requirement.
What is the best explanation to your patient about their opinion?
a. Its the correct action because milk is the ideal nutritional composition
b. Give more milk can complete with vegetable
c. Milk with multivitamin can solve the problem
d. That will be incorrect choice because of low intake non-protein calorie
e. You can add mole sugar to increase calorie from milk

142. Growth curve analysis is the most important step for the evaluation of children with
growth disorders. There are several aspects that must be evaluated carefully.
Which of the following steps that found in the growth curve analysis?
a. Anamnesis
b. Physical examination
c. Growth of velocity
d. Determination of upper and lower segments
Tim Akademik ANANTARA 2015
Divisi SL (Soal Latihan)
Fakultas Kedokteran Universitas Padjadjaran

e. Determination of target height

143. Increase food intake and improvement of nutrient quality content is important to
support failure to thrive management.
Which of the following effort that most likely to increase food intake?
a. Give the full calorie need
b. Increasing step by step
c. Give more animal protein sources
d. Give more milk
e. Limitation the physical activity

144. A 60-year-old postmenopausal woman is seeing you for her annual checkup. She has
been in good general health for the past several years. She experienced menopause in
her early 50s and initiated hormone therapy (HT with estrogen/progestin for her
menopausal symptoms; she has remained on HT primarily for prevention of
osteoporosis. She?is currently not on any chronic medications other than HT.Although
the patient has no personal history of fractures, her older sister experienced a hip
fracture. She is 165 cm and weighs 42 kg. She smokes cigarettes, drinks several cups of
coffee a day, and rarely exercises. Her height has remained stable over the past 4 years.
What is this patient's major risk of a future fracture?
a. Estrogen deficiency at an early age (<45 years
b. Low physical activity
c. Alcohol in amounts >2 drinks/day
d. High BMI
e. History of fragility fracture in a first-degree relative

145. A 45-year-old female with obese and hypertension stage 1 want to ask some advice
about suitable exercise program for her beside her diet program. She told the doctor that
she enjoy swimming and bowling. But she has difficulty to spare enough time to do that
activity because her activity as busy business woman.
Which of the following exercise program that most appropriate for her?
a. 60 minutes, jogging or treadmill, 3 times a week
b. 30 minutes jogging in the morning, 30 minutes swimming at the evening
c. 60 minutes bowling (once a week, 60 minutes swimming (twice a week
d. 60 minutes of any kind moderate intensity (free to choose by herself, 3 times a
week
e. 60 minutes of any kind high intensity exercise (free to choose by herself, once a
week

146. A 48-year-old man has manifested BP levels of 150/100, 145/95 and 170/105 on
consecutive separate days over a 3-week period. He has a family history of deaths by
stroke and renal failure.
If the patient was suggested to do physical exercise, which of the following is a valid
reason to terminate an exercise session?
Tim Akademik ANANTARA 2015
Divisi SL (Soal Latihan)
Fakultas Kedokteran Universitas Padjadjaran

a. The systolic BP rises and the diastolic falls during strenuous exercise
b. The pulse rate doubles during strenuous exercise
c. The patient experiences persistent headache during exercise
d. The patient is fatigued during strenuous exercise
e. Patient develops tachypnea

147. A 55-year-old male with type 2 diabetes mellitus came to his family doctor. He likes
play tennis for exercise. He plays tennis almost every day at least 2 hours a day. After
some history taking, physical examination and random blood sugar examination, the
doctor told him that he should stop his exercise activity for a while.
Which of the following condition that most appropriate for the doctor suggestion?
a. Systolic blood pressure more than 140 mmHg
b. Blister at his feet
c. Decrease of visual acuity
d. Blood sugar level >250 mg/dl
e. Blood sugar level <200 mg/dl

148. Nutritional intake can support the optimal growth and development in every step of
catch up growth in children. The doctor was trying to calculate a patient's nutritional
requirement.
Which of the following factors that can influence nutritional requirement?
a. Gender
b. Alergic status
c. Age
d. Body surface area
e. Liver function

149. A 60-year-old postmenopausal woman is seeing you for her annual checkup. She has
been in good general health for the past several years. She experienced menopause in
her early 50s and initiated hormone therapy (HT with estrogen/progestin for her
menopausal symptoms; she has remained on HT primarily for prevention of
osteoporosis. She?is currently not on any chronic medications other than HT.Although
the patient has no personal history of fractures, her older sister experienced a hip
fracture. She is 165 cm and weighs 42 kg. She smokes cigarettes, drinks several cups of
coffee a day, and rarely exercises. Her height has remained stable over the past 4 years.
What is the treatment option for her condition?
a. Discontinuation of hormone replacement therapy
b. Continued hormone replacement therapy
c. Continued hormone replacement therapy and initiation of bisphosphonate
d. No need treatment for current condition

150. Lisa is a 48-year-old woman who works in an animal laboratory. At the first visit, you
note that her blood pressure has been elevated 160/95 mmHg and 155/100 respectively.
She has no history of hypertension and is not on any medications.
Tim Akademik ANANTARA 2015
Divisi SL (Soal Latihan)
Fakultas Kedokteran Universitas Padjadjaran

Assuming that the initial evaluation does not turn up surprises, what should be your
initial management?
a. Looking for symptoms of end-organ disease
b. Exploring cardiac risk factors
c. Starting with single drug antihypertension
d. Starting with combination drug antihypertension
e. Starting with lifestyle modification

151. The causes of hypertension are multi-factorial, because blood pressure is the outcome
from many system organ and cellular regulation in the body.
Which of the following micronutrient that most affect the blood pressure regulation?
a. Ferrum and biotin
b. Vitamin A and C
c. Phosphor and riboflavin
d. Calcium and vitamin D
e. Zink and vitamin E

152. Bill is a 72-year-old retired driver who was admitted to your practice because of back
pain. He has been a smoker for 30 years and smokes 15 cigarettes every day. Bill lives
alone in a small house and has lost about 5 kg of weight in the past few months. His
diet has deteriorated since his wife passed away two years ago. Bill's daughter
accompanies him to visit his GP, as she is concerned about his recent weight loss.His
current medicine history includes ramipril 5 mg once daily in the morning for well-
controlled hypertension. He has no significant family medical history. On examination
Bill's blood pressure is 115/78 mmHg. A bone mineral density test T-score in the hip is
-1.8 and in the spine is -2.7.
What information would you provide to patient regarding "calcium intake"?
a. Calcium preparations should be taken in divided doses
b. Calcium preparations should be taken in single dose
c. Administered during waking hours
d. Should be taken before meal

153. Which the following tuberculosis drug is an inducer metabolism?


a. Streptomycin
b. Isoniazid
c. Rimpamycin
d. Pirazynamid
e. Ethambutol

154. A 2-year-old boy, Body Weight 6.5 kg, Body Length 80 cm, complains of weight loss
for the last 6 months, along with loss of appetite. He has the following anthropometric
measures: weight-for-age z-score <-3 SD, weight for height = 59%, height for age =
93%.
The diagnosis of this patient is
Tim Akademik ANANTARA 2015
Divisi SL (Soal Latihan)
Fakultas Kedokteran Universitas Padjadjaran

a. Severe malnutrition marasmic type


b. Severe malnutrition kwarshiorcor type
c. Severe malnutrition marasmic- kwarshiorcor type
d. Failure to thrive
e. Short stature

155. A 40-year-old-man came with occasional episodes of headache. And he also


complained blurred vision since two days ago. His father died due to hypertension and
stroke. He enjoyed high cholesterol food and very rare to go for exercise. He is found to
have a blood pressure of 170/90.
Which can we do to increase adherence in consuming medical treatment?
a. Give expensive medicine
b. Education to the patient
c. Prescribed a lot of medicine
d. Poor communication
e. Watch TV a lot

156. What are the risks associated with advanced maternal age(>35 years)?
a. Pre-eclampsia, gestational diabetes, prolonged labour
b. Chromosomal anomalies, spontaneous abortion, pre-eclampsia
c. Fertility increases, post partum haemorhage, gestational diabetes
d. Chromosomal anomalies, fertility increases, gestational diabetes

157. What are the most effective and efficient prevention effort for cervical cancer below?
a. Early detection by pap smear and VIA
b. Vaccination
c. Tumor marker
d. Ultrasound

158. An 18-month-old boy with 13.6 kg body weight. This is at the 95th percentile for his
height and sex. His mother was concerned because his family has many members who
are overweight and she worries that the child's future health is at risk.
Which of the following advice that most likely for the patient?
a. She should commit the child to a weight-reduction program forthwith
b. The child’s weight-related health is not an issue until he reaches adolescence
c. The child should be committed to a weight-loss program but not before he reaches
the age of 2 years
d. A child’s obesity-related health issues involve vascular disease, diabetes and joint
health only, and these problems do not begin until adulthood
e. Obesity in early childhood is not a health issue

159. Rani is 26-year-old woman, G2P1A0, present in the initial visit with a 4-week history
of fatigue and nausea. She had a positive pregnancy test 2 weeks ago.Rani has been on
a multivitamin preparation with folic acid, she has been immunized against Hepatitis B
Tim Akademik ANANTARA 2015
Divisi SL (Soal Latihan)
Fakultas Kedokteran Universitas Padjadjaran

within the past 3 years. From physical examination, she is a well appearing, mildly
overweight. Vital signs and other examination are within normal limit. Your assessment
is intrauterine pregnancy.
Considering her weight, what is recommended weight gain during her pregnancy?
a. 12,5 – 18 kg
b. 11,5 – 16 kg
c. 7 – 11,5 kg
d. 5 – 6,8 kg
e. Maximum 5,8 kg

160. If the body mass index measurement of the child above 5-year-old is between 2 SD and
3 SD percentiles of WHO 2010 growth chart , what would you inform the parents
regarding their child nutritional status?
a. Well-nourished
b. Risk of overweight
c. Overweight
d. Risk of obesity
e. Obesity

161. Control of food intake influenced by many factors such as individual development and
emotional condition. This phenomenon especially occurred among teenager.
Which of the following roles that can be influenced by the family?
a. Decrease amount of main course menu
b. Disappear between meal snack
c. Behavioral modification motivation
d. Increase physical activity
e. Pharmacological treatment

162. A 40-year-old-man came with occasional episodes of headache. And he also


complained blurred vision since two days ago. His father died due to hypertension and
stroke. He enjoyed high cholesterol food and very rare to go for exercise. He is found to
have a blood pressure of 170/90.
Which of the following JNC VIII classification is the most likely for this patient?
a. Normal
b. Pre-hypertension
c. Stage 1
d. Stage 2
e. Stage 3

163. You was doing education to a group of Posyandu cadre. You was explaining about food
source and food exchange.
What is the best supportive tool to make the same perception this?
a. Video presentation
b. Calculator
Tim Akademik ANANTARA 2015
Divisi SL (Soal Latihan)
Fakultas Kedokteran Universitas Padjadjaran

c. Food model
d. Poster
e. Real food

164. Meal planning is about fulfilling adequate nutritional intake. It must be arranged to
support growth, development, immunity and body function maintenance.
After we determine the quantity of total calorie intake, which of the following steps
need to be taken?
a. Amount of carbohydrate intake
b. Amount of protein calorie
c. Fiber sources
d. Carbohydrate exchange
e. Animal protein food sources

165. A27-year-old woman using oral contraceptive, with tuberculosis infection. Which of
the following is right?
a. TB drugs are given but the contraceptive is changed into Intrauterine Device (IUD
b. Oral contraceptive are given but the TB drug only taken for 2 months
c. Oral contraceptive is stopped but TB drug is given all the time
d. Oral contraceptive and TB drug is taken both
e. Oral contraceptive is changed into another drug that have interaction with TB
drugs

166. A 19-year-old teenage boy was brought due to obesity. The parents were worried
because his body weight was increasing significantly and they concerned about his
psychological problem also. He was refuse go out and socialized and prefer stay at
home.
What is the main focus of his treatment?
a. Multidiscipline approach
b. Increase daily activity
c. Low calorie intake
d. Behavioral counseling
e. Maintenance health status

167. A 64-year-old woman, P4A2, Mrs. Yusi visited you with severe pain in hip area. The
pain is constant, boring sometimes sharp with infrequent electric-shock like sensation
radiating to left foot.She was diagnosed as cervical cancer stadium IIIB patient and
receiving chemo radiotherapy. Complaintaccompanied bynausea,vomiting, and dyspnea
forthe last3 days. She has not received any medication for relieving pain.
Screening is one strategy to prevent cervical cancer. If screening test were negative,
what would be your recommendation?
a. It is important to be screened annually
b. It is important to be screened every 2 years
c. It is important to be screened every 3–5 years
Tim Akademik ANANTARA 2015
Divisi SL (Soal Latihan)
Fakultas Kedokteran Universitas Padjadjaran

d. Not necessarily to be screened in the future

168. A 64-year-old woman, P4A2, Mrs. Yusi visited you with severe pain in hip area. The
pain is constant, boring sometimes sharp with infrequent electric-shock like sensation
radiating to left foot.She was diagnosed as cervical cancer stadium IIIB patient and
receiving chemo radiotherapy. Complaintaccompanied bynausea,vomiting, and dyspnea
forthe last3 days. She has not received any medication for relieving pain.
After several weeks, Mrs Yusi looked severely ill and she was asking you to assure her
family members that her condition is better and no worries any more.
According to "five stages of dying", the stage of Mrs. Yusi is the following:
a. Denial
b. Anger
c. Acceptance
d. Bargaining
e. Depression

169. A young medical doctor was doing home visit to a patient, a 65-year-old man, post
stroke infarction. He was bedridden and he looked grimace if the right shoulder is
moved. Four weeks ago, he has been diagnosed stroke infarction and been hospitalized
for that. He also had history of hypertension and diabetes mellitus for 15 years ago but
didn't control regularly. He got antihypertension drug from the doctor after hospitalized
and he took it only if he felt headache. He always ate "sate kambing" in the weekend.
He was a retired government officer and had health insurance.
Which of the following is the main problem of this patient?
a. Post stroke infarction.
b. Disability and bad adherence.
c. Hypertension and diabetes mellitus.
d. Diabetes mellitus.
e. Bedridden and dislipidemia.

170. Various tests to determine the cause of short statured one. Complete blood count and
examination of hormonal.
Which of the following hormonal test that most appropriate?
a. Craetinin and ureum
b. Karyotype test
c. T4 and TSH
d. Bone age
e. Brain MRI

171. Bill is a 72-year-old retired driver who was admitted to your practice because of back
pain. He has been a smoker for 30 years and smokes 15 cigarettes every day. Bill lives
alone in a small house and has lost about 5 kg of weight in the past few months. His
diet has deteriorated since his wife passed away two years ago. Bill's daughter
accompanies him to visit his GP, as she is concerned about his recent weight loss.His
Tim Akademik ANANTARA 2015
Divisi SL (Soal Latihan)
Fakultas Kedokteran Universitas Padjadjaran

current medicine history includes ramipril 5 mg once daily in the morning for well-
controlled hypertension. He has no significant family medical history. On examination
Bill's blood pressure is 115/78 mmHg. A bone mineral density test T-score in the hip is
-1.8 and in the spine is -2.7.
Considering his condition, he has risk for fall. His risk of fall is categorized as follow:
a. Acute and chronic medical conditions
b. Social/ behavioral factors
c. Medication factors
d. Extrinsic factors
e. Activity related causes

172. Here are the stages of emotion reactions of terminal illness patient?
a. Anger, depression, bargaining, denial and acceptance
b. Denial, anger, bargaining, depression and acceptance
c. Anger, denial, bargaining, acceptance and depression
d. Anger, denial, guilt, depression and acceptance

173. A 55-year-old male with hypertension stage 2 with suspected cardiovascular disease
need exercise prescription for his family doctor
Which of the following that most appropriate exercise prescription for him?
a. 20-30 minutes/day continuous aerobic activity at maximum heart rate 50-65%
b. Light-moderate intensity until evaluated and conditioned with periodic monitoring
c. Light-moderate start with 20-30 minutes lower intensity/day with periodic
monitoring
d. 60 minutes/day of aerobic exercise without periodic monitoring
e. Start with 60 minutes/day of resistance training

174. A young medical doctor was doing home visit to a patient, a 65-year-old man, post
stroke infarction. He was bedridden and he looked grimace if the right shoulder is
moved. Four weeks ago, he has been diagnosed stroke infarction and been hospitalized
for that. He also had history of hypertension and diabetes mellitus for 15 years ago but
didn't control regularly. He got antihypertension drug from the doctor after hospitalized
and he took it only if he felt headache. He always ate "sate kambing" in the weekend.
He was a retired government officer and had health insurance.
Which of the following complication is most likely for this patient?
a. Ulkus decubitus.
b. Hypertension.
c. Contracture.
d. Paralysis.
e. Hyperglicemia.

175. Prior clinical audit in the management of asthma patient, Dr. Yono develop a standard,
target and criterion.
After developing the standard, what should Dr. Yono do?
Tim Akademik ANANTARA 2015
Divisi SL (Soal Latihan)
Fakultas Kedokteran Universitas Padjadjaran

a. Reviewing literature
b. Compare practice with standard
c. Data collection
d. Implement change
e. Re-audit

176. You was planning the menu for a-5-year-old patient with failure to thrive. Your goal
was to provide adequate protein and energy needed to maintain the chief function for
every nutrient.
Which of the following nutrition function that most important for this condition?
a. Protein sparring effect.
b. Main function of fat as amino acid precursor
c. Determine the supplementation need.
d. Determine the food processing
e. To prevent increase energy synthesis

177. A 45-year-old female with prehypertensive and no cardiovascular disease need exercise
prescription for her family doctor.
Which of the following that most appropriate exercise prescription for her?
a. 20-30 minutes/day continuous aerobic activity at maximum heart rate 50-65%
b. Light-moderate intensity until evaluated and conditioned with periodic monitoring
c. Light-moderate start with 20-30 minutes lower intensity/day with periodic
monitoring
d. 60 minutes/day of aerobic exercise without periodic monitoring
e. Start with 60 minutes/day of resistance training

178. A young medical doctor was doing home visit to a patient, a 65-year-old man, post
stroke infarction. He was bedridden and he looked grimace if the right shoulder is
moved. Four weeks ago, he has been diagnosed stroke infarction and been hospitalized
for that. He also had history of hypertension and diabetes mellitus for 15 years ago but
didn't control regularly. He got antihypertension drug from the doctor after hospitalized
and he took it only if he felt headache. He always ate "sate kambing" in the weekend.
He was a retired government officer and had health insurance.
Where is the location of stroke infarction in this patient?
a. Cerebellum.
b. Medulla oblongata.
c. Pons.
d. Right hemisphere.
e. Left hemisphere.

179. A 50-year-old man was diagnosed having stage 1 hypertension. He was doing exercise
and diet to control blood pressure.
Which of the following fat that most appropriate for him?
a. Low fat intake
Tim Akademik ANANTARA 2015
Divisi SL (Soal Latihan)
Fakultas Kedokteran Universitas Padjadjaran

b. Low cholesterol intake


c. The kind of fat is saturated fatty acid
d. Omega 3 fatty fatty acid consist in fat food sources
e. They need only monounsaturated fatty acid

180. A 40-year-old male came to his family doctor with headache. His blood pressure was
150/90 mmHg. The family doctorthen does head to toe physical examination,
laboratory examination and other supportive examination. The doctor then decides to
treat him with supportive therapy for headache and lifestyle modification including
exercise.
Which of the following that most appropriate for the target heart rate?
a. 220 x/minute
b. 200 x/minute
c. 153 x/minute
d. 135 x/minute
e. 90 x/minute

181. People are motivated to do the exercise programs because of these reason, except
a. Weight control.
b. Increase the blood pressure.
c. Reduce risk of hypotension.
d. Reduction in stress and depresion.
e. Enjoyment.

182. A 58-year-old woman had been diagnosed with cervical cancer and complained back
pain since 4 weeks ago. She also had radiating pain and tingling in her left leg.
Neurological examination revealed : there are Laseque and Kernig limitation of left leg
due to pain, normal physiological reflex, and pathological reflex (-).
Which of the following management for her back pain?
a. Pain management.
b. Medical rehabilitation.
c. Physioterapy.
d. Surgery procedure.
e. Chemoteraphy.

183. Good communication skill doctor may help terminal stage patient feel comfortable
talking about their illness. Here is following suggestions communication technique?
a. Ask closed-ended questions (‘yes’ or ‘no’ answer)
b. Begin by talking about personal topics then moving to general ones
c. Ask open-ended question
d. Always use medical terms to give explanation
Tim Akademik ANANTARA 2015
Divisi SL (Soal Latihan)
Fakultas Kedokteran Universitas Padjadjaran

184. A 48-year-old man has manifested BP levels of 150/100, 145/95 and 170/105 on
consecutive separate days over a 3-week period. He has a family history of deaths by
stroke and renal failure.
Which of the following is the screening should be performed by his children?
a. Blood pressure every 6 months for people with hypertension
b. Blood pressure every 12 months for people with hypertension
c. Blood pressure every 2 years for people with hypertension
d. Blood pressure every 18 months for people with hypertension
e. Blood pressure every 18 months for people without hypertension

185. A 75-year-old had been diagnosed with stroke infarction. He still has communication
disorder and weakness of right extremities as a sequele. He lived with his wife and son
that has been married and has a job with 3 years old boy. He bed ridden only. He was a
retired government officer. He cannot work. He was often looked angry because of
communication difficulty.
What rehabilitation problems does patient have?
a. Mobilization , vocational, Impairment, occupational and disability
b. Mobilization, communication, vocational, occupational, disability
c. Mobilization, communication, Impairment, occupational and disability
d. Mobilization, communication, vocational, occupational
e. Mobilization, communication, occupational, social, vocational

186. A 5-year-old girl was brought to the office because she is shorter than other girls at her
age. On examination you found, she was less than fifth percentile on growth curves,
even when you correct for midparental height. There are multiple potential causes for
short stature.
She reveals that she speaks only six-word sentences but they are intelligible.
What the family influence could be a potential cause for her condition?
a. Psychological and social characteristics of parents
b. Spreading of infectious diseases among family members
c. Parental deprivation
d. Klinefelter syndrome
e. Environmental condition of the family

187. A young mother brings to you her 18-month-old boy who weights 13.6 kg. This is at
the 95th percentile for his height and sex. She is concerned because her family has
many members who are overweight and she worries that the child's future health is at
risk.
His family is in stage "Childbearing families". What developmental task will be raised
in this stage?
a. Adapting to the critical needs and interests of preschool children in stimulating,
growth promoting ways
b. Coping with energy depletion and lack of privacy as parents
c. Encouraging children’s educational achievement
Tim Akademik ANANTARA 2015
Divisi SL (Soal Latihan)
Fakultas Kedokteran Universitas Padjadjaran

d. Establishing a mutually satisfying marriage


e. Having, adjusting to and encouraging the development of children

188. A 35-year-old male had hypertension. But after do adequate exercise and diet program
for 6 month, now his condition was improving. The blood pressure is normal, and there
is no abnormality at his cardiovascular condition.
Which of the following that most appropriate reason for this improvement?
a. Increasing the symphatetic tone
b. Increasing the blood cathecolamine
c. Lower cardiac output and peripheral vascular resistance
d. Altered renal function lead to decrease elimination of sodium
e. Increasing submaximal exercise peripheral vascular resistance

189. A 74-year-oldwoman came with left hip pain. She reported that she awoke with pain in
her hip approximately 2 weeks ago, after her fall. It did not radiate, initially sharp and
severe and limiting her activities for first 4 or 5 days. She denied fall, trauma and
unusual or new activities. Before she fall, she often felt dizziness if she stand after
sitting. The pain was occurred when she was getting out of bed. Your diagnosis was left
hip fracture secondary to osteoporosis. You prescribed calcium, vitamin D,
bisphosphonate, and educated her and her husband about fall prevention and caring for
elderly.
When will you refer to PM& R specialist?
a. After operating
b. One week before operating
c. Immediately
d. After she can getting out of bed
e. After she can gait

190. A 5-year-old girl was brought to the office because she is shorter than other girls at her
age. On examination you found, she was less than fifth percentile on growth curves,
even when you correct for midparental height. There are multiple potential causes for
short stature.
She reveals that she speaks only six-word sentences but they are intelligible.
What screening would you recommend for a girl?
a. Blood screening (hemoglobinopathy, phenylketonuria, congenital hypothyroidism,
HIV
b. Hearing
c. Fluoride to prevent dental cavitation
d. Injury prevention
e. Breast feeding

191. In meal planning, formulating some variations of menu is needed to maintain balanced
diet and adequate appetite.
What tool is useful to formulate some menu variations?
Tim Akademik ANANTARA 2015
Divisi SL (Soal Latihan)
Fakultas Kedokteran Universitas Padjadjaran

a. Nutritional requirement data.


b. Food composition list.
c. Food weighing data.
d. Food frequency questionnaire data.
e. Food recall data.

192. Who should the pre-conceptional care should be performed?


a. Women with a hystory of a congenital anomaly child
b. Women with underlying medical problems
c. All women of childbearing age
d. Only a and b are true

193. Boy 13-year-old boy brought to your clinic by his parents with complaint of physical
aggression to other children including his younger sister. The school was complained to
the parents because he regularly gets into trouble at school for bullying and truants
from school and also stealing from school's cafetaria. He was a nice boy but his
behaviour start to change and became worse since 2 years ago. Boy comes from a high
socioeconomic family. He lives with his parents, his sister, his grandmother and
grandfather, his aunt and his nephews. His father is a harsh and very punitive. Her
mother gives more attention to his younger sister.. His grandmother wants him to be a
doctor and always encourages him to study hard.
Which of the following is the correct management for this patient in primary health
care?
a. Parent skills training is a core management
b. Use medications for severe disruptive behavior
c. Consider methylphenidate for aggressive behavior
d. Anticipate major life changes, arrange personal and social support.
e. For confidentiality, avoid involving school’s teachers in management plans

194. A 64-year-old woman, P4A2, Mrs. Yusi visited you with severe pain in hip area. The
pain is constant, boring sometimes sharp with infrequent electric-shock like sensation
radiating to left foot.She was diagnosed as cervical cancer stadium IIIB patient and
receiving chemo radiotherapy. Complaintaccompanied bynausea,vomiting, and dyspnea
forthe last3 days. She has not received any medication for relieving pain.
What medicine should be given to relieve pain?
a. Codeine
b. Ibuprofen
c. Diazepam
d. Morphine
e. Haloperidol

195. Risk factor of obesity that can't modified :


a. Sedentary life
b. Genetic
Tim Akademik ANANTARA 2015
Divisi SL (Soal Latihan)
Fakultas Kedokteran Universitas Padjadjaran

c. Low physical activity


d. Unhealthty diet
e. Lack of sleep

Number 196-200 refer to the diagram below!

196. A 30-years-old man with diagnose mental illness but he feel healthy, isolated by their
neighbors. Which of the following sentence is the most correct area for the case above?
a. A
b. B
c. C
d. D

197. A 25-years-old woman comes to family physician for health screening and based on the
result from anamnesis, physical examination and laboratory examination found that all
value in the within normal limit. Which of the following sentence is the most correct
area for the case above?
a. A
b. B
c. C
d. D

198. A 60-years-old woman comes to family physician with loss of body weight since 5
months ago with mild fever and malaise. Based on the result from anamnesis, physical
examination and laboratory examination found that some value in abnormal limit.
Which of the following sentence is the most correct area for the case above?
a. A
b. B
c. C
d. D

199. A 70-years-old woman with sleeping disorders comes to family physician and based on
the result from anamnesis, physical examination and laboratory examination found that
Tim Akademik ANANTARA 2015
Divisi SL (Soal Latihan)
Fakultas Kedokteran Universitas Padjadjaran

all value in the within normal limit. Which of the following sentence is the most correct
area for the case above?
a. A
b. B
c. C
d. D

200. A 30-years-old woman without any sign and symptoms but based on laboratory
examination found abnormal value for SGOT and SGPT. Which of the following
sentence is the most correct area for the case above?
a. A
b. B
c. C
d. D

ANANTARA
TROPICAL MEDICINE – UTS

1. Tobacco use is estimated to contribute to 440.000 deaths per year in the US and
associated with 10 million cases of chronics disease. In other hand there are 15 million
alcoholics in the US and 100.000 deaths attributed to alcohol abuse each year.
Which one responsible for tobacco addiction?
a. Arsenic
b. Acetaldehyde
c. Nitrogen dioxide
d. Hydrogen cyanide
e. Nicotine

2. A 34-year-old male, from Bandung city, presents to the clinic with the chief complaint
of fever for 15 days. The fever start by low grade fever, and then step ladder fever,
followed by continues high grade fever. He had never gone to the East Indonesian
province or to the beach area. Physical examination revealed alert, high grade fever,
relative bradicardia, and tongue tremor.
Laboratory result of that patients : Tubex T (+)
What is the possible diagnosis ?
a. Dengue Haemoragic Fever.
b. Leptospirosis.
Tim Akademik ANANTARA 2015
Divisi SL (Soal Latihan)
Fakultas Kedokteran Universitas Padjadjaran

c. Thypoid Fever.
d. Filariasis.
e. Malaria.

3. The following manifestation is very rare presenting features of severe malaria in


children:
a. Multiple convulsions.
b. Hypoglycemia.
c. Renal failure.
d. Severe anemia.
e. Lactic acidosis.

4. A 36-year-old man presented to ER with jaundice in the last 3 days. Symptom started 7
days before with gradually onset of fever with muscle ache especially in the calves,
headache and nausea.
What laboratory examination do you want to perform to establish the diagnosis?
a. Blood ureum, creatinin and CBC.
b. Blood alkali phosphatase and glucose.
c. Blood creatinin, lipid and haemoglobine.
d. Blood glucose and CBC.
e. Blood cholesterol and ureum.

5. Seven days ago, a 54-year old woman developed a right shoulder abscess. The doctor
took the specimen from the location of infection and performed culture on the media
followed by identification.
The result showed that the organism was Staphylococcus sp. based on the following
criteria:
a. Produce beta-lactamase.
b. Encapsulated.
c. Catalase positive.
d. Gram negative cocci.
e. Produce alpha-haemolytic.

6. A 22-year-old man came to you with chief complaints of experienced high-grade


intermittent fever every each two day that last for several weeks two months ago. The
fevers disappear after he took antimalaria drug given by his uncle. He feels better now
but he still weak. From physical examination you found hepatosplenomegaly.
Which of the most appropriate drug?
a. Quinine.
b. Mefloquine.
c. Doxicycline.
d. Primaquine.
Tim Akademik ANANTARA 2015
Divisi SL (Soal Latihan)
Fakultas Kedokteran Universitas Padjadjaran

e. Chloroquine.

7. You are seeing a 5-month-old child who has had 1 DTaP, 1 IPV and 1 Hibat 2 months
of age.
When would you schedule the next visit?
a. 2 weeks later.
b. 4 weeks later.
c. 8 weeks later.
d. 12 weeks later.
e. -

8. A 25-years-old, male present to the emergency department with chief complain of


having fever for 2 days. He felt the fever occur all day, and only relief after taking
antipyretic drugs. Headache and myalgia were complained as additional symptoms that
occurred. There was no abnormality for urination and defecation.
Physical examination:
General : Fever (38oC), chills, fatigue
Skin : Petechial rash on lower extremities
GI : Abdominal pain
Ext : Myalgia
An hour later, the laboratory result of the patient:
WBC 6.6 ; Hb 14.8 ; Hct 41 ; Plt 80.000
Differential:
5% Bands, 42% Segmented Neutrophils, 26% Lymphocytes.
13% Monocytes, 8% Eosinophils, 6% Variably Lymphocytes
INR 1.0 ; PT 11.9 ; PTT 34.9
Electrolytes normal
AST 302 ; ALT 157
Total Bilirubin 0.9, Alkaline Phosphatase 95
Urinalysis unremarkable
Stool occult blood positive
On the 6 day of admission, the patient is checked for Widal test, and the result as
below:
S. typhii - O : 1/160
S. typhii - H : 1/320
S. paratyphii A - O : 1/80
S. paratyphii A - H : 1/40
S. paratyphii B - O : Non reaktif
S. paratyphii B - H : 1/40
S. paratyphii C - O : Non reaktif
S. paratyphii C - H : 1/80
On the 13 day of admission, the patient is re-checked for Widal test, and the result as
below:
Tim Akademik ANANTARA 2015
Divisi SL (Soal Latihan)
Fakultas Kedokteran Universitas Padjadjaran

S. typhii - O : 1/320
S. typhii - H : 1/1280
S. paratyphii A - O : 1/80
S. paratyphii A - H : 1/80
S. paratyphii B - O : 1/80
S. paratyphii B - H : 1/40
S. paratyphii C - O : 1/80
S. paratyphii C - H : 1/80
What does it mean?
a. Probable case for Salmonella typhii infection.
b. Confirm case for Salmonella typhii infection.
c. Probable case for Salmonella paratyphii infection.
d. Confirm case for Salmonella paratyphii infection.
e. Probable case for both Salmonella typhii and paratyphii infection.

9. There are food poisoning cases among 40 students who gave lunch at Boy Scout
activity. 30 of them get nausea and vomiting, 10 of them got diarrhea and vomiting.
There were true for xenobiotics
a. Xenobiotics are lipophobic
b. Xenobiotics are hidrofilic
c. Xenobiotics are lipophofilic
d. Xenobiotics are amphofilic
e. Xenobiotics are amphoter

10. A 40-year-old man with G6PD deficiency came to you in the clinic. He told you that he
will be travelling to endemic area of malaria in Nusa Tenggara Timur.
Which of the most appropriate antimalaria agent for him?
a. Quinine.
b. Mefloquine.
c. Doxicycline.
d. Primaquine.
e. Chloroquine.

11. Which microorganism is a common etiology in endotoxic shock?


a. Staphylococcus aureus.
b. Streptococcus pyogenes.
c. Streptococcus pneumoniae
d. Escherichia coli.
e. -

12. A 5-year-old girl is brought into the hospital for evaluation of a sore throat and fever,
which she has had for about 4 days. She is the daughter of parents of low economic
Tim Akademik ANANTARA 2015
Divisi SL (Soal Latihan)
Fakultas Kedokteran Universitas Padjadjaran

status, she has not had much medical care in her life, and her immunization status is
unknown. On examination of her pharynx reveals tonsillar and pharyngeal edema with
presence of a gray membrane coating of the tonsil, this extends over the uvula and soft
palate. You immediately consult to pediatrician and diagnose as diphtheria. What is the
name of special staining for identifying the specific characteristic of bacteria that cause
the disease?
What is the name of special staining for identifying the specific characteristic of
bacteria that cause the disease?
a. Ziehl-Neelsen staining.
b. Gram staining.
c. Neisser staining.
d. Burri-Gins staining.
e. Negative staining.

13. A 21-year-old female came with two days fever. Her vital sign revealed blood pressure
110/80 mmHg, heart rate 104 bpm, temperature 38 oC, respiration rate 28 per minute.
Which of the following mechanism that most likely for her vital sign condition?
a. Increasing temperature lead to increase cardiac output.
b. Infection process lead to respiratory alkalosis.
c. Fever cause excitement of sympathetic system.
d. Stimulated pacemaker cells by increase temperature.
e. Cytotoxic of infectious agent increase metabolic process.

14. A 24-year-old man admitted to hospital because of sudden onset of high fever. This
complain also accompanied by headache, jaundice, in last 3 days. The patient worked
as cleaning service in a warehouse where there were many rats and exposed to polluted
water. Blood test shows renal impairment, increased bilirubin and liver function test.
What is the definitive laboratory test needed to confirm diagnosis?
a. Macroscopic inhibition test.
b. Urine culture on human fibroblast cell.
c. Microscopic agglutination test.
d. Tube agglutination test.
e. Culture of CSF on blood and chocolate agar.

15. A 39-year-old man presents to you with a 10 days history of headache, fever, chills,
sweats and myalgia. Physical examination reveals a fever of 40°C, a rapid pulse rate
and generalized sweating. Blood examination with thin blood smear revealed intra-
erythrocytic microorganism with typical banana shape.
Which of the following statement describe the causing agent?
a. Causing agent is female Anopheles sp.
b. Causing agent intra-erythrocytic cycle occur in every day
c. Causing agent always infects young red blood cell
Tim Akademik ANANTARA 2015
Divisi SL (Soal Latihan)
Fakultas Kedokteran Universitas Padjadjaran

d. Causing agent infective stage is gametocyte


e. Causing agent is the only plasmodium that has hypnozoitestate

16. A 41-year-old man working as a freelance news photographer from The Netherlands
flies to Indonesia. After 5 days in the field he returns home and 4 days later experiences
the abrupt onset of fever, chills, headache, nausea and lower back pain. His symptoms
subside after 3 days and the man assumes he has recovered but 24 hours later his fever
returns along with vomiting and abdominal pain. In the emergency department (ED), he
notices several spontaneously appearing bruises on his arms and lower legs and
bleeding from his gums. Vital signs are temperature 39.2 °C, blood pressure (BP) 110
/60 mm Hg, and heart rate 55 beats per minute and respiration rate 16 breaths per
minute. The man is ill appearing with markedly injected conjunctivae. There is some
oozing gingival bleeding and sparsely distributed ecchymoses on the lower arms and
legs. Chest is clear to auscultation and percussion. Abdominal examination shows
hepatomegaly and right upper quadrant tenderness to palpation. Stool sample is heme
positive. Available laboratory tests are shown: Hemoglobin (Hb) 20 g/dL, Hematocrit
(Hct) 55%, Platelets 40k/uL, White blood count (WBC) 2700/µL, neutrophils: 30%,
bands: 1%, lymphocytes: 65%
Which of the following is cytopathic effect of the causative agent of the disease?
a. Causing morphological changes of the cells.
b. Often associated with changes in mitochondrial membranes.
c. Can be seen using Gram staining.
d. Rarely associated with cell death.
e. Can only be seen with electron microscope.

17. A 12-years-old boy who lives in dormitory comes to yor clinic with chief complain
pruritic skin lesion. He feels very itchy especially at night. Skin Lession distribute
generally mainly on the finger webs, wrist, axillary fold and scortum.
On direct microscopical examination from skin scraping revealed mite and fecal pellets.
How did he acquired the infection?
a. Ingestion Food or water contaminated with eggs
b. Inhalation air containing of larva
c. Trematode
d. By the transfer of the impregnated females during person-to-person, skin-to-skin
contact
e. Inhalation air containing eggs

18. The etiology of Scabies has the following characteristic:


a. Only the female species of all stages sucks blood.
b. Produce itch of the skin at scalp and pubic area.
c. Lateral lobes has no function as motor organ.
d. One of the stage has 4 pairs of legs.
Tim Akademik ANANTARA 2015
Divisi SL (Soal Latihan)
Fakultas Kedokteran Universitas Padjadjaran

e. The eggs are glued to the hair shaft.

19. Miss R died after being hospitalised for 4 days and she was diagnosed as suffering from
Avian Influenza. You were informed beforehand that in the area where she lived, there
were cases of deaths of poultry. However, you believe that the area had never been an
endemic area, and there had never been any case from there.
What will you do to the body if there is no staff in mortuary who can manage infectious
dead body?
a. Send the body for cremation, since it is the most recommended by the World
Health Organization.
b. Embalm, so that the body will be sterilized, then bathe the body with hypochlorite
solution.
c. Bathe the body with hypochlorite solution.
d. Bathe the body with soap and water, and then give the deceased to the family.
e. Bathe the body with soap and water and put the deceased in the tight-sealed coffin.

20. Which of the following is NOT a characteristic of the diphtheria organism?


a. Non motile
b. Gram negative
c. Gram positive
d. Aerobic
e. Producing spore

21. A 21-year-old girl is admitted to the hospital because of fever, headache, weakness, and
anorexia. Ten days prior to admission she had diarrhea illness that lasted about 36
hours. She has been constipated for the last 3 days.
What is the best specimen for culture to detect the causative agent of the disease?
a. Cerebrospinal fluid.
b. Blood.
c. Stool.
d. Urine.
e. Bone marrow.

22. You are seeing a 24-month-old with no immunizations. What immunization will you
give now:
a. BCG without PPD test.
b. DTP, HiB, HepB, measles, and OPV/IPV.
c. Varicella.
d. Typhoid vaccine.
e. -
Tim Akademik ANANTARA 2015
Divisi SL (Soal Latihan)
Fakultas Kedokteran Universitas Padjadjaran

23. Siti, a 17-year-old female was admitted to the hospital due to 14 days of fever
accompanied by malaise, chills, headache, and constipation. Blood count showed
leukopenia and blood antibody for Salmonella typhi was positive. On 2nd day of
hospitalization she complained severe abdominal pain. She was diagnosed to have
peritonitis due to ileal perforation. The emergency surgery was undertaken and the
tissue sample from perforation areas was sent to Pathology Department.
What is the most closely associated histopathology finding :
a. Grabuloma.
b. Molluscum bodies.
c. Koilositosis.
d. Erytrophage.
e. Eosinofil.

24. This activity is used to determine changes in the geographical distribution and density
of the vectors, evaluate control programmes, obtain relative measurements of the vector
population over time and facilities appropriate and timely decisions regarding
interventions.
This activity is known as:
a. Biological control.
b. Environmental management.
c. Vector surveillance.
d. Vector control.
e. Larvae eradication.

25. This following statement is FALSE regarding clinical features of anthrax:


a. Occupational exposure to animals and animal products.
b. An incubation period of 1-3 weeks.
c. A painless cutaneous papule.
d. Gastroenteritis and bronchopneumonia.
e. Multiple antibiotic resistance.

26. During the febrile phase of DHF, a patient with history of febrile convulsion should be
given :
a. Salicylic acid 10 mg/kg BW.
b. Ibuprofen 10 mg/kg BW.
c. Pyramidon 10-15 mg/kg BW.
d. Acetaminophen 10-15 mg/kg BW.
e. Metampiron 10 mg/kg BW.

27. What is the case definition of typhoid fever?


a. Confirmed case is a patient with fever that lasted for at least three days, with a
positive serodiagnosis or antigen detection test but without S. Typhi isolation.
Tim Akademik ANANTARA 2015
Divisi SL (Soal Latihan)
Fakultas Kedokteran Universitas Padjadjaran

b. Probable case of is a patient with fever that lasted for at least three days, with a
laboratory-confirmed positive culture (blood, bone marrow, bowel fluid) of S.
Typhi.
c. Chronic carrier is excretion of S.typhi in stool or urine for longer than 1 month
after the onset of acute typhoid fever.
d. Chronic carrier is excretion of S.typhi in stool or urine for longer than 1 year after
the onset of acute typhoid fever.
e. -

28. Thomson, a 45-year-old man, was a resident of Sukabumi city. He has suffered from
malaria since he was a boy. 6 days ago he experienced fever and the doctor told him
that it is malaria that cause the symptom.
The clinical manifestations of the cyclic fever in this case are caused by the:
a. Pre-erythrocytic phase.
b. Hepatic stage.
c. Erythrocytic stage.
d. Sexual stage.
e. -

29. You are seeing a 5-month-old child who has had 1 DTaP, 1 IPV and 1 Hibat 2 months
of age.
What would you give at that time?
a. DTaP3, IPV3, Hib3, HepB2.
b. OPV 3.
c. MMR.
d. Hep B2.
e. -

30. A 25-years-old, male present to the emergency department with chief complain of
having fever for 2 days. He felt the fever occur all day, and only relief after taking
antipyretic drugs. Headache and myalgia were complained as additional symptoms that
occurred. There was no abnormality for urination and defecation.
Physical examination:
General : Fever (38oC), chills, fatigue
Skin : Petechial rash on lower extremities
GI : Abdominal pain
Ext : Myalgia
An hour later, the laboratory result of the patient:
WBC 6.6 ; Hb 14.8 ; Hct 41 ; Plt 80,000
Differential:
5% Bands, 42% Segmented Neutrophils, 26% Lymphocytes.
13% Monocytes, 8% Eosinophils, 6% Variably Lymphocytes
Tim Akademik ANANTARA 2015
Divisi SL (Soal Latihan)
Fakultas Kedokteran Universitas Padjadjaran

INR 1.0 ; PT 11.9 ; PTT 34.9


Electrolytes normal
AST 302 ; ALT 157
Total Bilirubin 0.9, Alkaline Phosphatase 95
Urinalysis unremarkable
Stool occult blood positive
What is the stage of plasmodium that can occur in mosquito?
a. Trophozoite, merozoite, schizonte, oocyst.
b. Trophozoite, schizonte, oocyst, ookinete.
c. Oocyst, sporozoite, merozoite, ookinete.
d. Trophozoite, sporozoite, oocyst, ookinete.
e. Sporozoite, oocyst, ookinete, macrogametocyte.

31. A previously healthy 17-year-old girl comes to the physician because of a 1-week
history of itching and progressive rash. She has no history of skin problems or
associated symptoms. She takes no medications. Her sister with whom she shares a
room had similar symptoms during the previous week. The patient's temperature is
36.8°C. There are multiple 2- to 5-mm erythematous papules over the trunk, especially
at the waistline, and over the forearms, hands, and fingers. There is no
lymphadenopathy or hepatosplenomegaly.
Which cells and which signaling molecules are responsible for initiating an
inflammatory response in this case?
a. Phagocytes: lysozymes.
b. Phagocytes: chemokines.
c. Dendritic cells: interferons.
d. Mast cells: histamines.
e. Lymphocytes: interferons.

32. Miss R died after being hospitalised for 4 days and she was diagnosed as suffering from
Avian Influenza. You were informed beforehand that in the area where she lived, there
were cases of deaths of poultry. However, you believe that the area had never been an
endemic area, and there had never been any case from there.
Will you consider that this might be an act of bioterrorism? Why?
a. Yes, because it happened not in the endemic area and it never happened there.
b. No, because avian influenza is not an unusual disease and it can happen anywhere.
c. Yes, because all microorganism can be used as bioterrorism agent.
d. No, because it is impossible to obtain the virus.
e. Yes, because avian influenza virus is an ideal bioterrorism agent.

33. A 30-year-old first trimester pregnant woman came to the doctor for consultation. She
will be visiting Papua, which is an endemic area of malaria next week.
Which of antimalaria agent can be given safely?
Tim Akademik ANANTARA 2015
Divisi SL (Soal Latihan)
Fakultas Kedokteran Universitas Padjadjaran

a. Quinine.
b. Mefloquine.
c. Doxicycline.
d. Primaquine.
e. Chloroquine.

34. A 25-years-old, male present to the emergency department with chief complain of
having fever for 2 days. He felt the fever occur all day, and only relief after taking
antipyretic drugs. Headache and myalgia were complained as additional symptoms that
occurred. There was no abnormality for urination and defecation.
Physical examination:
General : Fever (38oC), chills, fatigue
Skin : Petechial rash on lower extremities
GI : Abdominal pain
Ext : Myalgia
An hour later, the laboratory result of the patient:
WBC 6.6 ; Hb 14.8 ; Hct 41 ; Plt 80,000
Differential:
5% Bands, 42% Segmented Neutrophils, 26% Lymphocytes.
13% Monocytes, 8% Eosinophils, 6% Variably Lymphocytes
INR 1.0 ; PT 11.9 ; PTT 34.9
Electrolytes normal
AST 302 ; ALT 157
Total Bilirubin 0.9, Alkaline Phosphatase 95
Urinalysis unremarkable
Stool occult blood positive
What is the next test should be done as diagnostic test for this case?
a. Complete blood count, as screening test.
b. Widal test, as screening and confirmatory test.
c. Antigen NS1 Dengue, as confirmatory test.
d. Antibody IgM-IgG Dengue, as screening test.
e. IgM Leptospira, as screening and confirmatory test.

35. Zulkfli, a 14-year-old male, previously healthy, presents to the emergency department
with a one week history of fever, malaise, abdominal discomfort, calf pain and a mild
sore throat. He denies any cough, hemoptysis, dyspnea, chills, night sweats, anorexia,
nausea, vomiting, or dysuria. His urine output is good. There is no history of trauma,
blood transfusions, or recent travel.
He lives with his parents. His father is a fisherman and his mother is a housewife. He
likes to swim in streams and canals. He denies substance abuse and sexual activity.
His vital sign is normal except he has a high temperatur of 38.4C,. He is alert in no
acute distress. He is noted to have a moderate conjunctival suffusion, with scleral
icterus. His pharynx is injected, but his tonsils are not enlarged. His neck is supple
Tim Akademik ANANTARA 2015
Divisi SL (Soal Latihan)
Fakultas Kedokteran Universitas Padjadjaran

with 1 cm anterior lymphadenopathy. Heart is regular with no murmur. His lungs are
clear. His abdomen has normoactive bowel sounds with tender hepatomegaly; but no
rebound or guarding. His extremity muscle strength is 4/5. His lower extremity
muscles are tender to palpation. No nodules are palpable.
Labs: WBC 9,000 with 80% polymorphonuclear leukocytes, hemoglobin 14 g/dL,
platelet count 60,000 /dl. Coagulation times are normal. BUN 70 mg/dL, creatinine 3
mg/dL, total bilirubin 10 mg/dL, direct bilirubin 8 mg/dL, AST 80 UI/L, ALT 70 U/L. .
Urine analysis shows ketones >160mg/dL, >100 red blood cells per high-powered field.
No pyuria, organisms or casts are seen on UA. Chest and abdominal radiographs are
normal.
He is admitted to the hospital and started on IV penicillin empirically for possible
leptospirosis. Culture and serology studies are pending.
Leptospira are best cultured from:
a. Blood.
b. Plasma.
c. Urine.
d. Csf.
e. None of the above.

36. What is the CORRECT statement regarding Herpes zoster infection?


a. Gamma Interferon is an effective treatment.
b. Produces latent infection within the anterior horn cells.
c. Causes urinary incontinence.
d. Causes congenital limb deformity.
e. Associated pneumonitis is equally common in smokers and nonsmokers.

37. A 1½-years-old unimmunized male child came to emergency department presented


with generalized edema since 6 day. 15 days back he had an episode of fever with bull
neck and generalized tonic-clonic convulsion. His investigation at that time revealed
Hb of 10 gm%, WBC count 20.100 cell/cu mm. At this time x-ray chest revealed
cardiomegaly with bilateral infiltrates in both lower zones.
What is the most likely diagnosis of this patient when he came 15 days back?
a. Diphteriae
b. Peritonsiler abscess
c. Perimadibular abscess
d. Croup
e. Encephalitis

38. Tobacco use is estimated to contribute to 440.000 deaths per year in the US and
associated with 10 million cases of chronics disease. In other hand there are 15 million
alcoholics in the US and 100.000 deaths attributed to alcohol abuse each year.
What is environmental tobacco smoke:
a. Tobacco abuse
Tim Akademik ANANTARA 2015
Divisi SL (Soal Latihan)
Fakultas Kedokteran Universitas Padjadjaran

b. Active smoking
c. Second-hand smoke
d. Maternal smoking
e. Exogen smokers

39. Criteria for dengue shock syndrom?


a. Pulse pressure >20 mmhg with increased diastolic pressure
b. Pulse pressure <20 mmhg with decreased diastolic pressure
c. Pulse pressure >20 mmhg with increased diastolic pressure
d. Hypotension by age, defined as systolic pressure <90 mmhg for those aged <5
years
e. Hypotension by age, defined as systolic pressure <80 mmhg forthose aged <5 years

40. A 32-year-old man has lived for many years in Surabaya, mostly outdoors.
Which of the following include adaptation he exhibits to this very hot environment?
a. A large increase in the maximal rate of sweating.
b. Decreases in the mass of brown adipose tissue.
c. Decreases in plasma aldosterone levels.
d. Facilitation of the stretch reflex.
e. Increases in the plasma thyroxine levels.

41. A 24-year-old man admitted to hospital because of sudden onset of high fever. This
complain also accompanied by headache, jaundice, in last 3 days. The patient worked
as cleaning service in a warehouse where there were many rats and exposed to polluted
water. Blood test shows renal impairment, increased bilirubin and liver function test.
What cells are most responsible to the reaction occurred to this patient?
a. Mast cells.
b. Schwann Cells.
c. B Cells.
d. T Cells.
e. Hepatocyte.

42. A 25-years-old, male present to the emergency department with chief complain of
having fever for 2 days. He felt the fever occur all day, and only relief after taking
antipyretic drugs. Headache and myalgia were complained as additional symptoms that
occurred. There was no abnormality for urination and defecation.
Physical examination:
General : Fever (38oC), chills, fatigue
Skin : Petechial rash on lower extremities
GI : Abdominal pain
Ext : Myalgia
What is most important laboratory examination that should performed?
Tim Akademik ANANTARA 2015
Divisi SL (Soal Latihan)
Fakultas Kedokteran Universitas Padjadjaran

a. Complete blood count, as screening test.


b. Widal test, as screening and confirmatory test.
c. Antigen NS1 Dengue, as screening test.
d. Antibody IgM-IgG Dengue, as confirmatory test.
e. IgM Leptospira, as screening and confirmatory test.

43. Which of the following is the most important step to prevent nosocomial infection?
a. Gloving.
b. Gowning.
c. Hand washing.
d. Room cleaning.
e. Maskering.

44. Shanty, a 7-year-old girl, came to the hospital with high fever for 7 days, platelet count,
white blood cell count and hematocrit level were 59,000/mm3 , 2,100/mm3 , and 48%
respectively She looked restlessness, body temperature was 36.00C, blood pressure
was 70/55 mmHg, pulse rate 156/minute.
This dengue virus infection is classified as :
a. Dengue fever.
b. Dengue haemorrhagic fever grade I.
c. Dengue haemorrhagic fever grade II.
d. Dengue haemorrhagic fever grade III.
e. Dengue haemorrhagic fever grade IV.

45. A four-month-old baby girl took by her mom to outward clinic for immunization. She
had previous immunization : BCG, Polio 0, Hepatitis B 0, Hepatitis B + HiB + DTP +
Polio 1, Hepatitis B + HiB + DTP + Polio 2. No adverse events were recorded.
What is your suggestion for her next visit?
a. Her next immunization is 5 month later to get measles immunization.
b. Her next immunization is 2 month later to get measles immunization.
c. This is her last immunization.
d. Her next immunization is 14 month later to get measles immunization.
e. -

46. What is the current routine method for finding the etiology from the skin?
a. Using cellophane tape method.
b. Inserting needle into the tunnels on the skin.
c. Injecting black Indian ink into the burrows to identify the canalization.
d. Examining skin scraping microscopically.
e. Use physical examination to examine the eruptions on the skin.
Tim Akademik ANANTARA 2015
Divisi SL (Soal Latihan)
Fakultas Kedokteran Universitas Padjadjaran

47. A 28-year-old woman has a fever of 40oC as a result of Influenza. Which of the
following is likely to occur during the fever?
a. Cutaneous vasoconstriction.
b. Reduction of hypothalamic set-point temperature.
c. Decrease in shivering.
d. Increase in sweating.
e. Strong subjective sensation of increased heat.

48. Culex quinquefasciatus may act as vector of the worm with nocturnal periodicity.
What is the most likely worm?
a. Brugia timori.
b. Wuchereria bancrofti.
c. Loa loa.
d. Oncocerca volvulus.
e. Brugia malayi.

49. A six-years-old girl presents with a 3-day history of fever, headache, malaise, and
upper respiratory trac infection symptoms including cough, coryza, conjungtivitis. Two
days ago she develop an erythematous rash on her face (starting at the hair line) wich
later spread throughout her body
What is the most likely diagnosis of this patient?
a. Varicella
b. Roseola
c. Measles
d. Scabies
e. Parvovirus

50. Thomson, a 45-year-old man, was a resident of Sukabumi city. He has suffered from
malaria since he was a boy. 6 days ago he experienced fever and the doctor told him
that it is malaria that cause the symptom.
Prophylaxis for this disease includes all of the following except:
a. Chloroquine.
b. Mefloquine.
c. Permethrin impregnated mosquito nets.
d. 35% deet.
e. Avoiding mosquitoes during the day.

51. A 37-year-old man came to Primary Health Care with psoriasis as the chief complain.
The patient has been taking antimalarial agent for the last two weeks.
Which of the most possible antimalaria he took?
a. Quinine.
b. Mefloquine.
Tim Akademik ANANTARA 2015
Divisi SL (Soal Latihan)
Fakultas Kedokteran Universitas Padjadjaran

c. Doxicycline.
d. Primaquine.
e. Chloroquine.

52. A 1½-year-old unimmunized male child came to emergency department presented with
generalized edema since 6 day. 15 days back he had an episode of fever with bull neck
and generalized tonic-clonic convulsion. his investigation at that time revealed Hb of 10
gm%, WBC count 20,100 cell/cu mm. at this time x-ray chest revealed cardiomegaly
with bilateral infiltrates in both lower zones
What is the most likely diagnosis of this patient right now?
a. Nefrotic syndrome.
b. Myocarditis.
c. Pericarditis.
d. GNAPS.
e. Cirrhosis hepatic.

53. A one-year-old male presents with one day history of a non prurutic rash. He had five
days of preceding fever but is afebrile currently. He is otherwise healthy. Physical exam
reveals 2-3 mm erythematous pink macules and papules arranged in rosettes. The rash
begins on trunk and spreads to face, neck and extremities. A white halo surround
lesions
What is the most common complication of this disease?
a. Pneumonia
b. Diarrhea
c. Convulsions
d. Miokarditis
e. Hepatitis

54. Zulkfli, a 14-year-old male, previously healthy, presents to the emergency department
with a one week history of fever, malaise, abdominal discomfort, calf pain and a mild
sore throat. He denies any cough, hemoptysis, dyspnea, chills, night sweats, anorexia,
nausea, vomiting, or dysuria. His urine output is good. There is no history of trauma,
blood transfusions, or recent travel.
He lives with his parents. His father is a fisherman and his mother is a housewife. He
likes to swim in streams and canals. He denies substance abuse and sexual activity.
His vital sign is normal except he has a high temperatur of 38.4C,. He is alert in no
acute distress. He is noted to have a moderate conjunctival suffusion, with scleral
icterus. His pharynx is injected, but his tonsils are not enlarged. His neck is supple
with 1 cm anterior lymphadenopathy. Heart is regular with no murmur. His lungs are
clear. His abdomen has normoactive bowel sounds with tender hepatomegaly; but no
rebound or guarding. His extremity muscle strength is 4/5. His lower extremity
muscles are tender to palpation. No nodules are palpable.
Tim Akademik ANANTARA 2015
Divisi SL (Soal Latihan)
Fakultas Kedokteran Universitas Padjadjaran

Labs: WBC 9,000 with 80% polymorphonuclear leukocytes, hemoglobin 14 g/dL,


platelet count 60,000 /dl. Coagulation times are normal. BUN 70 mg/dL, creatinine 3
mg/dL, total bilirubin 10 mg/dL, direct bilirubin 8 mg/dL, AST 80 UI/L, ALT 70 U/L. .
Urine analysis shows ketones >160mg/dL, >100 red blood cells per high-powered field.
No pyuria, organisms or casts are seen on UA. Chest and abdominal radiographs are
normal.
He is admitted to the hospital and started on IV penicillin empirically for possible
leptospirosis. Culture and serology studies are pending.
More characteristic findings in the immune phase of anicteric leptospirosis include:
a. Fever.
b. Jaundice.
c. Renal failure.
d. Aseptic meningitis.
e. -

55. A 39-year-old man presents at the emergency room with a 10 days history of headache,
fever, chills, sweats and myalgia. Physical examination reveals a fever of 40oC , a rapid
pulse rate and generalized sweating. Blood examination with thin blood smear revealed
intraerythrocytic microorganism . Doctor diagnose him as having Malaria tropica.
Which of the following stages of parasite is most likely to be found from a peripheral
blood sample of this patient?
a. Amuboid.
b. Schizont mature.
c. Gametocytes.
d. Macrogamet.
e. Schizont immature.

56. A 27-year-old man was admitted to hospital because of sudden onset of fever to 39oC
headache and jaundice in the last 3 days. The patient had history of working in a
warehouse where there were many rats, and exposed to polluted water. Blood tests done
shortly after admission indicated renal function abnormality, elevated bilirubin and
liver function test.
Which of the following is the characteristic of this microorganism?
a. Rod with microgranular form.
b. Spirochetes gram negative.
c. Rod gram positive.
d. Cocci gram positive.
e. Spirochetes with hooked ends.

57. A 25-years-old, male present to the emergency department with chief complain of
having fever for 2 days. He felt the fever occur all day, and only relief after taking
Tim Akademik ANANTARA 2015
Divisi SL (Soal Latihan)
Fakultas Kedokteran Universitas Padjadjaran

antipyretic drugs. Headache and myalgia were complained as additional symptoms that
occurred. There was no abnormality for urination and defecation.
Physical examination:
General : Fever (38oC), chills, fatigue
Skin : Petechial rash on lower extremities
GI : Abdominal pain
Ext : Myalgia
An hour later, the laboratory result of the patient:
WBC 6.6 ; Hb 14.8 ; Hct 41 ; Plt 80,000
Differential:
5% Bands, 42% Segmented Neutrophils, 26% Lymphocytes.
13% Monocytes, 8% Eosinophils, 6% Variably Lymphocytes
INR 1.0 ; PT 11.9 ; PTT 34.9
Electrolytes normal
AST 302 ; ALT 157
Total Bilirubin 0.9, Alkaline Phosphatase 95
Urinalysis unremarkable
Stool occult blood positive.
If the result of dengue fever antibody panel: IgM positive and IgG negative. What does
it mean?
a. Primary infection of dengue viral.
b. Secondary infection of dengue viral.
c. Reinfection of dengue viral.
d. Non-dengue infection / other viral infection.
e. False positive of dengue viral infection.

58. Mala, a 23-year-old girl from Cianjur city came to Emergency department with the
chief complaint of fever for 6 days. The fever was high grade and intermittent. The
fever accompanying with chills, and sweating. She went to NTT one month ago and
stayed there for 1 week. Physical examination revealed alert, pale, and hepato-
splenomegaly. Lab Test showed anemia hemolytic.
A period of 48 hours is required for the development of the mature schizont, which
merozoites large and number of merozoites usually 12 -18 merozoites. What is the most
likely parasite?
a. Plasmodium falciparum
b. Plasmodium ovale
c. Plasmodium vivax
d. Plasmodium malariae
e. Plasmodium knowlesi

59. DTP Vaccine consist of:


a. Diphtheria toxoid, tetanus toxoid, and pertussis toxoid.
b. Diphtheria toxoid, live attenuated tetanus, and pertussis toxoid.
Tim Akademik ANANTARA 2015
Divisi SL (Soal Latihan)
Fakultas Kedokteran Universitas Padjadjaran

c. Diphtheria toxoid, tetanus toxoid, and inactive pertussis.


d. Inactive diphtheria, tetanus toxoid, and inactive pertussis.
e. -

60. A 35-year-old man develops high fever two weeks after the big float occurred in his
house. He also complains jaundice, red eye, fatigue, and severe headache. From
physical examination reveals hepatomegaly.
Which of the following is the most suitable medium culture for above causative
microbes?
a. Feeley Gorman Blood.
b. Saboraud Glucose Agar.
c. Egg yolk agar.
d. Fletcher Semisolid agar.
e. Eosin metylen blue agar.

61. 50-year-old man has white patches at his chest and upper extremity. There was no
itching or pain at the lesion. Histopathology leprosy examination may be interpreted a
granulomatous not developed, but consists of only a few chronic inflammation
What is the most appropriate diagnosis?
a. Borderline tuberculoid leprosy.
b. Borderline lepromatous leprosy.
c. Indeterminate leprosy.
d. True borderline leprosy.
e. Tuberculoid leprosy.

62. Daryl, 9-year-old boy, experienced a high sustained fever for 5 days accompanied by
headache, retroorbital pain, myalgia and arthralgia. Repetitive hematocrit level
examination on day 5, 6, and 7 are 40%, 42%, and 42% respetively, and platelet count
are 140,000/mm3, 112,000/mm3, and 89,000/mm3 respectively. Tourniquette Test
revealed more than 20 petechiae on the observed area.
This dengue virus infection is classified as :
a. Undifferentiated fever.
b. Dengue fever.
c. Dengue haemorrhagic fever grade I.
d. Dengue haemorrhagic fever grade II.
e. Dengue haemorrhagic fever grade III.

63. Tobacco use is estimated to contribute to 440.000 deaths per year in the US and
associated with 10 million cases of chronics disease. In other hand there are 15 million
alcoholics in the US and 100.000 deaths attributed to alcohol abuse each year.
There are carcinogenic metal content in ciggarete. Which one the correct constituents
a. Arsenic
Tim Akademik ANANTARA 2015
Divisi SL (Soal Latihan)
Fakultas Kedokteran Universitas Padjadjaran

b. Acetaldehyde
c. Nitrogen dioxide
d. Hydrogen cyanide
e. Carbonmonoxyde

64. IL-1, IL-6 and TNF alpha released by macrophages act on the hypothalamus and induce
fever. The doctor give antipyretic if the fever if the temperature reach 38 oC or above.
Fever helps immune system fight the infection.
Which of the following condition that most likely for above statement?
a. Most pathogen grow better at temperature below 37 oC.
b. Human cells become more resistant to TNF alpha at temperature above 38 oC.
c. Adaptive immunity becomes more potent at low temperature.
d. Pyrogens help sympathetic activity to increase.
e. Hypothalamus set point will not decrease if there is no pyrogen activators.

65. Because of increasing resistance to the commonly used insecticides among important
vector species, this alternative effort can be taken such as involving the alteration of the
breeding sites of the vectors.
This effort is known as;
a. Biological control.
b. Environmental management.
c. Vector surveiilance.
d. Vector control.
e. Larva eradication.

66. A 36-year-old man presented to ER with jaundice in the last 3 days. Symptom started 7
days before with gradually onset of fever with muscle ache especially in the calves,
headache and nausea.
If we suspect this patient to have Leptosirosis, what is the definitive laboratory
examination to confirm the diagnosis in the serovar level?
a. Macroscopic inhibition test.
b. Slide peroxidase test.
c. Microscopic agglutination test.
d. Tube agglutination test.
e. Coagulation test.

67. Which statement is false regarding criteria for diagnosis dengue haemorrhagic fever?
a. Acute onset of fever two to seven days duration
b. Haemorrhagic manifestations
c. Platelet count <100.000 cells/mm3
d. Objective evidence of plasma leakage due increased vascular permeability show by
haemorrhagic manifestations
Tim Akademik ANANTARA 2015
Divisi SL (Soal Latihan)
Fakultas Kedokteran Universitas Padjadjaran

e. Evidence of plasma leakage such as pleural effusion, ascites or hypoproteinaemia

68. A number of different principles apply to the immediate management of a child in


septic shock. In general, management should be prioritized in order of urgency.
Which of the following is not an immediate priority in the resuscitation phase of a child
in septic shock (2)?
a. Ensure adequate airway support.
b. Correct anemia.
c. Administer volume resuscitation.
d. Cardiovascular support.
e. Empiric antibiotic treatment.

69. A young soldier comes to you with a recurrence fever for every 3 days. He noticed that
he got the same fever pattern after he came back from military service in the Papua half
a year ago. He got treated and showing no symptoms since then. Blood examination
with thin blood smear revealed intra-erythrocytic microorganism with typical banana
shape.
What is the most likely parasite infected this soldier?
a. Plasmodium falciparum
b. Plasmodium ovale
c. Plasmodium vivax
d. Plasmodium malariae
e. Plasmodium knowlesi

70. A 1½-years-old unimmunized male child came to emergency department presented


with generalized edema since 6 day. 15 days back he had an episode of fever with bull
neck and generalized tonic-clonic convulsion. His investigation at that time revealed
Hb of 10 gm%, WBC count 20.100 cell/cu mm. At this time x-ray chest revealed
cardiomegaly with bilateral infiltrates in both lower zones.
What is the most likely diagnosis of this patient right now?
a. Nefrotic syndrome
b. Myocarditis
c. Pericarditis
d. GNAPS
e. Cirrhosis hepatic

71. Nurhasanah, a 19-year-old girl came with her mother to the primary health clinic. They
lived in an epidemic area of filariasis. She heard that everyone should take some drugs
for the disease prevention. The doctor gave Nurhasanah and her mother
diethylcarbamazine citrate (DEC) and albendazole.
What is the most reasonable factor why the government chooses this type of
combination for mass therapy?
Tim Akademik ANANTARA 2015
Divisi SL (Soal Latihan)
Fakultas Kedokteran Universitas Padjadjaran

a. The drugs are effective to lower the microfilarial rate.


b. Relatively safe with minimal side effect.
c. Single dose therapy.
d. Wide therapeutic range with low cost.
e. Cheap and easy to access.

72. Tuti, a 7-year-old girl, admitted to R.S dr. Hasan Sadikin Hospital as having Dengue
Virus infection. She looked restlessness, body temperature 38.60C, blood pressure was
100/75 mmHg, pulse pressure 100/minute, platelet count 67,000/mm3 and hematocrit
level 36%.
What is the correct treatment in this case?
a. Acetaminophen 10 mg/Kg bodyweight .
b. Salycilate 10 mg/Kg bodyweight .
c. Phenobarbital to prevent convulsion.
d. Plain water by mouth is preferable than fruit juice.
e. Lactated Ringer i.v.f.d 10 cc/ Kg bodyweight.

73. The type of malaria that can recurs as relapse because of hypnozoit stage in the liver :
a. Vivax and falciparum malaria.
b. Vivax and ovale malaria.
c. Vivax and malariae malaria.
d. Falciparum and ovale malaria.
e. Falciparum and malarie malaria.

74. A 26-year-old male came to the primary health care with left leg swelling for the last 5
years. He lives in an endemic area of filariasis. He also diagnoses with advanced left
leg elephanthiasis.
What is the main method to control lymphatic filariasis?
a. Mosquito control.
b. Physiotherapy.
c. Health education.
d. Mass chemotherapy.
e. Hygiene and sanitation.

75. Male, a 26-year-old male native of Soreang Bandung West Java, went to the primary
health care because of scrotal swelling of 3 years' duration. About 3 years prior to the
visit the patient noted the onset of painless small size scrotal swelling on the right side.
The patient had no other complaints and no other past medical history. Some of his
neighbours have the same condition, with foot swelling. Other physical examination
were within normal limits. Laboratory finding positive skin prick test.
Which is proper laboratory examination bellow?
a. Microscopic agglutination test.
Tim Akademik ANANTARA 2015
Divisi SL (Soal Latihan)
Fakultas Kedokteran Universitas Padjadjaran

b. IgM anti Dengue.


c. Widal test.
d. IgM anti leptospira.
e. Thin and thick blood film smears.

76. A six-years-old girl presents with a 3-day history of fever, headache, malaise, and
upper respiratory trac infection symptoms including cough, coryza, conjungtivitis. Two
days ago she develop an erythematous rash on her face (starting at the hair line) wich
later spread throughout her body.
What is the most likely etiology of this disease?
a. Paramyxovirus
b. Parvovirus
c. Human herpes virus 6 and 7
d. Varicella-Zoster virus
e. Epstein-Barr virus

77. A 36-year-old woman, from Bandung city, present to the clinic with the chief complaint
of fever for 5 days. The fever is high grade, intermittent every each two day. Typically
it is started by chills, followed by high fever and sweating. She had ever gone to
Borneo, two month ago and stayed there for 6 week. Plasmodium vivax were found on
blood smear examination.
What is the effective prevention of this disease?
a. Mass use of Fansidar chemoprophylaxis.
b. Use of mosquito nets.
c. Vaccination.
d. Vitamin.
e. Body lotion.

78. A 15-year-old boy presented with fever for 7 days with chills, headache, nausea, and
abdominal discomfort. Physical examination: T= 38.5oC with relative bradicardia.
Laboratory result show leucopenia. A thyphoid fever diagnosis is make. Widal test is
perform in the first day of admission.
The next Widal test is best done at ?
a. 2-4 days after the first Widal test.
b. 1-2 weeks after the first Widal test.
c. 3-4 weeks after the first Widal test.
d. 4-5 weeks after the first Widal test.
e. >6 weeks after the first Widal test.

79. Indonesia Minister of Health decided to use dapson as mass treatment for leprosy,
although many studies show that this drug increase in resistance.
Which of the following is the most likely reason for that condition?
Tim Akademik ANANTARA 2015
Divisi SL (Soal Latihan)
Fakultas Kedokteran Universitas Padjadjaran

a. It inhibits protein synthesis.


b. Resistance can emerge if very high doses are given.
c. Minimal renal side effect
d. Dapsone is well tolerated.
e. Dapsone can not be given to pregnant women.

80. A 34-year-old male, from Bandung city, presents to the clinic with the chief complaint
of fever for 15 days. The fever start by low grade fever, and then step ladder fever,
followed by continues high grade fever. He had never gone to the East Indonesian
province or to the beach area. Physical examination revealed alert, high grade fever,
relative bradicardia, and tongue tremor.
What is the best prevention of this case?
a. Improved sanitation.
b. Providing clean towel.
c. Health epidemiology survey.
d. Improving personality.
e. Education to the family member.

81. Which of this statement false regarding diagnostic tests for dengue?
a. Dengue viraemia in patient is short, typically occurs 24 hours prior to the onset and
lasts for 48 hours of illness
b. Igm antibodies are detectable by 3-5 days after onset of illness
c. Igg antibodies are detectable at low level by the end of the first week
d. Because of the late appearance of igm antibody, serological test based on this
antibody done during the first five days of clinical illness are usually negative
e. During the secondary dengue infection, igg antibodies are detectable at high levels

82. A 25-years-old, male present to the emergency department with chief complain of
having fever for 2 days. He felt the fever occur all day, and only relief after taking
antipyretic drugs. Headache and myalgia were complained as additional symptoms that
occured. There was no abnormality for urination and defecation.
Physical examination:
General : Fever (38oC), chills, fatigue
Skin : Petechial rash on lower extremities
GI : Abdominal pain
Ext : Myalgia
An hour later, the laboratory result of the patient:
WBC 6.6 ; Hb 14.8 ; Hct 41 ; Plt 80.000
Differential:
5% Bands, 42% Segmented Neutrophils, 26% Lymphocytes.
13% Monocytes, 8% Eosinophils, 6% Variably Lymphocytes
INR 1.0 ; PT 11.9 ; PTT 34.9
Tim Akademik ANANTARA 2015
Divisi SL (Soal Latihan)
Fakultas Kedokteran Universitas Padjadjaran

Electrolytes normal
AST 302 ; ALT 157
Total Bilirubin 0.9, Alkaline Phosphatase 95
Urinalysis unremarkable
Stool occult blood positive
On the 6 day of admission, the patient is checked for Widal test, and the result as
below:
S. typhii - O : 1/160
S. typhii - H : 1/320
S. paratyphii A - O : 1/80
S. paratyphii A - H : 1/40
S. paratyphii B - O : Non reaktif
S. paratyphii B - H : 1/40
S. paratyphii C - O : Non reaktif
S. paratyphii C - H : 1/80
From the result of Widal test as above, which one is more important to evaluate?
a. The H antigen, represent for the flagella of the bacteria.
b. The O antigen, represent for the capsule of the bacteria.
c. The O antigen, represent for the flagella of the bacteria.
d. The H antigen, represent for the capsule of the bacteria.
e. The O and H antigen, represent for capsule of the bacteria.

83. A 10-year-old boy is admitted to the hospital because of fever, headache, and anorexia.
Ten days prior to admission, he had diarrhea that lasted about 36 hours. He has been
constipated for the last 3 days. Physical examination shows BP 110/70 mmHg,
Temperature 38.,1 C, RR 20x/min, Pulse Rate 88x/min. CBC performed. Hb: 12.5
gr/dL, Ht: 38%, Leukocyte: 2500/mm3, Thrombocyte: 165,000/mm3.
What is the best specimen to do the serologic test in this case?
a. Blood.
b. Stool.
c. CSF.
d. Urine.
e. Bone marrow.

84. There are millions of cases of leprosy (Hansen's disease) worldwide, but predominately
in tropical countries of Asia and Africa. The clinical spectrum of Hansen's disease is
best characterized by peripheral neuritis.
How is the morphology of the bacteria that cause the disease?
a. Gram negative rods.
b. Acid fast bacilli.
c. Gram positive non sporeforming rods.
d. Gram negative cocci.
e. Gram positive sporeforming rods.
Tim Akademik ANANTARA 2015
Divisi SL (Soal Latihan)
Fakultas Kedokteran Universitas Padjadjaran

85. You observed a 40-year-old man begging on a street. He had clawing of the fourth and
fifth digits with loss of distal parts of the digits of both hands, strongly leprosy.
Which of the following regarding causative agent is true?
a. It is susceptible to isoniazid and rifampin.
b. It grows in parts of the body that are cooler than 37o C.
c. It can be cultured in the laboratory using Middlebrook 7H11 medium.
d. It is seen in high numbers in biopsies of tuberculoid leprosy lesions.
e. It commonly infects people live surrounding armadillos.

86. A 20-year-old man is seen in the university walk-in health clinic for evaluation of
recurrent fever. He reports fever greater than 101°F lasting less than a day occurring
approximately weekly for the past 3 weeks. He feels otherwise relatively poorly with
diffuse myalgias and headache that are much worse during the febrile episodes. Of
note, he returned recently from a mission trip to Central America and reports not taking
malaria prophylaxis. Examination of the peripheral smear confirms the diagnosis of
Plasmodium vivax.
If present, which of the following findings indicates that the patient has severe malaria
and is not a candidate for outpatient therapy?
a. Fever >40°C.
b. One seizure last week.
c. Parasitemia of 300,000 /uL on peripheral smear.
d. Serum bilirubin level of > 1 mg/dL.
e. The presence of headache.

87. Dede visit the hospital with fever as chief complain, he experiencing high fever for 3
days, and a gap of reduced fever for one day, followed by an additional high fever for 2
days.
What type of fever does this patient has?
a. Continous fever.
b. Saddle back ( biphasic ) fever.
c. Intermittent fever.
d. Remittent fever.
e. Relapsing fever.

88. A 22-year- old mother come to the Primary Health Care to have her son vaccine. The
schedule is for DTP vaccine.
What does DTP vaccine consists?
a. Diphteria, pertussis and tetanus toxoid.
b. Diphteria, pertussis and tetanus toxoid.
c. Diphteria , tetanus and pertussis live bacteria.
d. Diphteria , tetanus and pertussis inactivated bacteria.
Tim Akademik ANANTARA 2015
Divisi SL (Soal Latihan)
Fakultas Kedokteran Universitas Padjadjaran

e. Diphteria and tetanus toxoid, and inactivated pertussis bacteria.

89. A 25-years-old, male present to the emergency department with chief complain of
having fever for 2 days. He felt the fever occur all day, and only relief after taking
antipyretic drugs. Headache and myalgia were complained as additional symptoms that
occurred. There was no abnormality for urination and defecation.
Physical examination:
General : Fever (38oC), chills, fatigue
Skin : Petechial rash on lower extremities
GI : Abdominal pain
Ext : Myalgia
An hour later, the laboratory result of the patient:
WBC 6.6 ; Hb 14.8 ; Hct 41 ; Plt 80,000
Differential:
5% Bands, 42% Segmented Neutrophils, 26% Lymphocytes.
13% Monocytes, 8% Eosinophils, 6% Variably Lymphocytes
INR 1.0 ; PT 11.9 ; PTT 34.9
Electrolytes normal
AST 302 ; ALT 157
Total Bilirubin 0.9, Alkaline Phosphatase 95
Urinalysis unremarkable
Stool occult blood positive
On the 6 day of admission, the patient is checked for Widal test, and the result as
below:
S. typhii - O : 1/160
S. typhii - H : 1/320
S. paratyphii A - O : 1/80
S. paratyphii A - H : 1/40
S. paratyphii B - O : Non reaktif
S. paratyphii B - H : 1/40
S. paratyphii C - O : Non reaktif
S. paratyphii C - H : 1/80
What does it mean?
a. Probable case for Salmonella typhii infection.
b. Confirm case for Salmonella typhii infection.
c. Probable case for Salmonella paratyphii infection.
d. Confirm case for Salmonella paratyphii infection.
e. Probable case for both Salmonella typhii and paratyphii infection.

90. A one-year-old male presents with one day history of a non prurutic rash. He had five
days of preceding fever but is afebrile currently. He is otherwise healthy. Physical exam
reveals 2-3 mm erythematous pink macules and papules arranged in rosettes. The rash
Tim Akademik ANANTARA 2015
Divisi SL (Soal Latihan)
Fakultas Kedokteran Universitas Padjadjaran

begins on trunk and spreads to face, neck and extremities. A white halo surround
lesions
What is the most likey diagnosis of this patient
a. Varicella
b. Roseola
c. Measles
d. Scabies
e. Parvovirus

91. A 41-year-old man working as a freelance news photographer from The Netherlands
flies to Indonesia. After 5 days in the field he returns home and 4 days later experiences
the abrupt onset of fever, chills, headache, nausea and lower back pain. His symptoms
subside after 3 days and the man assumes he has recovered but 24 hours later his fever
returns along with vomiting and abdominal pain. In the emergency department (ED), he
notices several spontaneously appearing bruises on his arms and lower legs and
bleeding from his gums. Vital signs are temperature 39.2 °C, blood pressure (BP) 110
/60 mm Hg, and heart rate 55 beats per minute and respiration rate 16 breaths per
minute. The man is ill appearing with markedly injected conjunctivae. There is some
oozing gingival bleeding and sparsely distributed ecchymoses on the lower arms and
legs. Chest is clear to auscultation and percussion. Abdominal examination shows
hepatomegaly and right upper quadrant tenderness to palpation. Stool sample is heme
positive. Available laboratory tests are shown: Hemoglobin (Hb) 20 g/dL, Hematocrit
(Hct) 55%, Platelets 40k/uL, White blood count (WBC) 2700/µL, neutrophils: 30%,
bands: 1%, lymphocytes: 65%
What is the most widely used serologic test needed to diagnose the disease?
a. Coagulation test.
b. Widal test.
c. NS1.
d. Thick blood smear.
e. MAT.

92. Sammy, a 17-year-old girl present with 9 days of fever which was recurrent in the first
7 days and sustained in the last two days. The fever accompanied with malaise,
anorexia, myalgia, frontal headache, and abdominal pain. She frequent eats at side walk
restaurant surrounding her school. On physical examination reveal acutely ill, relative
bradycardia, high fever, abdominal distended with hepatosplenomegaly, and rose spot
on lower chest and upper abdomen. CBC revealed leucopenia with aneosinophilia on
differential counting. Working diagnosis of the patient is typhoid fever and treated with
Chloramphenicol, blood culture was drawn. Five days later the blood culture reveals
Salmonella typhi.
What is the most serious side effect of the drug?
a. Megaloblastic anemia.
Tim Akademik ANANTARA 2015
Divisi SL (Soal Latihan)
Fakultas Kedokteran Universitas Padjadjaran

b. Aplastic anemia.
c. Hemolytic anemia.
d. Iron deficiency anemia.
e. Sideroblastic anemia

93. A 65-year-old man develops an abscess on the back of his neck. Culture yields grape-
like clusters with large, group and golden-yellow colonies.
What is characteristic of the isolate?
a. Grow on Mac Conkey agar.
b. Gram negative cocci.
c. Catalase negative.
d. Coagulase positive.
e. Resistant to Novobiocin.

94. A one-year-old male presents with one day history of a non prurutic rash. He had five
days of preceding fever but is afebrile currently. He is otherwise healthy. Physical exam
reveals 2-3 mm erythematous pink macules and papules arranged in rosettes. The rash
begins on trunk and spreads to face, neck and extremities. A white halo surround
lesions
What is the most likely etiology of this disease?
a. Paramyxovirus
b. Parvovirus
c. Human herpes virus 6 and 7
d. Varicella-zoster virus
e. Epstein-Barr virus

95. A 23-year-old man develops a perirectal abscess. This is drained surgically. A


specimen is cultured and grows anaerobic bacteria.
What is the most likely sign for anaerobic bacteria as etiology?
a. Positive in condition facultative anaerobic culture.
b. Special cotton swab can be used for specimen collection .
c. Foul smelling discharge.
d. Infect to mucosal area.
e. Can be grown on Mac Conkey agar.

96. A one-year-old male presents with one day history of a non prurutic rash. He had five
days of preceding fever but is afebrile currently. He is otherwise healthy. Physical exam
reveals 2-3 mm erythematous pink macules and papules arranged in rosettes. The rash
begins on trunk and spreads to face, neck and extremities. A white halo surround
lesions
Etiology of hand-foot and mouth disease
a. Coxsackievirus A16 and parvovirus B19
Tim Akademik ANANTARA 2015
Divisi SL (Soal Latihan)
Fakultas Kedokteran Universitas Padjadjaran

b. Enterovirus 71 and Coxsackievirus A16


c. Enterovirus A16 and parvovirus B19
d. Parvovirus B19
e. Paramyxoviridae

97. An 8-year-old boy develops a severe sore throat. On examination, grayish exudate is
seen over the tonsils and pharynx. The differential diagnosis of this symptom includes
group A streptococcal infection and diphtheria.
The cause of this boy symptom is most likely:
a. Gram-negative rods.
b. Gram-negative cocci.
c. Catalase-positive Gram-positive cocci arrange in clusters.
d. Club-shaped Gram-positive rods.
e. Gram positive spore-forming rods.

98. Larva stage has only one row comb scale in the nineth of abdominal segmen, and adult
stage doesn't have "lyre " in mesonotum.
Which of the following is the most likely mosquito?
a. Anopheles barbirostris.
b. Aedes albopictus.
c. Aedes aegypti.
d. Culex fatigans.
e. Anopheles maculates.

99. Which of the following is NOT found in the current vaccine for diphtheria?
a. Inactivated diphtherotoxin
b. Inactivated tetanus toxin
c. Pertussis toxin
d. Endotoxin
e. There is no correct answers

100. You are seeing a 5-month-old child who has had 1 DTaP, 1 IPV and 1 Hibat 2 months
of age.
What immunizations would you give now?
a. Measles.
b. MMR.
c. DTaP2, IPV2, Hib2, HepB1.
d. No schedule.
e. -

101. Leprosy patient is given dapson 50 mg daily for maintenance.


Which of the following is most likely drug distribution within human body?
Tim Akademik ANANTARA 2015
Divisi SL (Soal Latihan)
Fakultas Kedokteran Universitas Padjadjaran

a. Body fluids and brain


b. Tissues and nerves
c. Body fluids and tissues.
d. CNS and PNS
e. Skin and nerves

102. Tobacco use is estimated to contribute to 440.000 deaths per year in the US and
associated with 10 million cases of chronics disease. In other hand there are 15 million
alcoholics in the US and 100.000 deaths attributed to alcohol abuse each year.
Which one tumor promoter
a. Arsenic
b. Acetaldehyde
c. Nitrogen dioxide
d. Hydrogen cyanide
e. Nicotine

103. Tobacco use is estimated to contribute to 440.000 deaths per year in the US and
associated with 10 million cases of chronics disease. In other hand there are 15 million
alcoholics in the US and 100.000 deaths attributed to alcohol abuse each year.
There were true for ethanol abuse
a. Cirrhosis in chronicsetanol abuse
b. Cardiomyopathy in acute etanol abuse
c. Testicular hypertrophy
d. Strength myocyte
e. Fetal alcohol syndrome

104. A 50-year-old woman presented with sudden onset of decreased vision in her better left
eye. She had 1 week of redness, photophobia, pain, and decreased vision. She reported
no underlying systemic disease or recent health change and no family history of
unusual eye disease. She was not on medication.
She had developed severe vision loss in her right eye about 20 years prior from
recurrent attacks. She said that she grew up with a lot of cats that roamed the outdoors.
Her right eye was damaged in a motor vehicle accident at some point after it had
developed very poor vision from the parasitic infection.
Which of the following parasite which its infective stage is NOT transmitted through
cat feces?
a. Toxocaracati
b. Toxoplasma gondii
c. Ancylostomabraziliense
d. Toxocaracanis
e. Gnatostomaspinigerum
Tim Akademik ANANTARA 2015
Divisi SL (Soal Latihan)
Fakultas Kedokteran Universitas Padjadjaran

105. Which of the following is NOT found in the current vaccine for diphtheria?
a. Inactivated diphtherotoxin.
b. Inactivated tetanus toxin.
c. Pertussis toxin.
d. Endotoxin.
e. -

106. Sammy, a 17-year-old girl present with 9 days of fever which was recurrent in the first
7 days and sustained in the last two days. The fever accompanied with malaise,
anorexia, myalgia, frontal headache, and abdominal pain. She frequent eats at side walk
restaurant surrounding her school. On physical examination reveal acutely ill, relative
bradycardia, high fever, abdominal distended with hepatosplenomegaly, and rose spot
on lower chest and upper abdomen. CBC revealed leucopenia with aneosinophilia on
differential counting. Working diagnosis of the patient is typhoid fever and treated with
Chloramphenicol, blood culture was drawn. Five days later the blood culture reveals
Salmonella typhi.
The blood culture showed that the Salmonella typhi is resistant to chlorampenicol, and
the fever hasn't resolved yet. The doctor decides to change the antibiotic which has
mechanism mostly as bactericidal in normal dose.
Which of the following antibiotics the doctor will give?
a. Trimeptoprim-sulfametoxazol.
b. Tetracycline.
c. Azithromycin.
d. Chlarithromycin.
e. Erythromicin.

107. A 39-year-old man presents at the emergency room with a 10 days history of headache,
fever, chills, sweats and myalgia. Physical examination reveals a fever of 40°C, a rapid
pulse rate and generalizedsweating. Blood examination with thin blood smear revealed
intra erythrocytic microorganism.Doctor diagnose him as having Malaria tropica.
Which of the following stages of parasite is most likely to be found from a peripheral
blood sample of this patient?
a. Amuboid
b. Schizont mature
c. Gametocytes
d. Macrogamet
e. Schizont immature

108. Microscopic examination of a Thin blood smear from a patient suspected of having
Malaria reveals numerous normal size erythrocytes without stippling but with ring
stages, many with multiple ring stages and appliquéd forms. Several erythrocytes show
developing trophozoites that are spread across the erythrocytes in a bandpattern.
Tim Akademik ANANTARA 2015
Divisi SL (Soal Latihan)
Fakultas Kedokteran Universitas Padjadjaran

Which of the following is the most likely cause of infection?


a. Mixed infection Plasmodium falciparum and Plasmodium vivax.
b. Plasmodium malariae.
c. Plasmodium falciparum.
d. Mixed infection Pl.malariae and Pl. Vivax
e. Mixed infection Pl. falciparum and Pl. malariae.

109. A 12-year-old boy presents to ER with high fever for the last 5 days accompanied by
headache, myalgia and epigastric pain. The body temperature is range from 38.6 to
39.4oC, occurs all day and night. On the 3rd day the body temperature decreased but on
the next day elevated again. Physical examination revealed high fever, petechiae, and
positive tourniquet test. Laboratory findings revealed Hb 15.8 g/dl, WBC 3,400/mm3,
Hematocrit level 48%, and platelet count 80,000/mm3.
The result of rapid test for Dengue infection is C-line and M-line are pink color.
a. Negative.
b. Primary infection.
c. Secondary infection.
d. Invalid.
e. Primary and secondary infection.

110. Sammy, a 17-year-old girl present with 9 days of fever which was recurrent in the first
7 days and sustained in the last two days. The fever accompanied with malaise,
anorexia, myalgia, frontal headache, and abdominal pain. She frequent eats at side walk
restaurant surrounding her school. On physical examination reveal acutely ill, relative
bradycardia, high fever, abdominal distended with hepatosplenomegaly, and rose spot
on lower chest and upper abdomen. CBC revealed leucopenia with aneosinophilia on
differential counting. Working diagnosis of the patient is typhoid fever and treated with
Chloramphenicol, blood culture was drawn. Five days later the blood culture reveals
Salmonella typhi.
a. Half life of phenytoin is prolonged.
b. The serum concentration of phenytoin are increased.
c. Half life of phenytoin is prolonged.
d. Chloramphenicol does not inhibits hepatic microsomal enzymes
e. Both serum concentration are increased

111. A 25-years-old, male present to the emergency department with chief complain of
having fever for 2 days. He felt the fever occur all day, and only relief after taking
antipyretic drugs. Headache and myalgia were complained as additional symptoms that
occurred. There was no abnormality for urination and defecation.
Physical examination:
General : Fever (38oC), chills, fatigue
Skin : Petechial rash on lower extremities
Tim Akademik ANANTARA 2015
Divisi SL (Soal Latihan)
Fakultas Kedokteran Universitas Padjadjaran

GI : Abdominal pain
Ext : Myalgia
An hour later, the laboratory result of the patient:
WBC 6.6 ; Hb 14.8 ; Hct 41 ; Plt 80,000
Differential:
5% Bands, 42% Segmented Neutrophils, 26% Lymphocytes.
13% Monocytes, 8% Eosinophils, 6% Variably Lymphocytes
INR 1.0 ; PT 11.9 ; PTT 34.9
Electrolytes normal
AST 302 ; ALT 157
Total Bilirubin 0.9, Alkaline Phosphatase 95
Urinalysis unremarkable
Stool occult blood positive
If we have a thin blood smear for this patient, what examination could we check to
differentiae with malaria infection?
a. Identify the erythrocyte morphology, sexual stage of plasmodium.
b. Identify the sexual/asexual stage of plasmodium, parasite count/index.
c. Identify the leukocyte morphology, parasite count/index.
d. Identify the asexual stage of plasmodium, parasite count/index.
e. Identify the asexual stage of plasmodium, the erythrocyte morphology.

112. Tia, experiencing fever for 14 days, during the first week the fever is not too high, the
highest temperature was found in the late evening and during night; in the morning the
fever slightly reduced but not reached the normal body temperature.
What type of fever does this patient has?
a. Continous fever.
b. Saddle back ( biphasic ) fever.
c. Intermittent fever.
d. Remittent fever.
e. Relapsing fever.

113. A four-month-old baby girl took by her mom to outward clinic for immunization. She
had previous immunization : BCG, Polio 0, Hepatitis B 0, Hepatitis B + HiB + DTP +
Polio 1, Hepatitis B + HiB + DTP + Polio 2. No adverse events were recorded.
Choose one of the following for her immunization?
a. Measles.
b. MMR.
c. Hepatitis B + HiB + DTP + Polio 3.
d. TT.
e. -

114. Which one of the following is not a parameter in the definition of SIRS?
a. Hypotension.
Tim Akademik ANANTARA 2015
Divisi SL (Soal Latihan)
Fakultas Kedokteran Universitas Padjadjaran

b. Tachycardia.
c. Tachypnea.
d. Leukocytosis.
e. Hypothermia.

115. A 9-month-old infant, 7 kg body weight, has been giving corticosteroid therapy 15 mg
per day for 2 weeks. This infant is brought to your clinic for routine immunization; he
had only BCG immunization at 1 month old.
Which of the following immunization should be administered at this time?
a. Measles virus vaccine and DTP.
b. Measles virus vaccine and DTaP.
c. Measles virus vaccine and Hepatitis.
d. DTP and Oral Polio Vaccine (OPV).
e. DTP and Hepatitis B.

116. A 25-year-old male patient presents to dermato-veneorological out-patient clinic with


anaesthetic hypopigmentation macule and paresthetic on his left hand. After taking
history, physical examination, and preliminary investigation are done, he is diagnosed
with tuberculoid leprosy, with neuritis left ulnar nerve later. He is treated with
rifampicin 600 mg once in a month and dapson 100 mg daily. After treatment the
patient develop some hemolysis problem.
Which of the following is the reason for that condition?
a. Gastrointestinal intolerance.
b. G6PD.
c. Methemoglobinemia.
d. Erythema nodosum.
e. Lipolisis.

117. A 1-month-old healthy infant is brought to the Primary Care Clinic for routine
examination. The infant has never had immunization before.
Which of the following immunization should be administered at this time?
a. BCG, OPV, Hepatitis B.
b. BCG, DTP, Hepatitis B.
c. DTaP, OPV, Hepatitis B.
d. DTaP, IPV, Hepatitis B.

118. A 1½-year-old unimmunized male child came to emergency department presented with
generalized edema since 6 day. 15 days back he had an episode of fever with bull neck
and generalized tonic-clonic convulsion. his investigation at that time revealed Hb of 10
gm%, WBC count 20,100 cell/cu mm. at this time x-ray chest revealed cardiomegaly
with bilateral infiltrates in both lower zones
Which of the following is NOT a characteristic of the diphtheria organism?
Tim Akademik ANANTARA 2015
Divisi SL (Soal Latihan)
Fakultas Kedokteran Universitas Padjadjaran

a. Non motile.
b. Gram negative.
c. Gram positive.
d. Aerobic.
e. Anaerobic.

119. What is the anaerobic bacterium that causes antibiotic associated diarrhea and also
pseudomembranous colitis?
a. Clostridium perfringens
b. C. botulinum
c. C. difficile
d. Propionibacterium sp
e. Prevotella sp

120. Which of the following skin examination findings is generally not associated with
sepsis?
a. Pyogenic granuloma.
b. Ecthyma gangrenosum.
c. Purpura fulminans.
d. Petechiae.
e. -

121. There are food poisoning cases among 40 students who gave lunch at Boy Scout
activity. 30 of them get nausea and vomiting, 10 of them got diarrhea and vomiting.
There were true for phase I reactions
a. The P-450 system is located in rough endoplasmic reticulum hepatosit
b. A Flavin containing monooxygen system is located in liver smooth endoplasmic
reticulum
c. Prostaglandin H synthase occurs in rough endoplasmic reticulum kidneys
d. Glucoronidation system
e. Biomethylation as a common pathway for xenobiotic metabolism

122. Chikungunya is one of the differential diagnosis of DHF. What is NOT one of sign and
symptom is this case?
a. Positive Tourniquette test.
b. Hepatomegaly.
c. Maculopapular rash.
d. Hypotension.
e. Myalgia.

123. A 22-year-old woman is diagnosed as struma diffuse toxic. Her vital sign is BP 120/80
mmHg, HR 120 x/mnt, RR: 32 x/mnt, T: 40.9 oC.
Tim Akademik ANANTARA 2015
Divisi SL (Soal Latihan)
Fakultas Kedokteran Universitas Padjadjaran

What is the mechanism of fever in this case?


a. Increased basal metabolic rate.
b. Lesion that involves the temperature regulating centers.
c. Infection.
d. Inflamation.
e. Idiopathic.

124. To grow Diphtheria, Loeffler serum agar, Tellurite agar and Blood agar have been
used. How are there agars designated
a. General purpose
b. Enriched
c. Differential
d. Selective
e. All statement above false

125. The drug of malaria which has action for preventing relaps and acting as gametocidal
is:
a. Chloroquine.
b. Primaquine.
c. Quinine.
d. Mefloquine.
e. Amodiaquine.

126. A 39-year-old man presents to you with a 10 days history of headache, fever, chills,
sweats and myalgia. Physical examination reveals a fever of 40oC, a rapid pulse rate
and generalized sweating. Blood examination with thin blood smear revealed intra-
erythrocytic microorganism with typical banana shaped
Which of the following statement is moast likely describe the causing agent ?
a. Female Anopheles sp. is the intermediate host.
b. Intra erythrocytic cycle occur in every day.
c. Infects young red blood cell.
d. Gametocyte is the infective stage.
e. Hepatocyte is the hipnozoit phase takes place.

127. The following condition or diseases is not associated with severe falciparum malaria in
paediatric:
a. Acute renal failure.
b. Acute respiratory distress.
c. Nephrotic syndrome.
d. Cerebral malaria.
e. Black water fever.
Tim Akademik ANANTARA 2015
Divisi SL (Soal Latihan)
Fakultas Kedokteran Universitas Padjadjaran

128. A 25-years-old, male present to the emergency department with chief complain of
having fever for 2 days. He felt the fever occur all day, and only relief after taking
antipyretic drugs. Headache and myalgia were complained as additional symptoms that
occurred. There was no abnormality for urination and defecation.
Physical examination:
General : Fever (38oC), chills, fatigue
Skin : Petechial rash on lower extremities
GI : Abdominal pain
Ext : Myalgia
What is the first hypothesis for this patient?
a. Dengue fever.
b. Dengue hemorrhagic fever.
c. Typhoid fever.
d. Leptospirosis.
e. Malaria.

129. In giving vaccine, there are active immunity as well as passive immunity. What is the
example of passive immunity?
a. Diphtheria-Tetanus toxoid.
b. MMR.
c. Influenza vaccine.
d. Botulism antitoxin.
e. -

130. A young soldier comes to you with a recurrence fever for every 2 days. He just came
back from military service in the Papua. Microscopic examination of a thin blood smear
from the patient reveals numerous normal size erythrocytes without stippling but with
ring stages, many with multiple ring stages.
What is the most likely parasite infected this soldier?
a. Plasmodium falciparum
b. Plasmodium ovale
c. Plasmodium vivax
d. Plasmodium malariae
e. Plasmodium knowlesi

131. Case definition of typhoid fever:


a. Confirmed case is a patient with fever that lasted for at least three days, with a
positive sserodiagnosis or antigen detection test but without S. Typhi isolation
b. Probable case of is a patient with fever that lasted for at least three days, with a
laboratory-confirmed positive culture (blood, bone marrow, bowel fluid) of S.
Typhi
Tim Akademik ANANTARA 2015
Divisi SL (Soal Latihan)
Fakultas Kedokteran Universitas Padjadjaran

c. Chronic carrier is excretion of S.typhi in stool or urine for longer than 1 month
after the onset of acute typhoid fever
d. Chronic carrier is excretion of S.typhi in stool or urine for longer than 1 year after
the onset of acute typhoid fever
e. There is no correct statement

132. A 51-year-old woman is diagnozed with Plasmodium falciparum malaria after returning
from Sulawesi. Her parasitemia is 6%, hematocrit is 21%, bilirubin is 7.8 mg/dl, and
creatinine is 2.7 mg/dl. She is stil making 60 ml of urine per hour.
Which one of the following regiment is recommended?
a. Intravenous Cephalosporin.
b. Intravenous Artesunate.
c. Intravenous Metronidazole.
d. Oral Metronidazole.
e. Oral Mebendazole.

133. Choose the most likely parasite for the following description
Man and monkey is the definitive host and Anopheles barbirostris is intermediate host.
a. Brugia timori
b. Wuchereria bancrofti
c. Loa loa
d. Oncocerca volvulus
e. Brugia malayi

134. Loki, 10-year-old boy has been hospitalized for having DSS. What is the correct the
criteria of discharge in this case?
a. He has been fever free for 24 hours with the administration of antipyretic.
b. On routine examination revealed platelet count is 46,000/mm3.
c. Respiratory rate 52/minute.
d. His urine output about 1cc/ kg body weight/hour.
e. The series of hematocrit level examinations is 38%, 44% and 46%.

135. Serology is one of the diagnostic methods in Virology. A criterion for diagnosing
primary infection in viral disease is:
a. 2 fold or more increase in tit2 fold or more increase in titer of IgG or total antibody
between acute and convalescent sera.
b. 4 fold or more increase in titer of IgG or total antibody between acute and
convalescent sera.
c. A single low titer of IgG (or total antibody).
d. Absence or slight increase in IgM.
e. Presence of IgG antibody.
Tim Akademik ANANTARA 2015
Divisi SL (Soal Latihan)
Fakultas Kedokteran Universitas Padjadjaran

136. Four members of a football team develop diarrhea due to Salmonella enteritidis. Eating
which food was the most likely source of the infection?
a. Chicken at a fast food outlet 20 hours earlier.
b. Fried rice at a takeaway 4 hours earlier .
c. Raw oysters in milk 6 hours earlier.
d. Raw egg at a hotel 24 hours earlier.
e. Soft cheeses 48 hours earlier.

137. A 25-years-old, male present to the emergency department with chief complain of
having fever for 2 days. He felt the fever occur all day, and only relief after taking
antipyretic drugs. Headache and myalgia were complained as additional symptoms that
occurred. There was no abnormality for urination and defecation.
Physical examination:
General : Fever (38oC), chills, fatigue
Skin : Petechial rash on lower extremities
GI : Abdominal pain
Ext : Myalgia
What does make the fever of this case occur all day?
a. The activation of humoral immunity.
b. The activation of cellular immunity.
c. The activation of both cellular and humoral immunity.
d. The activation of antigen of the pathogen.
e. The activation of antigen-antibody of the pathogen.

138. A six-years-old girl presents with a 3-day history of fever, headache, malaise, and
upper respiratory tract infection symptoms including cough, coryza, conjungtivitis.
Two days ago she develop an erythematous rash on her face (starting at the hair line)
wich later spread throughout her body
Laboratory finding of this disease:
a. Increase of total white blood cell count
b. Without complicated by bacterial infection Erytrocyte sedimentation rate and C-
reactive protein level are elevated
c. Decreased of lymphocytes more than neutrophils with increase of total white blood
cell count
d. Decreased of lymphocytes more than neutrophils with reduction of total white
blood cell count
e. Pancytopenia

139. A 9-year-old girl presented with decreased vision in the right eye for several weeks.
The patient was diagnosed with vitritis and optic neuritis in the right eye and underwent
a short course of systemic steroid therapy.
Tim Akademik ANANTARA 2015
Divisi SL (Soal Latihan)
Fakultas Kedokteran Universitas Padjadjaran

Laboratory evaluation showed elevated toxoplasma IgG levels and non-detectable


toxoplasma IgM levels. Her only known risk factor for exposure was a family cat.
Sexual breeding of Toxoplasma gondii occur in:
a. Dog
b. Cat
c. Rat
d. Goat
e. Monkey

140. Sammy, a 17-year-old girl present with 9 days of fever which was recurrent in the first
7 days and sustained in the last two days. The fever accompanied with malaise,
anorexia, myalgia, frontal headache, and abdominal pain. She frequent eats at side walk
restaurant surrounding her school. On physical examination reveal acutely ill, relative
bradycardia, high fever, abdominal distended with hepatosplenomegaly, and rose spot
on lower chest and upper abdomen. CBC revealed leucopenia with aneosinophilia on
differential counting. Working diagnosis of the patient is typhoid fever and treated with
Chloramphenicol, blood culture was drawn. Five days later the blood culture reveals
Salmonella typhi.
Which of the following is the antimicrobial activity used for this patient?
a. A bactericidal broad spectrum.
b. A potent inhibitor of microbial lipid synthesis.
c. It inhibits the peptidyl transferase step of protein synthesis.
d. It binds reversibly to the 30S sub unit of the bacterial ribosome.
e. It is active against aerobis gram positive and negative organism only.

141. A 34-year-old male, from Bandung city, presents to the clinic with the chief complaint
of fever for 15 days. The fever start by low grade fever, and then step ladder fever,
followed by continues high grade fever. He had never gone to the East Indonesian
province or to the beach area. Physical examination revealed alert, high grade fever,
relative bradicardia, and tongue tremor.
Which is proper laboratory examination bellow?
a. IgM anti dengue.
b. Widal test.
c. IgG anti leptospira.
d. Thick blood smear.
e. Urine profile.

142. Mia, experiencing fever for 2 weeks. The fever occurred every 3 days and preceded by
chills and followed by sweating. After that, fever subsides reached the normal body
temperature.
What type of fever does this patient has?
a. Continous fever.
Tim Akademik ANANTARA 2015
Divisi SL (Soal Latihan)
Fakultas Kedokteran Universitas Padjadjaran

b. Saddle back ( biphasic ) fever.


c. Intermittent fever.
d. Remittent fever.
e. Relapsing fever.

143. A 12-years-old boy who lives in dormitory comes to yor clinic with chief complain
pruritic skin lesion. He feels very itchy especially at night. Skin Lession distribute
generally mainly on the finger webs, wrist, axillary fold and scortum.
On direct microscopical examination from skin scraping revealed mite and fecal pellets.
What the diagnosis of this child?
a. Pediculosis corporis
b. Scabies
c. Pediculosis capitis
d. Tinea cruris
e. -

144. Which of the following is CORRECT regarding infection with Salmonella typhi ?
a. Children are particularly likely to become carriers .
b. Most carriers are female.
c. Faecal culture is almost always positive during the first week of illness.
d. Relapse does not occur if antibiotics are taken for 2 weeks .
e. Vaccinated individuals who develop the disease will have a mild illness.

145. A 10-year-old boy is admitted to the hospital because of fever, headache, and anorexia.
Ten days prior to admission, he had diarrhea that lasted about 36 hours. He has been
constipated for the last 3 days. Physical examination shows BP 110/70 mmHg,
Temperature 38.,1 C, RR 20x/min, Pulse Rate 88x/min. CBC performed. Hb: 12.5
gr/dL, Ht: 38%, Leukocyte: 2500/mm3, Thrombocyte: 165,000/mm3.
The serologic test performed will influenced by:
a. Environment.
b. Antiviral.
c. Vitamin C.
d. Vaccination.
e. Sanitation.

146. Some viruses can damaging central nervous system. Below are viruses that can infect
central nervous system, EXCEPT:
a. Herpes virus.
b. Picorna virus.
c. Rhabdovirus.
d. Filovirus.
e. Corona virus.
Tim Akademik ANANTARA 2015
Divisi SL (Soal Latihan)
Fakultas Kedokteran Universitas Padjadjaran

147. A two-year-old boy, 10 kg, visit emergency department with 4 days of high fever,
spontaneously petechiae, without vomiting. Laboratory results shown platelet count of
88,000/mm3 and hematocrit value was 36 %, further examination revealed a platelet
count of 66,000/mm3 and hematocrit of 48%.
The fluid replacement should be given:
a. Orally, since there is no vomit.
b. 750 cc/24 hours parenteral and adjust when needed.
c. 1000 cc/24 hours parenteral and adjust when needed.
d. 1250 cc/24 hours parenteral and adjust when needed.
e. 1500 cc/24 hours parenteral and adjust when needed.

148. A patient, 6-year-old boy, body weight 20 kg, with high fever last for 6 days revealed
haematocrit value of 48% and platelet count of 87,000/mm3 , he got lactated Ringer
solution intra venously 60 cc/ kg body weight/hour since he experienced vomiting. Six
hours later haematocrit value was 37% and platelet count was 57,000/mm3 .
What is your working diagnosis?
a. A Dengue fever.
b. Dengue haemorrhagic fever grade I.
c. Dengue haemorrhagic fever grade 3.
d. Silent Infection of Dengue Virus infection.
e. Undifferentiated fever.

149. Shelly, a 30-year-old 32-34 weeks pregnant woman present with 11 days of fever
which was recurrent in the first 8 days and sustained in the last three days. The fever
accompanied with malaise, anorexia, myalgia, frontal headache, and abdominal pain.
She frequent eats at side walk restaurant surrounding her workplace. On physical
examination reveal acutely ill, relative bradycardia, high fever, abdominal distended
with hepatosplenomegaly, and rose spot on lower chest and upper abdomen. Her CBC
revealed normal range results. Working diagnosis of the patient is typhoid fever.
The doctor decided to give the 2000 mg chlorampenicol treatment for 14 days.
How many antimicrobial capsules should be prescribed to this patient?
a. Chlorampenicol caps 500 mg No. LVI.
b. Chlorampenicol caps 250 mg No. LVI.
c. Chlorampenicol caps 500 mg No. XXVIII.
d. Chlorampenicol caps 500 mg No. XIV.
e. Chlorampenicol caps 500 mg No. LII.

150. A one-year-old male presents with one day history of a non prurutic rash. He had five
days of preceding fever but is afebrile currently. He is otherwise healthy. Physical exam
reveals 2-3 mm erythematous pink macules and papules arranged in rosettes. The rash
Tim Akademik ANANTARA 2015
Divisi SL (Soal Latihan)
Fakultas Kedokteran Universitas Padjadjaran

begins on trunk and spreads to face, neck and extremities. A white halo surround
lesions.
Which of this statement are false regarding of this disease?
a. The fever usually resolves acutely after 24 hours
b. The rash usually lasts 1-3 days but is often described as evanescent and may be
visible only for hours
c. The most characteristic laboratory findings are lower mean number of total white
blood cells, lymphocytes, and neutrophils
d. Specific antiviral therapy is not recommended for rutin cases
e. Generally self-limited illness associated with complete recovery

151. A 3-year-old boy was brought to the hospital with fever for 10 days. He went to several
clinics and had received symptomatic treatments with paracetamol and Erytromycin.
His mother denied any gastrointestinal symptom. Physical examination: Temperature =
39.5oC., mild anemia, hepatosplenomegaly.
Laboratory investigations: :
Hb 9.0 g/dL (decrease), ESR = 75 mm/hr (slightly increase). Other in normal limit
Widal: day 1 of admission (day 11 of fever): O titer 1:80, H titer 1:160, day 12 of
admission: O titer 1:320, H titer 1:320.
On day 6 of admission the culture which was taken on day 1 of admission revealed
Gram negative rod.
Which one of this following statement is true?
a. The patient was already treated with antibiotics does not effect Widal test.
b. The best sample for culture taken on day 1 of admission in this patient is urine.
c. Patients who have received vaccines against Salmonella may give false negative
reactions in Widal test.
d. The best sample for culture taken on day 1 of admission in this patient is stool.
e. Patients already treated with antibiotics does not effect culture rest.

152. Tobacco use is estimated to contribute to 440.000 deaths per year in the US and
associated with 10 million cases of chronics disease. In other hand there are 15 million
alcoholics in the US and 100.000 deaths attributed to alcohol abuse each year.
There were true for ethanol toxicity can cause :
a. Hyprtension
b. Testicular atrofi
c. Cerebellar degeneration
d. Fatty change
e. Spontan abortion

153. A 27-year-old athlete male visited the Dermatology clinic with the complaint of a white
patch of his skin mainly on the axilla, back, and the trunk. There was no itching.
Dermatological examination revealed multiple hypo pigmented macules covered by
Tim Akademik ANANTARA 2015
Divisi SL (Soal Latihan)
Fakultas Kedokteran Universitas Padjadjaran

fine scales. Woods lamp examination showed positive yellow fluorescence. He was
diagnosing as tinea versicolor.
Which of the following fungus is the cause of this disease?
a. Microsporum canis.
b. Malassezia furfur.
c. Trichophyton beigelii.
d. Epidermophyton werneckii.
e. Epidermophyton floccosum.

154. Male 32-year-old, from Bandung city, present to the clinic with the chief complaint of
fever for 6 days. The fever is high grade, intermittent every each two day. Typically it
is started by chills, followed by high fever and sweating. He went to Papua province
one month ago and stayed there for 4 week. Physical examination revealed alert, pale,
and hepato-splenomegaly. All stadia of Plasmodium falciparum were found on blood
smear examination.
Which following statement is true regarding the ethiology?
a. Not causes more severe disease in pregnancy.
b. Associated with recurrent relapses after initial treatment.
c. Causing less than 20% parasitaemia.
d. Not associated with thrombocytopaenia.
e. The only cause of cerebral malaria.

155. Your patient is a 13-year-old boy with a chief complaint of high fever for 3 days. Her
mother tells you that her son had bleeding from his nose, a symptom of headache and
some red spots in her forearm. Laboratory examination shows trombocytopenia.
Which of the following is the characteristic of virus causes the disease?
a. Transmitted by arthropod vectors.
b. Usually resistant to ether.
c. Usually cause symptomatic infection.
d. Closely related to parvoviruses.
e. Can not grow in a cell culture.

156. A 36-year-old woman, from Bandung city, present to the clinic with the chief complaint
of fever for 5 days. The fever is high grade, intermittent every each two day. Typically
it is started by chills, followed by high fever and sweating. She had ever gone to
Borneo, two month ago and stayed there for 6 week. Plasmodium vivax were found on
blood smear examination.
Which following statement is true regarding the etiology?
a. Anemia is rare complication.
b. May be complicated by jaundice.
c. The only cause of cerebral malaria.
d. Not sensitive to chloroquine.
Tim Akademik ANANTARA 2015
Divisi SL (Soal Latihan)
Fakultas Kedokteran Universitas Padjadjaran

e. Can not co-exist with falciparum malaria in the same patient.

157. What is the correct clinical manifestations of lymphatic filariasis?


a. Manifestations of acute infection include transient, recurrent lymphadenitis and
lymphangitis.
b. The early signs and symptoms include episodic fever, lymphangitis of an
extremity, headaches, and myalgias that last a few days to several weeks.
c. Lymphadenitis especially the inguinal and axillary areas.
d. All the statement is true.
e. All the statement is false.

158. Brain tissue is obtained at autopsy from a person who died of rabies.
What procedures could be used to confirm the presence of rabies virus-infected cells in
the brain tissue?
a. Negri inclusion bodies and the presence of viral proteins by immunofluorescence.
b. Detection of rising titers of antibody between acute and convalescent stages of
infection.
c. Virus isolation in chorio-allantoic membrane of embryonated egg.
d. Heme-agglutination inhibition (HI) tests when the disease manifested.
e. Rapid culture techniques whereby viral antigens are detected 2 to 4 days after
inoculation.

159. Zulkfli, a 14-year-old male, previously healthy, presents to the emergency department
with a one week history of fever, malaise, abdominal discomfort, calf pain and a mild
sore throat. He denies any cough, hemoptysis, dyspnea, chills, night sweats, anorexia,
nausea, vomiting, or dysuria. His urine output is good. There is no history of trauma,
blood transfusions, or recent travel.
He lives with his parents. His father is a fisherman and his mother is a housewife. He
likes to swim in streams and canals. He denies substance abuse and sexual activity.
His vital sign is normal except he has a high temperatur of 38.4C,. He is alert in no
acute distress. He is noted to have a moderate conjunctival suffusion, with scleral
icterus. His pharynx is injected, but his tonsils are not enlarged. His neck is supple
with 1 cm anterior lymphadenopathy. Heart is regular with no murmur. His lungs are
clear. His abdomen has normoactive bowel sounds with tender hepatomegaly; but no
rebound or guarding. His extremity muscle strength is 4/5. His lower extremity
muscles are tender to palpation. No nodules are palpable.
Labs: WBC 9,000 with 80% polymorphonuclear leukocytes, hemoglobin 14 g/dL,
platelet count 60,000 /dl. Coagulation times are normal. BUN 70 mg/dL, creatinine 3
mg/dL, total bilirubin 10 mg/dL, direct bilirubin 8 mg/dL, AST 80 UI/L, ALT 70 U/L. .
Urine analysis shows ketones >160mg/dL, >100 red blood cells per high-powered field.
No pyuria, organisms or casts are seen on UA. Chest and abdominal radiographs are
normal.
Tim Akademik ANANTARA 2015
Divisi SL (Soal Latihan)
Fakultas Kedokteran Universitas Padjadjaran

He is admitted to the hospital and started on IV penicillin empirically for possible


leptospirosis. Culture and serology studies are pending.
Therapy of leptospirosis may include all of the following except:
a. Alkalinization of urine.
b. Supportive therapy.
c. Doxycycline.
d. Penicillin.
e. -

160. Nurhasanah, a 19-year-old girl came with her mother to the primary health clinic. They
lived in an epidemic area of filariasis. She heard that everyone should take some drugs
for the disease prevention. The doctor gave Nurhasanah and her mother
diethylcarbamazine citrate (DEC) and albendazole.
The day after Nurhasanah took the drugs, she developed a mild fever, nausea, and
headache.
What is the most likely explanation to this kind of symptoms?
a. Side effect of the drugs.
b. Interaction between DEC and albendazole.
c. Idiosyncratic.
d. Allergy reaction.
e. Immune response due to adult filaria worm death.

161. A 40-year-old man came to your clinic due to white patch on his elbow and numbness
on left hand. Neurological examination show enlargement of left ulnar nerve. From
laboratory examination, biopsy from peripheral nerve found AFB (+). After 2 month of
therapy, the patient came again to you due to complain worsening. Neurologic
examination shows tenderness (+), pain, decreased range of motion, and enlargement of
left ulnar nerve.
The following are true about immunity reaction of the case above:
a. Antigen-specific function is the role of the T-lymphocytes.
b. Cell-mediated immunity can activate the complement system.
c. It is responsible for the delayed hypersensitivity reaction.
d. G-interferon is an important mediator of B-cell activation.
e. Acute graft rejection is caused by cell-mediated immunity.

162. A 70-year-old woman developed herpes zoster ophthalmicus.


Which one of the following is most likely to be a complication of this condition?
a. Hyphaema.
b. Keratitis.
c. Keratoconus.
d. Posterior subcapsular cataract .
e. Scleromalacia.
Tim Akademik ANANTARA 2015
Divisi SL (Soal Latihan)
Fakultas Kedokteran Universitas Padjadjaran

163. Zulkfli, a 14-year-old male, previously healthy, presents to the emergency department
with a one week history of fever, malaise, abdominal discomfort, calf pain and a mild
sore throat. He denies any cough, hemoptysis, dyspnea, chills, night sweats, anorexia,
nausea, vomiting, or dysuria. His urine output is good. There is no history of trauma,
blood transfusions, or recent travel.
He lives with his parents. His father is a fisherman and his mother is a housewife. He
likes to swim in streams and canals. He denies substance abuse and sexual activity.
His vital sign is normal except he has a high temperatur of 38.4C,. He is alert in no
acute distress. He is noted to have a moderate conjunctival suffusion, with scleral
icterus. His pharynx is injected, but his tonsils are not enlarged. His neck is supple
with 1 cm anterior lymphadenopathy. Heart is regular with no murmur. His lungs are
clear. His abdomen has normoactive bowel sounds with tender hepatomegaly; but no
rebound or guarding. His extremity muscle strength is 4/5. His lower extremity
muscles are tender to palpation. No nodules are palpable.
Labs: WBC 9,000 with 80% polymorphonuclear leukocytes, hemoglobin 14 g/dL,
platelet count 60,000 /dl. Coagulation times are normal. BUN 70 mg/dL, creatinine 3
mg/dL, total bilirubin 10 mg/dL, direct bilirubin 8 mg/dL, AST 80 UI/L, ALT 70 U/L. .
Urine analysis shows ketones >160mg/dL, >100 red blood cells per high-powered field.
No pyuria, organisms or casts are seen on UA. Chest and abdominal radiographs are
normal.
He is admitted to the hospital and started on IV penicillin empirically for possible
leptospirosis. Culture and serology studies are pending.
Which clinical factor best distinguishes the life threatening form of leptospirosis from
the more common self-limited form of leptospirosis?
a. Azotemia.
b. Pneumonia.
c. Meningitis.
d. Dehydration.
e. Jaundice

164. What is the false statement regarding typhoid fever management?


a. Adequate rest, hydration, and attention are important to correct fluid and
electrolyte imbalance.
b. A soft, easily digestible diet should be continued unless the patient has abdominal
distention or ileus.
c. Drug of choice for typhoid fever is chloramphenicol with daily dose 50-75
mg/kg/day for 14-21 days.
d. Alternative effective drugs for typhoid fever is cefixime with daily dose 5
mg/kg/day for 7-14 days.
e. -
Tim Akademik ANANTARA 2015
Divisi SL (Soal Latihan)
Fakultas Kedokteran Universitas Padjadjaran

165. Shelly, a 30-year-old 32-34 weeks pregnant woman present with 11 days of fever
which was recurrent in the first 8 days and sustained in the last three days. The fever
accompanied with malaise, anorexia, myalgia, frontal headache, and abdominal pain.
She frequent eats at side walk restaurant surrounding her workplace. On physical
examination reveal acutely ill, relative bradycardia, high fever, abdominal distended
with hepatosplenomegaly, and rose spot on lower chest and upper abdomen. Her CBC
revealed normal range results. Working diagnosis of the patient is typhoid fever.
Which is the most appropriate drug prescription?
a. Chloramphenicol cap 250 mg S4.d.d caps II a.c.
b. Chloramphenicol cap 250 mg S4.d.d caps I a.c.
c. Chloramphenicol cap 250 mg S4.d.d caps I p.c.
d. Chloramphenicol cap 250 mg S4.d.d caps II p.c.
e. Chloramphenicol cap 500 mg S4.d.d caps II p.c.

166. Virus-infected cells often develop morphologic changes referred to as cytopathic effect.
Which one of the following statements best describes the cytopathic effects of viruses
on host cells?
a. Cause morphological changes of the cells.
b. Often associated with changes in mithocondrial membranes.
c. Can be seen using the Gram staining.
d. Rarely associated with cell death.
e. Can only be seen with an electron microscope.

167. A 40-year-old man came to your clinic due to white patch on his elbow and numbness
on left hand. Neurological examination show enlargement of left ulnar nerve. From
laboratory examination, biopsy from peripheral nerve found AFB (+). After 2 month of
therapy, the patient came again to you due to complain worsening. Neurologic
examination shows tenderness (+), pain, decreased range of motion, and enlargement of
left ulnar nerve.
What is the best mechanism related to the case?
a. High cellular immune response.
b. Mast cells activation.
c. Innate immunity reaction.
d. B lymphocyte response.
e. Changes of skin normal flora.

168. There are food poisoning cases among 40 students who gave lunch at Boy Scout
activity. 30 of them get nausea and vomiting, 10 of them got diarrhea and vomiting.
There were true for phase II reactions
a. The P-450 system is located in rough endoplasmic reticulum hepatosit
b. A Flavin containing monooxygen system is located in liver smooth endoplasmic
reticulum
Tim Akademik ANANTARA 2015
Divisi SL (Soal Latihan)
Fakultas Kedokteran Universitas Padjadjaran

c. Prostaglandin H synthase occurs in rough endoplasmic reticulum kidneys


d. Glucoronidation system
e. Biomethylation as a common pathway for xenobiotic metabolism

169. A 5-year-old boy come with complaining diarrhea. He found bloody feces with mucus
since 4 days. From fecal examination, there is no bacteria found. Undergoes serologic
examination done. Indirect hemaglutination: 1:20. What is the interpretation of this
case?
a. Negative.
b. Positive.
c. Borderline.
d. Recurrent infection.
e. On going infection.

170. Male, 37-year-old, presents to hospital with jaundice in the last 4 days. Symptoms
started 7 days before, with gradually onset fever, accompanied by muscle ache
especially in the calves, headache and nausea. Physical examination revealed severe ill
patient with tachycardia, and fever. Conjunctiva was icteric with suffusion. Laboratory
findings revealed thrombocytopenia, leukocyte count of 21,800/mm3, plasma creatinine
concentration of 9.60 mg%, ureum 261 mg%, protein urine +++.
Which is proper laboratory examination bellow?
a. Microscopic agglutination test.
b. IgM anti Dengue.
c. Widal test.
d. Thick blood smear.
e. IgM anti salmonella.

171. A 12-years-old boy who lives in dormitory comes to yor clinic with chief complain
pruritic skin lesion. He feels very itchy especially at night. Skin Lession distribute
generally mainly on the finger webs, wrist, axillary fold and scortum.
On direct microscopical examination from skin scraping revealed mite and fecal pellets.
Sarcoptesscabiei undergoes four stages in its life cycle, mention it:
a. Egg, larva, nymph and adult
b. Egg, nymph, pupa and adult
c. Egg, larva , pupa, and adult
d. Egg, pupa, nymph, and adult
e. -

172. A 25-years-old, male present to the emergency department with chief complain of
having fever for 2 days. He felt the fever occur all day, and only relief after taking
antipyretic drugs. Headache and myalgia were complained as additional symptoms that
occurred. There was no abnormality for urination and defecation.
Tim Akademik ANANTARA 2015
Divisi SL (Soal Latihan)
Fakultas Kedokteran Universitas Padjadjaran

Physical examination:
General : Fever (38oC), chills, fatigue
Skin : Petechial rash on lower extremities
GI : Abdominal pain
Ext : Myalgia
An hour later, the laboratory result of the patient:
WBC 6.6 ; Hb 14.8 ; Hct 41 ; Plt 80,000
Differential:
5% Bands, 42% Segmented Neutrophils, 26% Lymphocytes.
13% Monocytes, 8% Eosinophils, 6% Variably Lymphocytes
INR 1.0 ; PT 11.9 ; PTT 34.9
Electrolytes normal
AST 302 ; ALT 157
Total Bilirubin 0.9, Alkaline Phosphatase 95
Urinalysis unremarkable
Stool occult blood positive
What is the interpretation of the result?
a. Thrombocytopenia, suggestive for dengue hemorrhagic fever.
b. Thrombocytopenia and present of variably lymphocytes, probable for dengue
hemorrhagic fever.
c. Thrombocytopenia and present of variably lymphocytes, suggestive for dengue
hemorrhagic fever.
d. Thrombocytopenia and elevated Liver Enzyme, suggestive for dengue hemorrhagic
fever.
e. Present of variably lymphocytes and elevated Liver Enzyme, probable for dengue
hemorrhagic fever.

173. A 52-year-old man came to you with Laboratory examination results that showed
creatinine level was 4.3 mg/dL. He will be taken antimalaria agent before going to
Papua.
Which of antimalaria agent can be given safely?
a. Quinine.
b. Malarone.
c. Proguanil.
d. Mefloquine.
e. Doxicycline.

174. Infection cases can be caused by its pathogenic agent or its toxin. In this following
diseases is NOT result largely from toxin production:
a. Tetanus.
b. Botulism.
c. Diphteria.
d. Scarlet fever.
Tim Akademik ANANTARA 2015
Divisi SL (Soal Latihan)
Fakultas Kedokteran Universitas Padjadjaran

e. Dengue fever.

175. Public health interventions to prevent typhoid include the provision of safe water
supply. What is the most likely indicator for safe water supply in the extent of
bacteriological parameter?
a. The presence of colored organic matter associated with the soil humus fraction.
b. The number of Salmonella typhii.
c. The growth of microorganisms
d. The increased taste, odor, color & corrosion problems.
e. Thermotolerant coliform bacteria.

176. A 30-year-old man came with 5 fever days accompanied by headache, myalgia and
gastrointestinal disturbances. His wife has the same symptom and diagnosed as
Salmonella typhi infections. His widal titer 1/640. The doctor treat him with
trimeptoprim-sulfametoxasazol.
What is the drug side effect that most likely occurred in this case ?
a. Aplastic anemia.
b. Leucopenia.
c. Trombocytopenia.
d. Agranulocytopenia.
e. Leukemia.

177. Zulkfli, a 14-year-old male, previously healthy, presents to the emergency department
with a one week history of fever, malaise, abdominal discomfort, calf pain and a mild
sore throat. He denies any cough, hemoptysis, dyspnea, chills, night sweats, anorexia,
nausea, vomiting, or dysuria. His urine output is good. There is no history of trauma,
blood transfusions, or recent travel.
He lives with his parents. His father is a fisherman and his mother is a housewife. He
likes to swim in streams and canals. He denies substance abuse and sexual activity.
His vital sign is normal except he has a high temperatur of 38.4C,. He is alert in no
acute distress. He is noted to have a moderate conjunctival suffusion, with scleral
icterus. His pharynx is injected, but his tonsils are not enlarged. His neck is supple
with 1 cm anterior lymphadenopathy. Heart is regular with no murmur. His lungs are
clear. His abdomen has normoactive bowel sounds with tender hepatomegaly; but no
rebound or guarding. His extremity muscle strength is 4/5. His lower extremity
muscles are tender to palpation. No nodules are palpable.
Labs: WBC 9,000 with 80% polymorphonuclear leukocytes, hemoglobin 14 g/dL,
platelet count 60,000 /dl. Coagulation times are normal. BUN 70 mg/dL, creatinine 3
mg/dL, total bilirubin 10 mg/dL, direct bilirubin 8 mg/dL, AST 80 UI/L, ALT 70 U/L. .
Urine analysis shows ketones >160mg/dL, >100 red blood cells per high-powered field.
No pyuria, organisms or casts are seen on UA. Chest and abdominal radiographs are
normal.
Tim Akademik ANANTARA 2015
Divisi SL (Soal Latihan)
Fakultas Kedokteran Universitas Padjadjaran

He is admitted to the hospital and started on IV penicillin empirically for possible


leptospirosis. Culture and serology studies are pending.
The most specific physical finding of in this case include:
a. Fever.
b. Conjunctival suffusion.
c. Renal failure.
d. Myalgia.
e. -

178. Which is an early finding in septic shock?


a. Decreased urine output.
b. Increased cardiac output.
c. Decreased blood pressure
d. Diffuse lung infiltrates
e. -

179. A 1½-year-old unimmunized male child came to emergency department presented with
generalized edema since 6 day. 15 days back he had an episode of fever with bull neck
and generalized tonic-clonic convulsion. his investigation at that time revealed Hb of 10
gm%, WBC count 20,100 cell/cu mm. at this time x-ray chest revealed cardiomegaly
with bilateral infiltrates in both lower zones
What is the role of antimicrobial therapy for this disease?
a. To halt toxin production.
b. Treat localized infection.
c. Prevent transmission of the organism to contacts.
d. All the statement is true.
e. -

180. Thomson, a 45-year-old man, was a resident of Sukabumi city. He has suffered from
malaria since he was a boy. 6 days ago he experienced fever and the doctor told him
that it is malaria that cause the symptom.
The species of malaria associated with adherence to endothelial walls, cerebral malaria,
and a high mortality rate is:
a. P. falciparum.
b. P. vivax.
c. P. malariae.
d. P. ovale.
e. -

181. A 50-year-old woman presented with sudden onset of decreased vision in her better left
eye. She had 1 week of redness, photophobia, pain, and decreased vision. She reported
Tim Akademik ANANTARA 2015
Divisi SL (Soal Latihan)
Fakultas Kedokteran Universitas Padjadjaran

no underlying systemic disease or recent health change and no family history of


unusual eye disease. She was not on medication.
She had developed severe vision loss in her right eye about 20 years prior from
recurrent attacks. She said that she grew up with a lot of cats that roamed the outdoors.
Her right eye was damaged in a motor vehicle accident at some point after it had
developed very poor vision from the parasitic infection.
What the diagnosis of this woman?
a. Diabetic retinophaty
b. Toxoplasmosis
c. Ankylostomiasis
d. Schistosomiasis
e. Glaucoma

182. A 1½-years-old unimmunized male child came to emergency department presented


with generalized edema since 6 day. 15 days back he had an episode of fever with bull
neck and generalized tonic-clonic convulsion. His investigation at that time revealed
Hb of 10 gm%, WBC count 20.100 cell/cu mm. At this time x-ray chest revealed
cardiomegaly with bilateral infiltrates in both lower zones.
The role of antimicrobial therapy for this disease:
a. To halt toxin production
b. Treat localized infection
c. Prevent transmission of the organism to contacts
d. A, b, and c
e. There is no indication for antimicrobial therapy

183. The false statement regarding W. Bancrofti


a. Also known as bancroftian filariasis.
b. Brugia malayi is threadlike nematode that cause similar infection.
c. Is transmitted restricted to several islands of Indonesia.
d. Infective larvae are introduced into human during blood feeding by the mosquito
vector.
e. -

184. There are food poisoning cases among 40 students who gave lunch at Boy Scout
activity. 30 of them get nausea and vomiting, 10 of them got diarrhea and vomiting.
These statement s were true for Toxicity
a. Endogenous chemicals are absorbed after ingestion
b. The site toxicity association with metabolic site of toxic metabolites
c. External dose may be the same as the biologic effective dose
d. Chemicals are metabolized to products more toxic than the parent compound
e. Ceiling effect is the response still increased
Tim Akademik ANANTARA 2015
Divisi SL (Soal Latihan)
Fakultas Kedokteran Universitas Padjadjaran

185. A 36-year-old woman, from Bandung city, present to the clinic with the chief complaint
of fever for 5 days. The fever is high grade, intermittent every each two day. Typically
it is started by chills, followed by high fever and sweating. She had ever gone to
Borneo, two month ago and stayed there for 6 week. Plasmodium vivax were found on
blood smear examination.
The patient then got medicine, and you have quinine only in your clinic.
What is the most likely true due to this drug?
a. It is gametocidal for mature gametocytes of Plasmodium falciparum.
b. Not active against schizonts of Plasmodium malariae.
c. Resistance occurs in Plasmodium falciparum.
d. In the cerebrospinal fluid levels are much higher than serum levels.
e. It can causes hyperglycaemia.

186. Serology is one of the diagnostic methods in Virology.


What is the criterion for diagnosing primary infection in viral diseases?
a. Two fold or more increase in titer of IgG or total antibody between acute and
convalescent sera.
b. Four fold or more increase in titer of IgG or total antibody between acute and
convalescent sera.
c. A single low titer of IgG (or total antibody).
d. Absence or slight increase in IgM.
e. Presence of IgG antibody.

187. When is the best period to obtain blood samples to demonstration of microfilariae in the
blood ?
a. 10 A.M. to 2 P.M.
b. 10 P.M. to 2 A.M.
c. 10 A.M. to 2 P.M.
d. 2 A.M. to 10 P.M.
e. 2 P.M. to 10 P.M.

188. A 16-year-old male presented to the clinic due 4 days of fever. The doctor examined
him and found thrombositopenia with IgM anti dengue positive in his laboratory result.
What is FALSE clinical feature of this disease?
a. Mosquito-borne infection with an incubation period of 2-7 days.
b. Continuous or 'saddle-back' fever.
c. Rigors, headache, and backache.
d. Morbilliform rash and cervical lymphadenopathy.
e. Protection by vaccination every 10 years in endemic areas.

189. A 9-year-old girl presented with decreased vision in the right eye for several weeks.
Tim Akademik ANANTARA 2015
Divisi SL (Soal Latihan)
Fakultas Kedokteran Universitas Padjadjaran

The patient was diagnosed with vitritis and optic neuritis in the right eye and underwent
a short course of systemic steroid therapy.
Laboratory evaluation showed elevated toxoplasma IgG levels and non-detectable
toxoplasma IgM levels. Her only known risk factor for exposure was a family cat.
Toxoplasma infection can occur due to eating:
a. Less ripe goat meat containing oocyst Toxoplasma gondiiInhalation of dust
contaminating egg
b. Less ripe cow meat containing gametocyte Toxoplasma gondii
c. Food contaminated with cat faeces containing oocyst Toxoplasma gondii
d. Food contaminated with cat faeces containing trophozoite Toxoplasma gondii
e. Drinks contaminated with mouse faeces containing oocyst Toxoplasma gondii

190. WHO classified leprosy as paucibacillar and multibacillar. Phenazine dye is given for
multibacillar leprosy patients as alternative to dapsone.
What is the specific characteristic of this alternative drug?
a. Involve RNA binding.
b. Excreted in urine.
c. Stored in reticuloencothelial tissues.
d. Fast release.
e. The serum half life is short.

191. A patient suspected with tick-borne disease experiencing sudden onset of fever with
chills, headache, and back pain. The high fever continuous for 5 days and followed by
afebrile phase for one week, and followed again by another similar cycle.
What type of fever does this patient has?
a. Continous fever.
b. Saddle back ( biphasic ) fever.
c. Intermittent fever.
d. Remittent fever.
e. Relapsing fever.

192. The true statement regarding Felix-Widal test:


a. This test measures agglutinating antibody level against O,H and Vi antigens
b. The test has a good sensitivitiy and specificity
c. Can be negative up to 30% of culture-proven cases of typhoid fever
d. This test still necessary if the diagnosis has already been confirmed by isolation of
S.typhi from a sterile site
e. S. Typhi antigen O and H can not be found in other salmonella serotypes

193. Shock in DHF usually occur on day 3 - 7, suspicion of early shock should be made in
patient which experience :
a. Sudden drop of respiratory rate.
Tim Akademik ANANTARA 2015
Divisi SL (Soal Latihan)
Fakultas Kedokteran Universitas Padjadjaran

b. Sudden drop of pulse rate.


c. Sudden drop of body temperature.
d. Sudden drop of blood pressure.
e. Sudden drop conscienceless.

194. A 1½-year-old unimmunized male child came to emergency department presented with
generalized edema since 6 day. 15 days back he had an episode of fever with bull neck
and generalized tonic-clonic convulsion. his investigation at that time revealed Hb of 10
gm%, WBC count 20,100 cell/cu mm. at this time x-ray chest revealed cardiomegaly
with bilateral infiltrates in both lower zones.
What is the most likely diagnosis of this patient when he came 15 days back?
a. Diphteriae.
b. Peritonsiler abscess.
c. Perimadibular abscess.
d. Croup.
e. Encephalitis.

195. Zulkfli, a 14-year-old male, previously healthy, presents to the emergency department
with a one week history of fever, malaise, abdominal discomfort, calf pain and a mild
sore throat. He denies any cough, hemoptysis, dyspnea, chills, night sweats, anorexia,
nausea, vomiting, or dysuria. His urine output is good. There is no history of trauma,
blood transfusions, or recent travel.
He lives with his parents. His father is a fisherman and his mother is a housewife. He
likes to swim in streams and canals. He denies substance abuse and sexual activity.
His vital sign is normal except he has a high temperatur of 38.4C,. He is alert in no
acute distress. He is noted to have a moderate conjunctival suffusion, with scleral
icterus. His pharynx is injected, but his tonsils are not enlarged. His neck is supple
with 1 cm anterior lymphadenopathy. Heart is regular with no murmur. His lungs are
clear. His abdomen has normoactive bowel sounds with tender hepatomegaly; but no
rebound or guarding. His extremity muscle strength is 4/5. His lower extremity
muscles are tender to palpation. No nodules are palpable.
Labs: WBC 9,000 with 80% polymorphonuclear leukocytes, hemoglobin 14 g/dL,
platelet count 60,000 /dl. Coagulation times are normal. BUN 70 mg/dL, creatinine 3
mg/dL, total bilirubin 10 mg/dL, direct bilirubin 8 mg/dL, AST 80 UI/L, ALT 70 U/L. .
Urine analysis shows ketones >160mg/dL, >100 red blood cells per high-powered field.
No pyuria, organisms or casts are seen on UA. Chest and abdominal radiographs are
normal.
He is admitted to the hospital and started on IV penicillin empirically for possible
leptospirosis. Culture and serology studies are pending.
Good prognostic factors for the patient in our case include all of the following, except:
a. Good urine output.
b. Normal leukocytes.
c. Normal coagulation tests.
Tim Akademik ANANTARA 2015
Divisi SL (Soal Latihan)
Fakultas Kedokteran Universitas Padjadjaran

d. No infiltrates on chest radiography.


e. -

196. A first semester student from Bekasi presented himself to the clinic because of painful
and swollen left leg. Laboratory examination from blood smear with giemsa stain
revealed a number of microfilariae. The doctor diagnose Lymphatic filariasis.
Which of the following statement is correct related for diagnosis?
a. Morphological appearance from adult stage is very important for diagnose
lymphatic filariasis.
b. Clinical manifestation is not important for diagnose lymphatic filariasis.
c. Microfilaria is the diagnostic stage and always revealed inside erythrocyte.
d. Laboratory examination for lymphatic filariasis not only with microscopic
examination.
e. Periodicity from species filariae is not important for diagnose lymphatic filariasis.

197. A 14-year-old girl has a mild sore throat, low grade fever, and a diffuse maculopapular
rash. Physical examination reveals mild tenderness and marked swelling of her
posterior cervical and occipital lymph nodes. Four days after the onset of her illness,
the rash has vanished.
The most likely diagnosis of her condition is:
a. Rubeola.
b. Scarlatina.
c. Rubella.
d. Erythema infectiosum.
e. Henoch Schonlein purpura.

198. A 34-year-old male, from Bandung city, presents to the clinic with the chief complaint
of fever for 15 days. The fever start by low grade fever, and then step ladder fever,
followed by continues high grade fever. He had never gone to the East Indonesian
province or to the beach area. Physical examination revealed alert, high grade fever,
relative bradicardia, and tongue tremor.
What is the possible complication after 2 weeks fever?
a. Intestinal perforation.
b. Hyperglycemia.
c. Lung oedema.
d. Skin haemorage.
e. Hypertension.

199. A group of investigator was studying fever mechanism due to viral infection. They
injected some substance from viral and measure the febrile activator level on each
mouse.
Which of the febrile factor that most likely found in this research?
Tim Akademik ANANTARA 2015
Divisi SL (Soal Latihan)
Fakultas Kedokteran Universitas Padjadjaran

a. TNF.
b. Endotoksin.
c. Antigen antibody complex.
d. INF.
e. CD8.

200. Which statement is false regarding criteria for discharging patients with dengue?
a. Absence of fever at least 24 hours
b. Return of appetite
c. Visible clinical improvement
d. A minimum 2-3 days after recovery from shock
e. All the statement if true

Anda mungkin juga menyukai